Musculoskeletal Imaging: Case Review Series [3 ed.] 0323341357, 9780323341356

Increase your knowledge and improve your image interpretation skills using the proven and popular Case Review approach!

103 79 376MB

English Pages 584 [561] Year 2016

Report DMCA / Copyright

DOWNLOAD FILE

Polecaj historie

Musculoskeletal Imaging: Case Review Series [3 ed.]
 0323341357, 9780323341356

Table of contents :
Series Page
Musculoskeletal Imaging
Copyright
Dedication
Series Foreword
Foreword
Preface
Preface to the First Edition
Case 1
CASE 1
Case 2
CASE 2
Case 3
CASE 3
Case 4
CASE 4
Case 5
CASE 5
Case 6
CASE 6
Case 7
CASE 7
Case 8
CASE 8
Case 9
CASE 9
Case 10
CASE 10
Case 11
CASE 11
Case 12
CASE 12
Case 13
CASE 13
Case 14
CASE 14
Case 15
CASE 15
Case 16
CASE 16
Case 17
CASE 17
Case 18
CASE 18
Case 19
CASE 19
Case 20
CASE 20
Case 21
CASE 21
Case 22
CASE 22
Case 23
CASE 23
Case 24
CASE 24
Case 25
CASE 25
Case 26
CASE 26
Case 27
CASE 27
Case 28
CASE 28
Case 29
CASE 29
Case 30
CASE 30
Case 31
CASE 31
Case 32
CASE 32
Case 33
CASE 33
Case 34
CASE 34
Case 35
CASE 35
Case 36
CASE 36
Case 37
CASE 37
Case 38
CASE 38
Case 39
CASE 39
Case 40
CASE 40
Case 41
CASE 41
Case 42
CASE 42
Case 43
CASE 43
Case 44
CASE 44
Case 45
CASE 45
Case 46
CASE 46
Case 47
CASE 47
Case 48
CASE 48
Case 49
CASE 49
Case 50
CASE 50
Case 51
CASE 51
Case 52
CASE 52
Case 53
CASE 53
Case 54
CASE 54
Case 55
CASE 55
Case 56
CASE 56
Case 57
CASE 57
Case 58
CASE 58
Case 59
CASE 59
Case 60
CASE 60
Case 61
CASE 61
Case 62
CASE 62
Case 63
CASE 63
Case 64
CASE 64
Case 65
CASE 65
Case 66
CASE 66
Case 67
CASE 67
Case 68
CASE 68
Case 69
CASE 69
Case 70
CASE 70
Case 71
CASE 71
Case 72
CASE 72
Case 73
CASE 73
Case 74
CASE 74
Case 75
CASE 75
Case 76
CASE 76
Case 77
CASE 77
Case 78
CASE 78
Case 79
CASE 79
Case 80
CASE 80
Case 81
CASE 81
Case 82
CASE 82
Case 83
CASE 83
Case 84
CASE 84
Case 85
CASE 85
Case 86
CASE 86
Case 87
CASE 87
Case 88
CASE 88
Case 89
CASE 89
Case 90
CASE 90
Case 91
CASE 91
Case 92
CASE 92
Case 93
CASE 93
Case 94
CASE 94
Case 95
CASE 95
Case 96
CASE 96
Case 97
CASE 97
Case 98
CASE 98
Case 99
CASE 99
Case 100
CASE 100
Case 101
CASE 101
Case 102
CASE 102
Case 103
CASE 103
Case 104
CASE 104
Case 105
CASE 105
Case 106
CASE 106
Case 107
CASE 107
Case 108
CASE 108
Case 109
CASE 109
Case 110
CASE 110
Case 111
CASE 111
Case 112
CASE 112
Case 113
CASE 113
Case 114
CASE 114
Case 115
CASE 115
Case 116
CASE 116
Case 117
CASE 117
Case 118
CASE 118
Case 119
CASE 119
Case 120
CASE 120
Case 121
CASE 121
Case 122
CASE 122
Case 123
CASE 123
Case 124
CASE 124
Case 125
CASE 125
Case 126
CASE 126
Case 127
CASE 127
Case 128
CASE 128
Case 129
CASE 129
Case 130
CASE 130
Case 131
CASE 131
Case 132
CASE 132
Case 133
CASE 133
Case 134
CASE 134
Case 135
CASE 135
Case 136
CASE 136
Case 137
CASE 137
Case 138
CASE 138
Case 139
CASE 139
Case 140
CASE 140
Case 141
CASE 141
Case 142
CASE 142
Case 143
CASE 143
Case 144
CASE 144
Case 145
CASE 145
Case 146
CASE 146
Case 147
CASE 147
Case 148
CASE 148
Case 149
CASE 149
Case 150
CASE 150
Case 151
CASE 151
Case 152
CASE 152
Case 153
CASE 153
Case 154
CASE 154
Case 155
CASE 155
Case 156
CASE 156
Case 157
CASE 157
Case 158
CASE 158
Case 159
CASE 159
Case 160
CASE 160
Case 161
CASE 161
Case 162
CASE 162
Case 163
CASE 163
Case 164
CASE 164
Case 165
CASE 165
Case 166
CASE 166
Case 167
CASE 167
Case 168
CASE 168
Case 169
CASE 169
Case 170
CASE 170
Case 171
CASE 171
Case 172
CASE 172
Case 173
CASE 173
Case 174
CASE 174
Case 175
CASE 175
Case 176
CASE 176
Case 177
CASE 177
Case 178
CASE 178
Case 179
CASE 179
Case 180
CASE 180
Case 181
CASE 181
Case 182
CASE 182
Case 183
CASE 183
Case 184
CASE 184
Case 185
CASE 185
Case 186
CASE 186
Case 187
CASE 187
Case 188
CASE 188
Case 189
CASE 189
Case 190
CASE 190
Case 191
CASE 191
Case 192
CASE 192
Case 193
CASE 193
Case 194
CASE 194
Case 195
CASE 195
Case 196
CASE 196
Case 197
CASE 197
Case 198
CASE 198
Case 199
CASE 199
Case 200
CASE 200
Supplemental Figures
CASE 1
CASE 2
CASE 3
CASE 4
CASE 5
CASE 6
CASE 7
CASE 8
CASE 9
CASE 10
CASE 11
CASE 12
CASE 13
CASE 14
CASE 15
CASE 16
CASE 17
CASE 18
CASE 19
CASE 20
CASE 21
CASE 22
CASE 23
CASE 24
CASE 25
CASE 26
CASE 27
CASE 28
CASE 29
CASE 30
CASE 31
CASE 32
CASE 33
CASE 34
CASE 35
CASE 36
CASE 37
CASE 38
CASE 39
CASE 40
CASE 41
CASE 42
CASE 43
CASE 44
CASE 45
CASE 46
CASE 47
CASE 48
CASE 49
CASE 50
CASE 51
CASE 52
CASE 53
CASE 54
CASE 55
CASE 56
CASE 57
CASE 58
CASE 59
CASE 60
CASE 61
CASE 62
CASE 63
CASE 64
CASE 65
CASE 66
CASE 67
CASE 68
CASE 69
CASE 70
CASE 71
CASE 72
CASE 73
CASE 74
CASE 75
CASE 76
CASE 77
CASE 78
CASE 79
CASE 80
CASE 81
CASE 82
CASE 83
CASE 84
CASE 85
CASE 86
CASE 87
CASE 88
CASE 89
CASE 90
CASE 91
CASE 92
CASE 93
CASE 94
CASE 95
CASE 96
CASE 97
CASE 98
CASE 99
CASE 100
CASE 101
CASE 102
CASE 103
CASE 104
CASE 105
CASE 106
CASE 107
CASE 108
CASE 109
CASE 110
CASE 111
CASE 112
CASE 113
CASE 114
CASE 115
CASE 116
CASE 117
CASE 118
CASE 119
CASE 120
CASE 121
CASE 122
CASE 123
CASE 124
CASE 125
CASE 126
CASE 127
CASE 128
CASE 129
CASE 130
CASE 131
CASE 132
CASE 133
CASE 134
CASE 135
CASE 136
CASE 137
CASE 138
CASE 139
CASE 140
CASE 141
CASE 142
CASE 143
CASE 144
CASE 145
CASE 146
CASE 147
CASE 148
CASE 149
CASE 150
CASE 151
CASE 152
CASE 153
CASE 154
CASE 155
CASE 156
CASE 157
CASE 158
CASE 159
CASE 160
CASE 161
CASE 162
CASE 163
CASE 164
CASE 165
CASE 166
CASE 167
CASE 168
CASE 169
CASE 170
CASE 171
CASE 172
CASE 173
CASE 174
CASE 175
CASE 176
CASE 177
CASE 178
CASE 179
CASE 180
CASE 181
CASE 182
CASE 183
CASE 184
CASE 185
CASE 186
CASE 187
CASE 188
CASE 189
CASE 190
CASE 191
CASE 192
CASE 193
CASE 194
CASE 195
CASE 196
CASE 197
CASE 198
CASE 199
CASE 200
Blank Page

Citation preview

Series Editor David M. Yousem, MD, MBA Professor of Radiology Director of Neuroradiology Russell H. Morgan Department of Radiology and Radiological Science The Johns Hopkins Medical Institutions Baltimore, Maryland Other Volumes in the CASE REVIEW Series Brain Imaging, Second Edition Breast Imaging, Second Edition Cardiac Imaging, Second Edition Duke Review of MRI Principles Emergency Radiology Gastrointestinal Imaging, Third Edition General and Vascular Ultrasound, Third Edition Genitourinary Imaging, Third Edition Head and Neck Imaging, Fourth Edition Nuclear Medicine, Second Edition Obstetric and Gynecologic Ultrasound, Third Edition Pediatric Imaging, Second Edition Spine Imaging, Third Edition Thoracic Imaging, Second Edition

Musculoskeletal Imaging CASE REVIEW Third Edition Joseph S. Yu, MD Vice Chair of Academic Affairs and Education Professor of Radiology and Orthopedic Surgery Section Chief of Musculoskeletal Radiology The Ohio State University College of Medicine Columbus, Ohio

1600 John F. Kennedy Blvd. Ste 1800 Philadelphia, PA 19103-2899

MUSCULOSKELETAL IMAGING: CASE REVIEW, THIRD EDITION Copyright © 2017 by Elsevier Inc. All rights reserved.

ISBN: 978-0-323-34135-6

No part of this publication may be reproduced or transmitted in any form or by any means, electronic or mechanical, including photocopying, recording, or any information storage and retrieval system, without permission in writing from the publisher. Details on how to seek permission, further information about the Publisher’s permissions policies and our arrangements with organizations such as the Copyright Clearance Center and the Copyright Licensing Agency, can be found at our website: www.elsevier.com/permissions. This book and the individual contributions contained in it are protected under copyright by the Publisher (other than as may be noted herein).

Notices Knowledge and best practice in this field are constantly changing. As new research and experience broaden our understanding, changes in research methods, professional practices, or medical treatment may become necessary. Practitioners and researchers must always rely on their own experience and knowledge in evaluating and using any information, methods, compounds, or experiments described herein. In using such information or methods they should be mindful of their own safety and the safety of others, including parties for whom they have a professional responsibility. With respect to any drug or pharmaceutical products identified, readers are advised to check the most current information provided (i) on procedures featured or (ii) by the manufacturer of each product to be administered, to verify the recommended dose or formula, the method and duration of administration, and contraindications. It is the responsibility of practitioners, relying on their own experience and knowledge of their patients, to make diagnoses, to determine dosages and the best treatment for each individual patient, and to take all appropriate safety precautions. To the fullest extent of the law, neither the Publisher nor the authors, contributors, or editors, assume any liability for any injury and/or damage to persons or property as a matter of products liability, negligence or otherwise, or from any use or operation of any methods, products, instructions, or ideas contained in the material herein. Previous editions copyrighted 2008, 2001 by Mosby Inc., an affiliate of Elsevier Inc. Library of Congress Cataloging-in-Publication Data Names:Yu, Joseph, author. Title: Musculoskeletal imaging : case review / Joseph S.Yu. Other titles: Case review series. Description:Third edition. | Philadelphia : Elsevier, [2017] | Series: Case review series | Includes bibliographical references and index. Identifiers: LCCN 2016003309 | ISBN 9780323341356 (pbk. : alk. paper) Subjects: | MESH: Musculoskeletal Diseases--diagnosis | Musculoskeletal   System--radiography | Diagnostic Imaging | Diagnosis, Differential | Case   Reports | Examination Questions Classification: LCC RC925.7 | NLM WE 141 | DDC 616.7/07548--dc23 LC record available at http://lccn.loc.gov/2016003309

Executive Content Strategist: Robin Carter Senior Content Development Specialist: Jennifer Ehlers Publishing Services Manager: Patricia Tannian Senior Project Manager: Carrie Stetz Design Direction: Amy Buxton

Printed in China Last digit is the print number: 9 8 7 6 5 4 3 2 1

To my mother, Teiko Yu, for her infinite support.

SERIES FOREWORD I have been very gratified by the popularity and positive feedback that the authors of the Case Review Series have received on the publication of the editions of their volumes. Reviews in journals and online sites as well as word-of-mouth comments have been uniformly favorable. The authors have done an outstanding job in filling the niche of an affordable, easy-to-access, case-based learning tool that supplements the material in THE REQUISITES series. I have been told by residents, fellows, and practicing radiologists that the Case Review Series books are the ideal means for studying for oral board examinations and subspecialty certification tests. Although some students learn best in a noninteractive study book mode, others need the anxiety or excitement of being quizzed. The selected format for the Case Review Series (which consists of showing a few images needed to construct a differential diagnosis and then asking a few clinical and imaging questions) was designed to simulate the board examination experience. The only difference is that the Case Review books provide the correct answers and immediate feedback. The limit

vi

and range of the reader’s knowledge are tested through scaled cases ranging from relatively easy to very hard.The Case Review Series also offers feedback on the answers, a brief discussion of each case, a link back to the pertinent THE REQUISITES volume, and up-to-date references from the literature. In addition, we have recently included labeled figures, figure legends, and supplemental figures in a new section at the end of the book, which provide the reader more information about the case and diagnosis. Because of the popularity of online learning, we have been rolling out new editions on the web. We also have adjusted to the new Boards format, which will be electronic and largely case based. We are ready for the new Boards! The Case Review Series is hosted online at ExpertConsult.com. The interactive test-taking format allows users to get real-time feedback,“pinchand-zoom” figures for easier viewing, and links to supplemental figures and online references. Personally, I am very excited about the future. Join us. David M. Yousem, MD, MBA

FOREWORD The second edition of the Case Review Series Musculoskeletal Imaging by Dr. Joseph Yu was very popular. His reputation as a great teacher led to a well-regarded case-based text that has been used across the country for board preparations and topic review. In this third edition, Dr. Yu has provided all-new case material that provides the latest and greatest techniques for demonstrating musculoskeletal pathology. Current concepts in diagnosis, pathophysiology, and therapy are clearly explicated. It is no wonder that musculoskeletal fellowships are widely

popular these days. We must have knowledgeable, well-trained experts in this area to provide quality interpretations and added value in demonstrating the pathology. I am sure that residents preparing for the Boards will find this volume to be a treasure trove of quality material that will serve them well in front of their reading room screens . . . or at the Boards . . . and beyond. David M. Yousem, MD, MBA

vii

PREFACE The third edition of Musculoskeletal Imaging: Case Review is the most comprehensive book to date in this franchise on this particular topic. Each case includes a series of questions that are similar to those encountered in standardized examinations but still focused on the images presented. Nearly all the images are new to this edition to maintain a freshness to the book. The most exciting change is that, in addition to the test images, the majority of cases have been enhanced with additional images to broaden the latitude of disease expression, and each image is labeled and captioned. There are numerous new t­opics as well as core material essential for any musculoskeletal ­specialist. As with the prior edition, the references have been updated to ­emphasize more recent literature, and the topics have been correlated to the fourth edition of the THE ­REQUISITES: Musculoskeletal ­Imaging by Manester, May, and Disler. I would like to thank the staff at Elsevier, in particular Robin Carter, Jennifer Ehlers, and Gabriella Benner, for their excellent

editorial management. A big “thank you” goes to Carrie Stetz for helping me with the final push to finish the book, and for facilitating the details necessary to produce the beautiful layout seen in the final product. Also, I am privileged to have been able to teach so many students in my career; these encounters remain the source of my youthful demeanor. As always, I am grateful for the unyielding support that my wife, Cindy, has provided me throughout my career, but especially during this last venture. Had she not sat in my office every night while I worked on this book, I am unsure that it would have been finished in time. Lastly, I want to thank Sarah, the jewel of my life, who continues to inspire me by her willingness to work on collaborative projects simply because she knows that it brings me joy to be with her. Joseph S. Yu, MD

ix

PREFACE TO THE FIRST EDITION One of the most difficult challenges in life is discovering the things that motivate you and help keep you going. We are faced with this dilemma in nearly every facet of our daily lives, whether it is professionally or at home. Education is one of my passions. Early in my medical training, I discovered that teaching fulfilled me, and it was at that time that I decided to set a career path in academia. Nearly a decade has now passed and I find myself undaunted by the constant renewal of residents going through our program and the task of taking them from a neophyte clinician to a competent radiologist. I find myself refreshed and rejuvenated by the enthusiasm and inquisitive nature of those whom I am surrounded by, and it is this curiosity that keeps me motivated to maintain a constant vigilance for updated information to pass on. I am thankful to have had the opportunity to contribute to the Case Review Series. I like the format provided in this series because it engages the reader to commit to a diagnosis or formulate a list of differential possibilities before divulging the answer. Not meant to be a comprehensive text, it still allows emphasis of important concepts regarding the entity covered in each chapter. The questions are meant to simulate those that one may encounter in an examination. In keeping with the theme of Dr. Yousem, author of the first book in the Case Review Series, Head and Neck Imaging, I have not cited articles that are necessarily “classics” in musculoskeletal imaging but have emphasized the recent literature. I have chosen representative cases from different areas of osteoradiology and have made an attempt to include as many imaging modalities as possible, with one notable exception—ultrasound. I admit that we rarely perform sonography for musculoskeletal conditions

in my institution, so I have little experience with and even less accessibility to such cases. I want to thank Dr. Donald Resnick, a teacher and colleague who continues to inspire me to reach for new heights. Under the tutelage of Dr. Resnick and his partners, Dr. Mini Pathria and the late Dr. David Sartoris, I gained valuable experience in both teaching and learning. I want to thank Dr. Javier Beltran for instilling in me the desire to become just like him, an accomplished osteoradiologist, when I was a second-year resident. I want to thank my partners, Drs. Carol Ashman and Marcella Dardani, for helping me compile an excellent teaching file from which to draw my cases for this book. I especially want to thank all residents and fellows, past and present, for nurturing my need to teach and responding favorably to my style of teaching. I also would like to thank my chairman, Dr. Dimitrios Spigos, and all my colleagues for their support. I want to thank Liz Corra and Stephanie Donley of W. B. Saunders for their excellent editorial management. I want to thank Nydic Open MRI of Cleveland, Boardman, and Kettering for providing me with many of the cases shown in this book and for continuing to keep my teaching file well stocked.And to Theron Ellinger and John Croyle for their excellent photographic support, and Sandy Baker for her secretarial assistance, my gratitude. Most of all, I want to thank Cindy, my wife, for enduring my idiosyncrasies during this year and for helping and encouraging me to finish despite many distractions.And lastly, to the jewel of my life, my daughter Sarah, thanks for your understanding during the times that I could not play with you. Joseph S. Yu, MD

xi

CASE 1

Figure 1-1

HISTORY:  This is an 11-year-old girl with pain in her shoulder after a fall. 1.  What could be included in the differential diagnosis? (Choose all that apply.) A. Chondroblastoma B. Simple bone cyst C. Eosinophilic granuloma D. Fibrous dysplasia E. Aneurysmal bone cyst 2. Regarding a simple bone cyst, at what age do most pathologic fractures occur? A. 20 years B. 15 years C. 10 years D. 5 years

Figure 1-2

. When do simple bone cysts stop growing? 3 A. When the physis fuses B. They never stop growing C. When the cyst reaches the growth plate D. When they fracture . What is a “fallen fragment” sign? 4 A. An absent piece of cortex indicating a pathologic fracture B. A fracture fragment in the dependent portion of a bone cyst C. A prominent internal septation caused by hemorrhage D. A displaced cortical fragment in the soft tissues after a biopsy

See the Supplemental Figures section for additional figures and legends for this case.

3

ANSWERS CASE 1 Simple Bone Cyst 1. B, C, D, and E. Simple bone cyst, eosinophilic granuloma, fibrous dysplasia, and aneurysmal bone cyst could be included in the differential diagnosis based on age and appearance. But if you notice the “fallen fragment” sign, then the only correct diagnosis would be simple bone cyst. Chondroblastoma is not correct because these lesions occur in the epiphysis or apophysis of a bone. 2. C. Most pathologic fractures occur when the patient is about 10 years old. 3. A. Simple bone cysts stop growing with fusion of the growth plate. 4. B. The “fallen fragment” sign represents a fracture fragment that has settled in the dependent portion of the fluid-filled cavity, thus revealing the true cystic nature of the lesion.

Comment Clinical Information A simple, or unicameral, bone cyst is common, and this lesion is a true fluid-filled cavity with a wall of fibrous tissue. The majority of lesions are asymptomatic. Most of these cysts are discovered in people younger than 20 years, either incidentally or because of a pathologic fracture. Males are twice as likely to have them. In adults, simple cysts have a predilection for the calcaneus and the innominate bones of the pelvis. In children and adolescents, 90% to 95% of cysts involve the long bones.

Figure S1-2  The external rotation view shows a pathologic unicortical fracture involving the medial cortex at the surgical neck (yellow arrow) with a “fallen fragment” sign (red arrow).

Imaging Findings The radiographic appearance of a simple bone cyst is that of a lucent, well-demarcated geographic lesion that has its long axis parallel to the axis of the bone (Figures S1-1 and S1-2). It arises in the metaphysis but may migrate into the diaphysis with skeletal growth.The cyst is broader toward the metaphysis than toward the diaphysis. Abnormal remodeling of the bone is noticeable and the cortex occasionally appears expanded. A rim of sclerosis surrounding the lesion is variable in thickness but may be absent. Cortical disruption and periosteal reaction do not occur unless there is a pathologic fracture through the lesion (Figure S1-3). On magnetic resonance (MR) imaging, the typical features of a simple cyst are low signal intensity on T1-weighted (T1W) images and uniformly increased signal intensity on T2W images. Remember that hemorrhage in the cyst may alter its MR appearance and result in a fluid-fluid level (Figure S1-4).The “fallen fragment” sign represents a fracture fragment that has settled in the dependent portion of the fluid-filled cavity, revealing the true cystic nature of the lesion (Figure S1-5). This finding is pathognomonic when detected but occurs in only 20% of cases. References Makley JT, Joyce MJ. Unicameral bone cyst (simple bone cyst). Orthop Clin North Am. 1989;20:407–415. Polat O, Saglik Y, Adiguzel HE, Arikan M, Yildiz HY. Our clinical experience on calcaneal bone cysts: 36 cysts in 33 patients. Arch Orthop Trauma Surg. 2009;129:1489–1494. Figure S1-1  Anteroposterior (AP) radiograph with internally rotated humerus shows a simple bone cyst abutting the physis of the proximal humerus. The cyst results in endosteal thinning. There is a fragment of bone in the dependent portion of the cyst (arrow).

4

Cross-Reference Musculoskeletal Imaging: The Requisites, 4th ed, 514–516.

CASE 2

Figure 2-1

Figure 2-2

HISTORY:  This young boy is being evaluated for lethargy and muscle weakness

3. What neurologic emergency can occur in patients with lead poisoning? A. Increased intracranial pressure B. Acute paralysis C. Syncopal episodes D. Dermatomal paresthesia

1. What should be included in the differential diagnosis in this case? (Choose all that apply.) A. Heavy metal poisoning B. Radiation C. Hyperthyroidism D. Treated leukemia E. Normal variant 2. How are the MAJORITY of lead poisoning cases discovered? A. Gastrointestinal disorders B. Neurologic disorders C. Screening D. Failure to thrive

4. At what blood concentration of lead will dense metaphyseal bands consistently occur? A. 0 to 10 μg/dL B. 20 to 40 μg/dL C. 50 to 70 μg/dL D. Above 90 μg/dL

See the Supplemental Figures section for additional figures and legends for this case.

5

ANSWERS CASE 2 Dense Metaphyseal Bands 1. A, B, D, and E. Heavy metals such as lead, bismuth, phosphorus, mercury, and arsenic can cause dense bands in the metaphyses. Formation of metaphyseal bands may occasionally be caused by treated leukemia, radiation, or prolonged exposure to sunlight, especially after winter. 2. C. Although children who ingest lead can present with nausea, cramping, vomiting, and constipation, or vague neurologic symptoms, such as headache, learning disability, hyperactivity, memory loss, or muscle weakness, the most readily available detection method is screening. 3. A. Some patients can present with a rapid increase in intracranial pressure leading to signs of encephalopathy, including headaches, seizures, and coma. 4. C. The bands form after the blood concentration reaches 70 μg/dL but have recently been reported to occur consistently in patients with a mean of 50 μg/dL.

Comment Differential Diagnosis The finding of dense metaphyseal bands elicits a differential diagnosis that includes toxic poisoning as well as insult to the growth of the bone (Figures S2-1 and S2-2). Lead poisoning is the most common disease of toxic environmental origin in the United States today. It results from prolonged ingestion of

lead-containing materials, such as paint, ceramics, and drinking water; inhalation of fumes from burning storage batteries; and occasionally from absorption of material from bullets or buckshot. It can be passed to the fetus if the mother is exposed.

Clinical Manifestations Clinically, lead is toxic to organs as well, leading to fatigue, colic, constipation, anemia, peripheral neuropathy, and encephalopathy.The bands form after the blood concentration reaches 70 μg/dL but have recently been reported consistently in patients with a mean of 50 μg/dL (and occasionally as low as 10 μg/dL).

Pathology Some have hypothesized that the lead is laid down within the cartilaginous matrix, a process termed leadification, resulting in the formation of the bands, but they are more likely the result of calcium deposition or increased bone formation. It may be subtle in smaller bones (Figure S2-3). The condition is termed chondrosclerosis, and is depicted as the presence of trabeculae composed of calcified thick cartilaginous cores covered by thin sleeves of endosteal bone almost devoid of osteoblasts. References Raber SA. The dense metaphyseal band sign. Radiology. 1999;211: 773–774. Tuzun M, Tuzun D, Salan A, Hekimoglu B. Lead encephalopathy: CT and MR findings. J Comput Assist Tomogr. 2002;26:479–481.

Cross-Reference Musculoskeletal Imaging: The Requisites, 4th ed, 387–388.

Figures S2-1 and S2-2  Frontal radiographs of both hands in this patient show prominent dense bands in the metaphyses of both distal radius and ulna (yellow arrows) and in the fingers (red arrows).

6

CASE 3

Figure 3-1

HISTORY:  This 24-year-old woman fell on her outstretched arm during an ice storm. 1.  What would be included in the differential diagnosis? (Choose all that apply.) A. Trochlear fracture B. Giant intraarticular body C. Capitellar fracture D. Parosteal osteosarcoma 2. What is the most common mechanism responsible for this injury? A. Fall on an outstretched hand B. Hyperextension of the elbow C. Hyperflexion of the elbow D. Dislocation of the elbow

Figure 3-2

3. Regarding these fractures, what other structure is at high risk for injury? A. Rupture of the radial collateral ligament B. Fracture of the coronoid process C. Rupture of the ulnar collateral ligament D. Tear of the insertion of the biceps tendon . What does disruption of the radiocapitellar line indicate? 4 A. Radial head dislocation B. Supracondylar fracture C. Abnormal carrying angle D. Displaced trochlear fracture

See the Supplemental Figures section for additional figures and legends for this case.

7

ANSWERS CASE 3 Capitellar Fractures 1. C. The appearance is an “Aunt Minnie,” but you need to recognize the findings. Note that the continuity of the cortex is disrupted on the lateral view, with interposition of the radial head. On the frontal view, the cortical margin of the capitellum is absent. The ossific fragment is too inferior to be an intraarticular (IA) body. IA bodies tend to localize more superiorly in either the coronoid fossa or the olecranon fossa. Bodies may also be found in the lateral or medial joint recesses and occasionally in the proximal distal radioulnar joint. 2. A. The most common mechanism of injury is either a direct blow or a fall on the outstretched hand. 3. C. Associated injuries that occur with capitellar fractures are radial head fractures and tears of the ulnar collateral ligament. 4. A. The radiocapitellar line is a reference line bisecting the proximal radial shaft that normally intersects the capitellum regardless of the position of the radius. If it does not intersect the capitellum, the radial head is dislocated or subluxed.

Comment Classification of Fracture Capitellar fractures account for 1% of all elbow fractures, but they remain a difficult diagnosis to observe on radiographs. The most common mechanism of injury is either a direct blow or a fall on the outstretched hand. Fractures of the capitellum are classified into three different types: type 1, a fracture that involves most or all of the capitellum (Figures S3-1 and S3-2);

Figure S3-2  Lateral radiograph shows a semispherical bone fragment anterior to the radial head (yellow arrow) representing a portion of the displaced capitellum. Note the abrupt termination of the cortex posteroinferiorly (red arrow).

type 2, a shearing injury producing an osteochondral defect with an occasional intraarticular body (Figure S3-3); and type 3, a comminuted fracture of the capitellum often associated with a radial head fracture. The difficulty with diagnosing these fractures is that the frontal projection often is not helpful. Type 1 fractures are readily apparent on lateral views because they are usually displaced. Type 3 fractures are best diagnosed by either computed tomography or magnetic resonance imaging. Type 2 fractures are most optimally depicted by magnetic resonance imaging (Figure S3-4), although it may require intraarticular contrast if the defect is purely cartilaginous and there is insufficient joint effusion.

Treatment Treatment is aimed at restoring normal biomechanics of the elbow. If the fragment is sufficiently large, reduction and fixation with small screws, Herbert screws, or Kirschner wires have had good success. Excision is recommended when fragments are too small for fixation. References Sonin A. Fractures of the elbow and forearm. Semin Musculoskeletal Radiol. 2000;4:171–191. Trinh TQ, Harris JD, Kolovich GP, Griesser MJ, Schickendantz MS, Jones GL. Operative management of capitellar fractures: a systematic review. J Shoulder Elbow Surg. 2012;21:1613–1622.

Cross-Reference Musculoskeletal Imaging: The Requisites, 4th ed, 131–132.

Figure S3-1  Frontal radiograph of the elbow shows a normal medial joint space with the articular surfaces of the trochlea and ulna well outlined. However, the articular surface of the capitellum in the lateral joint space is absent, although the radial head surface is clearly depicted (arrow), and the joint space is obscured.

8

CASE 4

Figure 4-1

HISTORY:  This 22-year-old man sustained knee trauma. 1. What is the diagnosis? (Choose all that apply.) A. Anterior dislocation B. Medial dislocation C. Posterior dislocation D. Lateral dislocation E. Rotatory dislocation 2. What makes this an emergent injury? A. Fractures B. Multiple ligament tears C. Nerve injuries D. Vascular injuries

Figure 4-2

3. What should be performed in all patients who do not require an immediate revascularization procedure? A. MR imaging to evaluate the ligaments B. Serial perfusion checks C. CT to evaluate the osseous integrity D. Neurologic consult . What would a foot drop indicate? 4 A. Tibial nerve injury B. Femoral nerve injury C. Medial plantar nerve injury D. Peroneal nerve injury

See Supplemental Figures section for additional figures and legends for this case.

9

ANSWERS CASE 4 Complete Knee Dislocation 1. A. The positional classification of dislocations is based on the position of the tibia relative to the femur. The tibia is anteriorly dislocated in this case. 2. D. A knee dislocation constitutes a true orthopedic emergency since vascular injuries occur in 33% to 40% of patients. Of these, up to 50% of patients require an amputation when vascular lesions are not treated promptly, especially with posterior and posterolateral dislocations. 3. B. Serial perfusion checks should be done on all patients since delayed intimal flap thromboses, arteriovenous (AV) fistulas, and pseudoaneurysms of significance may become evident only with purposeful follow-up and require surgical repair. 4. D. About 14% to 35% of patients present with a peroneal nerve injury, ocurring most frequently in patients with posterior knee dislocations.

Comment Mechanism of Injury A dislocation of the knee is a severe injury that is caused by high-energy trauma. The incidence of complete knee dislocations reportedly is low, although any estimate is speculative because many dislocations reduce spontaneously at the scene of the injury. Therefore a patient who had a knee dislocation may present with extensive ligamentous disruption without an obvious dislocation. Both cruciate ligaments are likely to be torn, although in certain situations the posterior cruciate ligament may be spared. Injuries involving both collateral ligaments and menisci are common as well.

Figure S4-2  Lateral radiograph of the same knee shows that the tibia is completely dislocated anteriorly and is perched against the anterior surface of the femoral condyles (yellow arrow). Note the lipohemarthrosis in the joint (blue arrows).

Classification The injury may be classified according to the position of the tibia relative to the femur.The positional classification uses five types of knee dislocations: anterior (Figures S4-1 and S4-2), posterior (Figures S4-3 and S4-4), lateral, medial, and rotatory types—although combinations are frequent (Figures S4-5 and S4-6). Anterior and posterior dislocations account for the majority of dislocations.

Complications An injury to the popliteal tendon denotes a more severe mechanism of injury and is likely to be the result of a posterior or posterolateral dislocation.A knee dislocation constitutes a true orthopedic emergency owing to the possibility of associated injuries to the popliteal artery or the common peroneal nerve. Vascular injuries occur in 33% to 40% of patients, and up to 50% of patients require an amputation when vascular lesions are not treated promptly, although recent literature suggests a much lower incidence. An emergent lower extremity angiogram is recommended to assess the vascular supply, even if a pulse is palpable in the foot since latent thrombosis may occur from an unsuspected intimal injury. Peroneal nerve injuries occur in 14% to 35% of patients and usually cause permanent functional defects. References

Figure S4-1  Frontal radiograph of the right knee shows malalignment of the tibia relative to the femur. There is overriding of the cortical margins of the femoral condyles with respect to the proximal tibia (arrows).

10

Arom GA, Yeranosian MG, Petrigliano FA, et al. The changing demographics of knee dislocation: a retrospective database review. Clin Orthop Relat Res. 2014;472:2609–2614. Yu JS, Goodwin D, Salonen D, et al. Complete dislocation of the knee: Spectrum of associated soft-tissue injuries depicted by MR imaging. Am J Roentgenol. 1995;164:135–139.

Cross-Reference Musculoskeletal Imaging: The Requisites, 4th ed, 223.

CASE 5

Figure 5-2

Figure 5-1

HISTORY:  A 47-year-old man with a chronic disease. 1.  What should be included in the differential diagnosis? (Choose all that apply.) A. Polyvinyl chloride toxicity B. Hyperparathyroidism C. Collagen vascular disease D. Hadju-Cheney disease E. Epidermolysis bullosa 2. What would be the leading diagnosis if there were associated soft tissue calcifications? A. Polyvinyl chloride toxicity B. Multicentric reticulohistiocytosis C. Diabetes mellitus D. Scleroderma

3. In skeletally immature people, what part of the distal phalanx is most vulnerable to thermal injuries? A. Growth plate B. Tuft C. Epiphysis D. Articular cartilage 4. What neuropathic disorder is associated with bone resorption and calcification of the digital nerves? A. Diabetes mellitus B. Paralysis C. Leprosy D. Neurofibromatosis

See the Supplemental Figures section for additional figures and legends for this case.

11

ANSWERS CASE 5 Acro-osteolysis 1. B, C, and E. The pattern of bone destruction determines the differential diagnosis.A diffuse pattern indicates collagen vascular disease and vasculitis, Raynaud disease, neuropathic disease, thermal injuries, hyperparathyroidism, trauma, epidermolysis bullosa, abnormal stresses, psoriasis, frostbite, sarcoidosis, hypertrophic osteoarthropathy, and pyknodysostosis. Bandlike resorption indicates polyvinyl chloride exposure and Hadju-Cheney disease.

2. D. Collagen vascular disease such as scleroderma would be the most likely diagnosis. 3. A. The growth plate is most vulnerable to thermal injuries in people with an immature skeleton. 4. C. The key to this answer is nerve calcification, which occurs in Hansen’s disease or leprosy.

Comment Clinical Considerations Destruction or resorption of the distal phalanges of the hand describes acro-osteolysis, and this process is associated with numerous etiologies. In many situations, it is impossible to determine the precise cause of the bone lysis when one is asked to rely exclusively on radiographs of the hands. It is important to observe findings elsewhere in the skeleton and to make use of relevant clinical history to render a more specific diagnosis.

Patterns of Acro-osteolysis The pattern of bone lysis is helpful in narrowing the diagnostic considerations. There are three patterns of acro-osteolysis in adults. One pattern preferentially involves the tufts of the terminal phalanges (Figures S5-1 and S5-2). The differential diagnosis for terminal acro-osteolysis is broad. The second pattern (Figure S5-3) preferentially involves the mid-portion of the distal phalanges. Bandlike resorption should bring to mind polyvinyl chloride toxicity and Hajdu-Cheney disease and occasionally hyperparathyroidism, collagen vascular diseases, Gaucher disease, and abnormal stresses. In the latter conditions, resorption of the tuft of the distal phalanges typically coexists. The third pattern (Figure S5-4) is a periarticular distribution seen in patients with late or severe multicentric reticulohistiocytosis. References

Figure S5-1  Frontal radiograph of the right hand shows distal acroosteolysis of the distal phalanges (arrows), including the thumbs. The overall bone density was also slightly diminished. This patient had hyperparathyroidism.

Kemp SS, Dalinka MK, Schumaker HR. Acro-osteolysis: etiologic and radiological considerations. JAMA. 1986;255:2058–2061. Shailesh P,Vernekar J, Pereira S, Desai A. Hajdu-Cheney syndrome: a case report with review of literature. J Radiol Case Rep. 2014;8:1–8.

Cross-Reference Musculoskeletal Imaging: The Requisites, 4th ed, 290, 292.

Figure S5-2  Close-up view of Figure 5-1 shows that resorption of the bone involves more than the tufts of the distal phalanges. There was subtle subperiosteal bone resorption in the radial cortex of the middle phalanges (arrows).

12

CASE 6

Figure 6-1

HISTORY:  A 35-year-old man presents with stiffness. 1.  What could be included in the differential diagnosis? (Choose all that apply.) A. Diffuse idiopathic skeletal hyperostosis B. Ankylosing spondylitis (AS) C. Baastrup disease D. Psoriatic spondyloarthropathy E. Klippel-Feil syndrome 2. What percentage of people with this condition are HLA-B27 positive? A. More than 90% B. 60% to 70% C. 40% to 50% D. Less than 30%

. What is a syndesmophyte? 3 A. Ossification of the ligamentum flavum B. Ossification of the annulus fibrosus C. Ossification of the longitudinal ligaments D. Ossification of the paravertebral soft tissues 4. All EXCEPT which of the following are potential associated spine abnormalities of this disease? A. Pseudoarthrosis B. Atlantoaxial instability C. Cranial settling D. Spondylodiskitis

See the Supplemental Figures section for additional figures and legends for this case.

13

ANSWERS CASE 6 Ankylosing Spondylitis 1. B. This patient shows ankylosis of the sacroiliac joints and apophyseal joints and classic syndesmophytes and ossification of the interspinous ligaments. Diffuse idiopathic skeletal hyperostosis (DISH) is characterized by prominent paravertebral ossification with preservation of disk spaces but does not cause apophyseal and sacroiliac ankylosis. Baastrup disease is characterized by apposition of adjacent spinous processes with resultant enlargement, flattening, and reactive sclerosis of apposing interspinous surfaces but no ankylosis of the spinous process. In psoriasis, the paravertebral ossification appears large, bulky, and asymmetric, often with asymmetric sacroiliitis. 2. A. About 96% of patients with AS are positive for this marker. 3. B. A syndesmophyte is defined as an ossification of the outer fibers of the annulus fibrosus. 4. C. Pseudoarthrosis, fractures, atlantoaxial instability, spondylodiskitis, cord compression, and spinal stenosis may occur in AS. Cranial settling, the protrusion of the dens into the foramen magnum, is a characteristic of rheumatoid arthritis (RA), and affects about 5% to 8% of patients with RA.

Comment Pathologic Considerations This case is an “Aunt Minnie.” AS is the most common seronegative spondyloarthropathy and it has an overwhelming male predilection. The onset of the disease occurs between 15 and 35 years of age and over 90% are HLA-B27 positive. Clinically, these patients present with low back pain and stiffness. As the

disease progresses, prominent thoracic kyphosis and limited lumbar lordosis become evident. The sacroiliac joint is classically the first site involved, and sacroiliitis is a hallmark of the disease. Initial periarticular osteoporosis and superficial cortical erosions on the iliac side of the joint are followed by more dramatic erosive changes resulting in joint space widening. Eburnation develops as a dense ill-defined band of sclerosis. As proliferative changes become more prominent, irregular bony bridges eventually lead to complete joint ankylosis (Figure S6-1). All of these findings occur in both the ligamentous and synovial portion of the joint. The hip joint is affected in 50% of patients with AS, with diffuse joint space narrowing leading to axial migration and acetabular protrusio (Figure S6-2).

Imaging Findings One unmistakable early feature of AS is corner osteitis in the vertebral bodies (Figure S6-3).The term “bamboo spine” characterizes the presence of extensive formation of syndesmophytes (ossification of the annulus fibrosus) (Figure S6-4). The “trolleytrack” sign is seen in frontal radiographs and denotes three vertically oriented dense lines corresponding to ossification of the supraspinous and interspinous ligaments, apophyseal joint capsules, and vertebral bodies. The trolley-track sign may be preceded by the “dagger” sign, which is a single radiodense line on frontal radiographs indicating ossification of the supraspinous and interspinous ligaments.A complication in long-standing disease is a fracture through the spine, often at a disk level, after minor trauma which can lead to a pseudoarthrosis if it is initially undetected (Figure S6-5). References Jang JH, Ward MM, Rucker AN, et al. Ankylosing spondylitis: patterns of radiographic involvement—a re-examination of accepted principles in a cohort of 769 patients. Radiology. 2011;258:192–198. Sieper J, Braun J, Rudwaleit M, Boonen A, Zink A. Ankylosing spondylitis: an overview. Ann Rheum Dis. 2002;61(suppl 3):iii8–iii18.

Cross-Reference Musculoskeletal Imaging: The Requisites, 4th ed, 318–321.

Figure S6-1  Anteroposterior view of the lumbar spine shows complete ankylosis of both sacroiliac joints (yellow arrows) and the “dagger” sign, with ossification of the supraspinous and interspinous ligaments (red arrows).

14

CASE 7

Figure 7-1

HISTORY:  This 74-year-old woman has had back pain for several weeks after minor trauma. 1.  What would you include in the differential diagnosis? (Choose all that apply.) A. Multiple myeloma B. Chondroblastoma C. Kümmell disease D. Eosinophilic granuloma (EG) E. Fracture . What is Kümmell disease, and does this patient have it? 2 A. Interspinous bursitis; no B. Avascular necrosis of the vertebral body; yes C. Ossification of posterior longitudinal ligament; no D. Numerous anteriorly wedged vertebral bodies; yes

3.  What is the likelihood of a concomitant gas-forming infection? A. None B. Somewhat likely C. Very likely D. Findings consistent with diagnosis of osteomyelitis 4. What sarcomatous neoplasm occasionally presents as vertebra plana? A. Ewing sarcoma B. Osteosarcoma C. Chondrosarcoma D. Angiosarcoma

See the Supplemental Figures section for additional figures and legends for this case.

15

ANSWERS CASE 7 Kümmell Disease (Vertebra Plana) 1. A, C, and E. Any neoplasm can cause vertebra plana, especially metastasis, but the adjacent disk spaces are usually preserved and do not contain gas. Avascular necrosis may present with complete collapse of the body, and when gas is identified within the vertebra, the diagnosis is nearly pathognomonic. A collapsed burst fracture can mimic a vertebra plana. EG is a classic cause of vertebra plana in children and young adults, but not in the elderly. 2. B. The most characteristic finding in Kümmell disease, or vertebral avascular necrosis, is gas within the body of the vertebra resulting from a vacuum phenomenon. 3. A. The presence of gas in the body and the absence of lysis of the end plates exclude infection. 4. A. In children, EG is the most common cause of vertebra plana, but you must remember that Ewing sarcoma may also present with this finding.

Comment Differential Diagnosis: Vertebra Plana A flattened vertebral body is referred to as a vertebra plana (Figure S7-1), and the deformity is classically caused by EG, although a number of other conditions can produce a flat vertebral morphology.The abnormality is not difficult to identify, and the diagnosis can be relatively straightforward if certain conditions are applied. In young patients, consider EG, neuroblastoma metastasis, possibly infection, possibly trauma, and rarely Ewing sarcoma; in the elderly, consider metastasis, multiple myeloma, trauma, possibly infection, and possibly lymphoma. With normal disk spaces, consider EG, metastasis, multiple myeloma,

Figure S7-1  Lateral lumbar radiograph shows marked flattening of the L2 vertebral body with retropulsion and linear gas within the body (arrow).

16

possibly Paget disease, and possibly compression fracture. With irregular or destroyed end plate(s), consider infection, tumor (Figures S7-2 and S7-3), possibly trauma, and possibly Scheuermann disease.

Clinical Considerations Kümmell disease is an uncommon spinal disorder characterized by avascular necrosis of a vertebral body, typically occurring in a delayed fashion after minor trauma. The initial force must be strong enough to cause trabecular or endplate injury but is sufficiently minor to escape detection. The interaction between vertebral microtrauma and processes that interfere with osseous repair predisposes to this disorder.

Imaging Findings The radiographic imaging of avascular necrosis of the spine is nonspecific. The vertebral body appears diminished in height, may be increased in density, and contains gas within the body (Figure S7-4). The gas is believed to represent a vacuum phenomenon, similar to gas in healthy joints, which fill in low-pressure intravertebral clefts and is conspicuous on computed tomography (CT) (Figure S7-5). Many patients who develop Kümmell disease receive corticosteroid therapy; however, other typical etiologies of osteonecrosis are major considerations. The observation of intravertebral vacuum clefts excludes acute fracture, infection, or a neoplastic process. References Baghaie M, Gillet P, Dondelinger RF, Flandroy P. Vertebra plana: benign or malignant lesion? Pediatr Radiol. 1996;6:431–433. Bhalla S, Reinus WR. The linear intravertebral vacuum: a sign of benign vertebral collapse. AJR Am J Roentgenol. 1998;170:1563–1569.

Cross-Reference Musculoskeletal Imaging: The Requisites, 4th ed, 349, 352, 521–522.

CASE 8

Figure 8-1

HISTORY:  A 45-year-old man with shoulder pain. 1.  What should be included in the differential diagnosis? (Choose all that apply) A. Tumoral calcinosis B. Hyperparathyroidism C. Calcific tendinitis D. Calcific bursitis E. Heterotopic ossification 2. Where is the focus of calcification most likely located? A. Supraspinatus tendon B. Infraspinatus tendon C. Teres minor tendon D. Subscapularis tendon

Figure 8-2

3. When are patients with this condition most likely to be symptomatic? A. When the calcifications are sharply defined B. When the calcifications become larger C. When the calcifications become multifocal D. When the calcifications are ill defined 4. The composition of the calcified crystal is most likely to be what compound? A. Calcium pyrophosphate dihydrate (CPPD) B. Uric acid C. Calcium oxalate D. Calcium hydroxyapatite

See the Supplemental Figures section for additional figures and legends for this case.

17

ANSWERS CASE 8 Calcific Tendinitis and Bursitis 1. C and D. Calcium hydroxyapatite deposits in tendons appear small and globular, with a “cloudlike” appearance, and those that occur in the subacromial/subdeltoid bursa are often slightly larger. They cause up to 40% of painful shoulder syndromes. Although deposits of tumoral calcinosis are often periarticular, they tend to be much larger calcifications and more amorphous. 2. A. The calcifications are on the bursal side of the supraspinatus tendon. 3. D. Calcifications may be asymptomatic and static in appearance for a long time. However, ill-defined calcifications indicate calcium resorption, the active phase of the disease. 4. D. The composition of calcium in calcific tendinitis and bursitis is calcium hydroxyapatite. CPPD tends to be linear. Uric acid does not typically calcify.

Comment Pathology Calcific tendinitis and subacromial/subdeltoid bursitis are two manifestations of a spectrum of conditions caused by the deposition of calcium hydroxyapatite crystal in the soft tissues. The shoulder joint is the articulation most commonly involved, although virtually any joint may be affected. Periarticular deposits of these crystals are often asymptomatic, but when they are associated with acute inflammation, patients complain of severe pain, swelling, occasional erythema, and fever, mimicking those of a septic joint. Deposits consisting of granular inclusions of calcium hydroxyapatite are associated with necrosis and inflammation.

Imaging Findings Radiographically, these deposits appear ill-defined and cloudlike initially, but become denser and more sharply defined. Ill-defined deposits show histologic evidence of calcium

Figure S8-2  Magnetic resonance (MR) image shows fluid in the subacromial/subdeltoid bursa from bursitis. The calcific deposit (arrow) arose within the supraspinatus tendon and then penetrated into the subacromial/subdeltoid bursa.

resorption, and correlate with symptomatic episodes. The insertion of the supraspinatus tendon is the most frequent site involved, and calcifications are conspicuous near the tendon footplate when the humerus is externally rotated (Figures S8-1 and S8-2). Deposits in the infraspinatus tendon are better depicted in the internal rotation view (Figure S8-3). Structural damage can produce rotator cuff defects and degenerative changes in the joint. Radiographic findings include loss of joint space, destruction of bone, subchondral sclerosis, intraarticular debris, and joint disorganization. Rotator cuff tears may occur and cause migration of the humeral head. Because these deposits may become extruded from the tendon into the surrounding bursa, both tendinitis and bursitis can coexist in the joint (Figures S8-4, S8-5, and S8-6). Calcifications that do not move with the humerus are characteristically located within the subacromial/subdeltoid bursa. References Hayes CW, Conway WF. Calcium hydroxyapatite deposition disease. Radiographics. 1990;10:1031–1048. Jim YF, Hsu HC, Chang CY, Wu JJ, Chang T. Coexistence of calcific tendonitis and rotator cuff tear: an arthrographic study. Skeletal Radiol. 1993;22:183–185.

Cross-Reference Musculoskeletal Imaging: The Requisites, 4th ed, 75–76.

Figure S8-1  An externally rotated view of the shoulder shows a focus of “cloudlike” calcification (arrow) located adjacent to the footplate of the greater tuberosity.

18

CASE 9

Figure 9-1

Figure 9-2

HISTORY:  A 13-year-old boy with progressive knee pain.

3. What percentage of lesions are osteoblastic? A. 30% B. 50% C. 70% D. 90%

1. What conditions would you consider in the differential diagnosis? (Choose all that apply.) A. Osteogenic sarcoma B. Hemophiliac pseudotumor C. Chondromyxoid fibroma D. Osteomyelitis E. Myositis ossificans 2. How would you characterize this lesion? A. Telangiectatic B. Juxtachondral C. Osteoblastic D. Chondroblastic

4. What type of lesions can appear cystic radiographically? A. Central B. Telangiectatic C. Sclerosing D. Periosteal

See the Supplemental Figures section for additional figures and legends for this case.

19

ANSWERS CASE 9 Osteogenic Sarcoma 1. A. There is a large mass in the distal metaphysis of the femur depicting bone matrix formation in the medullary component, and expansion beyond the bone resulting in a prominent Codman triangle.The appearance of a hemophiliac pseudotumor is scalloping and bone lysis from pressure necrosis. Some cases of osteosarcoma can mimic myositis ossificans, but not this case. Infection in young people often arises from the growth plate, which is not involved. Chondromyxoid fibroma does not cause aggressive periostitis and is geographic. 2. C. The answer would be osteoblastic, because this patient’s lesion demonstrates an osseous matrix in the medullary cavity. 3. B. About 50% are osteoblastic, while 25% are chondroblastic and 25% are fibroblastic. 4. B. Telangiectatic osteosarcomas may be completely lytic and have an appearance similar to an aneurysmal bone cyst.

Comment Clinical Presentation Osteosarcoma is the most common primary malignant neoplasm of bone in adolescents and young adults. Overall, it is the second most frequent primary bone lesion (15% to 20%) after multiple myeloma. About three fourths of lesions are the high-grade intramedullary, or conventional, type. The most common clinical

Figure S9-2  The distal femur is the most common location followed by the proximal tibia and proximal humerus. Periosteal reactive changes have variable appearances (arrows).

presentation is pain at the tumor site. It usually begins as an insidious pain but progresses to become severe and constant. As the tumor grows, a palpable soft tissue mass develops when it breaks through the cortex, predisposing the bone to a pathologic fracture.

Imaging Findings This case is typical.There is a large lytic mass in the distal femoral metaphysis associated with a prominent Codman triangle at the margin of the tumor, manifested as a prominent triangular area of periosteal new bone (Figures S9-1 and S9-2).The key observation is the presence of an osseous matrix in the intramedullary region of the tumor, producing variable density in the involved portion of the distal femur (Figures S9-3 to S9-6). Although an osseous matrix characterizes this tumor, the radiographic appearance can range from densely blastic to nearly completely lytic. Most lesions are larger than 5 cm at the initial presentation. Osteoid can also be formed from cartilaginous tissue, which often is present in abundance in this tumor. Only 50% of tumors produce sufficient osteoid to be termed osteoblastic, whereas 25% produce predominantly cartilage (chondroblastic), and 25% produce predominantly spindle cells (fibroblastic). References

Figure S9-1  Frontal radiograph shows a large, lytic lesion in the distal metaphysis of the femur associated with a wide zone of transition (yellow arrows). A Codman triangle is present in the superior edge of the lesion where the periosteum is elevated by the tumor (red arrow). Magnetic resonance imaging (MRI) depicts epiphyseal involvement in 75% of cases, although often not conspicuous on radiographs.

20

Murphey MD, Robbin MR, McRae GA, Flemming DJ, Temple HT, Kransdorf MJ. The many faces of osteosarcoma. Radiographics. 1997;17:1205–1231. Yarmish G, Klein MJ, Landa J, Lefkowitz RA, Hwang S. Imaging characteristics of primary osteosarcoma: nonconventional subtypes. Radiographics. 2010;30:1653–1672.

Cross-Reference Musculoskeletal Imaging: The Requisites, 4th ed, 373–382.

CASE 10

Figure 10-1

HISTORY:  A 65-year-old man presents with paresthesia in the upper extremities. 1.  What should be included in the differential diagnosis? (Choose all that apply.) A. Ankylosing spondylitis B. Ossification of the posterior longitudinal ligament (OPLL) C. Dystrophic spinal calcifications D. Congenital spinal stenosis 2.  What other disease commonly coexists in the spine in patients with this condition? A. Spina bifida or myelomeningocele B. Block vertebra or hemivertebra C. Pseudoarthrosis through the disk D. Diffuse idiopathic skeletal hyperostosis

. What is the cause of this patient’s symptoms? 3 A. Degenerative disk disease B. Spinal stenosis C. Disk herniation D. Senile myelopathy 4. What condition is most commonly associated with OPLL? A. Diabetes B. Gastrointestinal polyps C. Hydrocephalus D. Sacroiliitis

See the Supplemental Figures section for additional figures and legends for this case.

21

ANSWERS CASE 10 Ossification of the Posterior Longitudinal Ligament 1. A and B. The posterior longitudinal ligament appears ossified from C2 to the C7 level, contributing to the thick band of increased density paralleling the vertebral column. Ankylosing spondylitis may be considered because the paraspinal ossification of diffuse idiopathic skeletal hyperostosis mimics syndesmophytes in this patient.This condition is not congenital, so congenital spinal stenosis is incorrect. 2. D. Diffuse idiopathic skeletal hyperostosis (DISH) occurs in 30% to 50% of patients with OPLL. 3. B.This patient has developed true spinal stenosis.The thickness of the ossified posterior longitudinal ligament narrows the sagittal diameter of the spinal canal at multiple levels. When it exceeds 60% of the sagittal diameter, symptoms related to stenosis are very likely to occur.

Clinical Symptoms Symptoms associated with this disorder have been related to cord impingement resulting in motor and sensory disturbances in either the lower or upper extremity. Patients may also present with pain in the neck, shoulder, or arm, and possibly neck stiffness. This case is an “Aunt Minnie” and shows classic features of this condition (Figure S10-1). The OPLL is depicted as a dense strip of bone bridging the posterior aspect of the vertebral bodies from the C2 level to C7 (it occurs most frequently from C3 to C5).There is diffuse narrowing of the diameter of the spinal canal. Magnetic resonance (MR) imaging is useful in characterizing the severity of the condition (Figure S10-2), particularly when there is ossification of the ligamentum flavum as well. Fluid-sensitive sequences are most effective in evaluating the spinal cord compression and detecting abnormal signal intensity of the spinal cord (Figure S10-3). References

4. A. About 28% of patients with OPLL have diabetes, and another 18% are borderline diabetics.

Hirai T, Korogi Y, Takahashi M, Shimomura O. Ossification of the posterior longitudinal ligament and ligamentum flavum: imaging features. Semin Musculoskelet Radiol. 2001;5:83–88. Matsunaga S, Sakou T. Ossification of the posterior longitudinal ligament of the cervical spine: etiology and natural history. Spine. 2012;37:E309–E314.

Comment

Cross-Reference

Incidence

Musculoskeletal Imaging: The Requisites, 4th ed, 272.

OPLL is a distinct clinical entity characterized by the development of ossifications, either in the form of a dense strip of bone or small plaques in the posterior longitudinal ligament. The majority of patients are middle-aged, and there is a 2:1 male to female predilection. The highest prevalence of OPLL is in Japan where it affects 2% of asymptomatic adults.The cause of OPLL is unknown, although genetic factors have been implicated.The disorder frequently coexists in patients with DISH; however, there are distinct pathologic differences between the two entities.

Figure S10-1  Lateral radiograph of the cervical spine shows OPLL resulting in a dense band paralleling the vertebral column (arrows), producing spinal stenosis.

22

CASE 11

Figure 11-1 Figure 11-2

HISTORY:  This 28-year-old woman presented with back pain. Her past medical history is significant for several prior extremity fractures. 1.  What could be considered in the differential diagnosis? (Choose all that apply.) A. Hypervitaminosis D B. Renal osteodystrophy C. Osteopetrosis D. Heavy metal poisoning E. Myelofibrosis 2. What is a common complication in the mandible in patients with osteopetrosis? A. Osteomyelitis B. Malocclusion C. Adamantinoma D. Osteosarcoma

3.  At closer inspection, what finding excludes renal osteodystrophy? A. The sacroiliac joints show subchondral bone resorption B. There are areas in the end plates that are spared C. There is sparing of the humeral heads D. There is calcification in the soft tissues in the ribs . What is the cause of osteopetrosis? 4 A. A gene mutation B. Defective osteoblast function C. Defective osteoclast function D. Fibrous tissue depositing around bone trabeculae

See the Supplemental Figures section for additional figures and legends for this case.

23

ANSWERS CASE 11 Osteopetrosis 1. A, B, C, D, and E. Hypervitaminosis D and renal osteodystrophy can cause diffuse dense bones. Heavy metals such as lead, fluoride, and beryllium can cause dense bones in addition to dense metaphyseal bands in the tubular bones. This patient shows characteristic end-plate sclerosis and bonein-bone appearance in the spine that are typical findings of osteopetrosis. Although myelofibrosis can cause dense bones, splenomegaly is an important component of the diagnosis so it is the least likely of the choices. 2. A. Poor dentition may lead to osteomyelitis of the mandible or maxilla. 3. B. Look closely at the end plates and note that the area of sclerosis does not completely extend across the entire width of the vertebrae. This is characteristic of the “sandwich” vertebrae of osteopetrosis. Note that both humeral heads are sclerotic as well. 4. D. Failure of proper bone resorption characterizes the disease so that the primary spongiosa is not absorbed during enchondral bone formation, resulting in dense bones.

Comment Etiology Osteopetrosis, or Albers-Schönberg disease, is a rare hereditary bone disease caused by lack of absorption of the primary spongiosa in the process of enchondral bone formation. It is suspected that an underlying enzyme deficiency is responsible for this lack of absorption because the vascular mesenchyme that erodes the spongiosa fails to form.

Osteopetrosis Subtypes Clinically, the disease varies in its severity and age of presentation. Four forms of osteopetrosis are recognized. The congenital form is inherited as an autosomal recessive and is lethal. Hepatosplenomegaly and anemia are characteristic of this

Figure S11-2  Lateral radiograph shows end-plate sclerosis (arrows) mimicking a “rugger jersey” spine.

form, and the infant, if not stillborn at birth, fails to thrive. The tarda or adult form is inherited as an autosomal dominant and is clinically benign, although pathologic fractures are a prominent feature of the disease. Organomegaly and anemia do not occur. Poor dentition may lead to osteomyelitis of the maxilla and mandible. An intermediate subtype is autosomal recessive with radiographic features between the congenital and adult subtypes. The fourth subtype is osteopetrosis associated with renal tubular acidosis.

Imaging Findings The key to this case is that the bones are too dense (Figures S11-1 and S11-2). Osteopetrosis is characterized by a symmetric, generalized increase in bone density with loss of distinction between the cortical and medullary bone, particularly in tubular bones. Failure of tubulation results in an Erlenmeyer flask deformity (Figure S11-3). A characteristic finding in the spine is increased density only at the end plates (sandwich vertebrae sign). However, typically, there are areas of normal bone that remain, particularly in the anterior aspect of the end plates (Figure S11-4). The increased density in the skull affects both the base and calvarium, obliterating the diploic space. A bone-in-bone appearance may be evident in the long bones and spine. An important distinction differentiating osteopetrosis from hyperparathyroidism is the presence of normal acromioclavicular and sacroiliac joints. Reference Ihde LL, Forrester DM, Gottsegen CJ, et al. Sclerosing bone dysplasias: review and differentiation from other causes of osteosclerosis. Radiographics. 2011;31:1865–1882.

Cross-Reference Musculoskeletal Imaging: The Requisites, 4th ed, 386–388, 627–629.

Figure S11-1  Chest radiograph shows diffuse osteosclerosis involving the rib cage and both humeri (arrows). Note that the AC joints appear normal.

24

CASE 12

Figure 12-2 Figure 12-1

HISTORY:  A 44-year-old woman fell while ice skating and now has wrist pain. 1. Which of the following would apply in this case? (Choose all that apply.) A. Perilunate dislocation B. Greater arc injury pattern C. Scaphoid fracture D. Scapholunate ligament tear 2. What radiographic finding indicates that surgery may be required? A. Scapholunate interval widening B. Involvement of the scaphoid tubercle C. Oblique fracture orientation D. Greater than 2 mm displacement

3. Which fracture orientation has the greatest risk for developing avascular necrosis (AVN)? A. Longitudinal B. Oblique C. Transverse D. Coronal 4. What percentage of proximal fractures develop nonunion? A. 30% B. 50% C. 70% D. 90%

See the Supplemental Figures section for additional figures and legends for this case.

25

ANSWERS CASE 12 Scaphoid Fracture 1. B, C, and D. There is a fracture at the scaphoid waist with diffuse soft tissue swelling and widening of the scapholunate interval, suggesting a ligament tear. Fractures that involve the greater arc affect a zone of vulnerability that includes the radial styloid, scaphoid, capitate, hamate, triquetrum, and ulnar styloid. 2. D. Displacement usually calls for surgery, and in general this is sufficient for anything greater than 1 mm. Widening of the scapholunate interval when associated with dorsal intercalated segmental instability is an indication. A proximal fracture by itself does not necessarily necessitate surgery. 3. A. Longitudinal fractures promote displacement and have the highest risk for developing AVN. Oblique and transverse fracture orientations are usually stable. 4. D. About one fifth of scaphoid fractures involve the proximal pole. Of these, about 90% will develop nonunion.

Comment Incidence and Mechanism of Injury The most common carpal fracture is a scaphoid fracture, constituting about 60% to 70% of all fractures involving the carpus. Nearly 70% of scaphoid fractures occur at the waist and are nondisplaced. The most common mechanism of injury is a dorsiflexion injury, such as a fall on an outstretched hand. Over 90% of fractures of the scaphoid unite. However, of the one fifth of fractures that involve the proximal pole, 90% will fail to unite.

Treatment and Risk for Avascular Necrosis Treatment is dependent on location, displacement, and associated soft tissue injuries. Nondisplaced fractures are generally treated by cast immobilization. Displaced fractures require reduction (Figures S12-1 and S12-2). If the reduction is unstable, surgical fixation is advocated to avoid the potential complication of nonunion and development of AVN. Owing to the

Figure S12-2  Oblique view of the same wrist shows significant displacement of a scaphoid waist fracture (arrow).

vascular distribution of this bone (the nutrient artery enters the distal scaphoid and traverses the waist before reaching the proximal pole), the risk of developing AVN of the proximal pole increases the more proximal the fracture is located and with a more longitudinal fracture orientation. Increased proximal pole density is indicative of ischemia (Figure S12-3).

Signs of Instability Instability is defined as persistent displacement greater than 1 mm (Figure S12-4), associated dorsal intercalated instability, persistent intrascaphoid angulation of 35 degrees on frontal view or 25 degrees on lateral view, and motion with ulnar and radial deviation. Nondisplaced fractures may be difficult to detect, and magnetic resonance imaging is advocated in patients with ongoing pain (Figure S12-5). Reference Taljanovic MS, Karantanas A, Griffith JF, DeSilva GL, Rieke JD, Sheppard JE. Imaging and treatment of scaphoid fractures and their complications. Semin Musculoskelet Radiol. 2012;16:159–173.

Cross-Reference Musculoskeletal Imaging: The Requisites, 4th ed, 143–145.

Figure S12-1  Frontal view of the wrist shows displacement of the scaphoid fat stripe (yellow arrow) and widening of the scapholunate interval (red arrow).

26

CASE 13

Figure 13-1

Figure 13-2

HISTORY:  A 22-year-old woman has knee pain after trauma.

3. What percentage of patients with GCTs develop a pathologic fracture? A. Less than 1% B. 10% C. 25% D. 50%

1. What entities will you include in the differential diagnosis? (Choose all that apply.) A. Aneurysmal bone cyst B. Giant cell tumor (GCT) C. Chondroblastoma D. Enchondroma E. Telangiectatic osteosarcoma 2. Regarding GCTs, what causes the low signal intensity rim on magnetic resonance (MR) imaging? A. Rim of calcification B. Hemosiderin deposition C. Surrounding fibrosis D. Well-formed capsule

4. What is the estimated rate of recurrence of GCTs after initial resection? A. 50% B. 30% C. 10% D. Less than 1%

See the Supplemental Figures section for additional figures and legends for this case.

27

ANSWERS CASE 13 Giant Cell Tumor 1. A, B, C, and E. The differential diagnosis for a wellcircumscribed lytic lesion that extends to the epiphyseal portion of a long bone in young people includes chondroblastoma, GCT, aneurysmal bone cyst, and occasionally osteosarcoma, chondromyxoid fibroma, and brown tumors. In older people, metastasis becomes an important consideration. 2. B. On MR imaging, GCTs are sharply demarcated from the surrounding marrow by a low signal intensity rim of hemosiderin at the periphery of the tumor. 3. B. Pathologic fractures occur in about 10% of patients with GCTs. 4. A. After curettage, conventional GCTs have a high local recurrence rate of 40% to 60%. This is the reason that surgical excision is the preferred treatment of choice.

Comment Clinical Presentation A GCT is a tumor that can demonstrate benign and malignant features and is sometimes referred to as a quasi-malignant lesion. It is common, constituting 5% of primary bone tumors. The pathologic and radiologic findings do not reflect the potential behavior of a lesion; therefore it is difficult to predict which lesions are likely to be malignant. The incidence of malignancy has been estimated to be about 20%. Most GCTs present in the third or fourth decade of life, and 75% are discovered between

the ages of 20 and 40.Although there is a very slight female predilection for this tumor, the malignant form is three times more likely to occur in men. Typical presenting symptoms include pain and local swelling or a diminished range of motion of a joint.

Imaging Features The radiographic appearance of a GCT is an osteolytic, eccentrically located lesion originating in the metaphysis of the bone (Figures S13-1 and S13-2). In adults, it frequently extends to the articular surface of the bone, but in children the physis may impede the growth of the lesion into the epiphysis. About 10% present with a pathologic fracture (Figure S13-3). Cortical thinning and mild expansion may be evident and can break through the cortex, with soft tissue invasion in as many as 25% of cases (Figure S13-4). MR images depict sharp demarcation with low signal intensity owing to the deposition of hemosiderin at the periphery of the tumor.

Treatment Surgical resection is the treatment of choice. The rate of recurrence after initial resection is estimated to be 40% to 60% ­(Figure S13-5). Reference Chakarun CJ, Forrester DM, Gottsegen CJ, et al. Giant cell tumor of bone: review, mimics, and new developments in treatment. Radiographics. 2013;33:197–211.

Cross-Reference Musculoskeletal Imaging: The Requisites, 4th ed, 509–515.

Figures S13-1 and S13-2  Knee radiographs show an eccentrically located, well-marginated lytic lesion in the metaphyseal and epiphyseal portions of the distal femur (yellow arrows) extending to the articular surface and resulting in marked thinning of the cortex (red arrows). Although it has a narrow zone of transition, there is absence of a sclerotic margin.

28

CASE 14

Figure 14-1

HISTORY:  Sudden pain in a 45-year-old woman. 1.  What would you consider in the differential diagnosis? (Choose all that apply.) A. Lesser trochanter fracture B. Myositis ossificans C. Avulsion fracture D. Osteochondroma E. Pathologic fracture 2. What is the most likely cause of the injury? A. Overuse B. Infection C. Direct trauma D. Indirect trauma

3. What is the most likely diagnosis in this case? A. Lymphoma B. Metastasis C. Osteomyelitis D. Osteosarcoma 4. Regarding avulsion fractures of the pelvis in adolescents, which is INCORRECT? A. Avulsion of the left greater trochanter; gluteus medius B. Avulsion of the pubis; adductors C. Avulsion of the anterior superior iliac spine; sartorius D. Avulsion of the inferior iliac spine; quadratus femoris

See the Supplemental Figures section for additional figures and legends for this case.

29

ANSWERS CASE 14 Acute Pelvic Avulsion Fracture 1. A, C, and E. The pelvis radiograph shows a distracted avulsion of the right lesser trochanter. Note that there are lytic lesions in the intertrochanteric and subtrochanteric regions of the femur as well. The sacroiliac joints are normal. In myositis ossificans, there is peripheral ossification of the heterotopic bone. An osteochondroma shows a continuous cortical margin with the rest of the bone. 2. D. The most likely cause is failure of the pathologic bone from sudden traction by the iliopsoas muscle. 3. B. Although the lesion in the right femur is large, a primary bone tumor is less likely, given that there are other lesions in the pelvis including the left sacrum and the left acetabulum. This patient had breast cancer. 4. D. An avulsion of the anterior inferior iliac spine is caused by the rectus femoris muscle insertion, not the quadratus femoris.

Comment Risk Factors Avulsion fractures of the pelvis are not uncommon in the adolescent or young adult athlete. The activities associated with such injuries include those that require repetitive to-and-fro adduction and abduction, or flexion and extension. Hurdlers, sprinters, and cheerleaders are especially susceptible to this type of injury due to the traction forces exerted by muscles on their insertions. In older adults, a neoplastic process is an important consideration in patients presenting with an avulsion of the lesser trochanter (Figure S14-1).

Pelvic Avulsion Fractures An avulsion of the ischial tuberosity ossification center—the attachment of the hamstring muscle group—is common in hurdlers (Figure S14-2). An avulsion of the rectus femoris muscle origin is another injury (Figure S14-3), often heralded by the presence of small flecks of bone adjacent to the superior acetabular rim (reflected head) or in the region of the inferior anterior iliac spine (straight head). Other common potential sites of avulsion in the pelvis and hips include the anterior superior iliac spine (origin of the sartorius muscle or tensor fasciae femoris) (Figure S14-4), the greater trochanter (insertion of the gluteal cuff musculature) (Figure S14-5), the apophysis of the iliac crest (insertion of the abdominal musculature), and the parasymphyseal region of the pubis (origin of the adductor muscles).

Imaging Findings The radiographic hallmark of avulsion injuries is irregularity of the cortex at the site of avulsion and displaced pieces of bone of variable size. Follow-up radiographs may reveal hypertrophic new bone formation, which can occasionally be associated with marked skeletal overgrowth or deformity mimicking a neoplasm. Generally, only avulsions of the lesser trochanter yield a “knee jerk” response of neoplasm, which may require a biopsy if the diagnosis is unknown. References Sanders TG, Zlatkin MB. Avulsion injuries of the pelvis. Semin Musculoskelet Radiol. 2008;12:42–53. Singer G, Eberl R, Wegmann H, et al. Diagnosis and treatment of apophyseal injuries of the pelvis in adolescents. Semin Musculoskelet Radiol. 2014;18:498–504.

Cross-Reference Musculoskeletal Imaging: The Requisites, 4th ed, 161–162.

Figure S14-1  There is a large lytic lesion in the peritrochanteric region of the right femur (yellow arrow). In addition, notice the large lesion in the left acetabulum extending from the quadrilateral plate to the left ischium, as well as in the left sacrum (red arrows). This patient had metastatic breast cancer.

30

CASE 15

Figure 15-1

Figure 15-2

HISTORY:  A 43-year-old woman with pain in the back of her elbow. 1.  What would be included in the differential diagnosis? (Choose all that apply.) A. Fibromatosis B. Olecranon bursitis C. Subcutaneous hematoma D. Cellulitis 2. All of the following are common treatments for olecranon bursitis EXCEPT? A. Corticosteroid injection B. Nonsteroidal anti-inflammatory drugs C. Incision and drainage D. Resection

3. If this patient had long-standing effusions from repeated trauma to the elbow, what is MOST likely to be lining the bursa adjacent to the bone? A. Focal synovitis B. Fibrous adhesions C. Calcifications D. Thickened capsule 4. Which one of the following bursas is NOT associated with the triceps tendon? A. Superficial olecranon bursa B. Deep or subtendinous olecranon bursa C. Bicipital bursa D. Deep intratendinous bursa

See the Supplemental Figures section for additional figures and legends for this case.

31

ANSWERS CASE 15 Olecranon Bursitis 1. B, C, and D. The fluid collection directly posterior to the olecranon process is most likely a distended superficial olecranon bursa, but occasionally it can be from a hematoma in the soft tissues. Because the olecranon bursa can become infected, cellulitis can surround an inflamed bursa so it can be considered under certain situations. 2. D. Most cases of noninfectious olecranon bursitis are treated with aspiration and corticosteroid injections. In cases of infectious olecranon bursitis, incision and drainage or endoscopic drainage is often performed. Resection of the bursa is reserved for recalcitrant cases of bursitis or those complicated by chronic infection or poor wound healing. 3. B. Long-standing effusions may result in debris in the dependent part of the bursa or fibrous adhesions. Echogenic material within the bursa may indicate an inflammatory, hemorrhagic, or infectious etiology. 4. C. The bicipital bursa is located between the radius and the biceps tendon.

Comment Clinical Presentation The superficial olecranon bursa resides directly posterior to the olecranon process of the proximal ulna.The bursa may become distended with fluid as a result of trauma or infection, or from a synovial inflammatory process such as gout. Clinical diagnosis is not difficult but may be confirmed with imaging (Figures S15-1 to S15-4).

Imaging Fluid in the bursa can be detected radiographically, although small volumes of fluid are generally confirmed either with magnetic resonance (MR) imaging or sonography. Both modalities are sensitive to effusions, synovial proliferation, calcifications, loose bodies, gouty tophi, and septic processes. Sonography has the advantage of dynamic imaging, while MR imaging allows direct inspection of the bone. Septic and nonseptic olecranon bursitis have a considerable imaging overlap,

Figure S15-2  Longitudinal sonogram (right side is distal) of the elbow over the olecranon process with Doppler shows that the echogenic material adjacent to the bone is not hyperemic, and does not suggest an inflammatory condition. The cortex of the olecranon process is hyperechoic (arrow) and associated with acoustic shadowing.

and differentiation often requires the use of either intravenous contrast or direct aspiration.

Sonographic Features On ultrasound, the bursa appears as hypoechoic clefts in the soft tissue, often bounded by a hyperechoic periphery. This patient shows characteristic findings of an uncomplicated, posttraumatic olecranon bursitis. There is distension of the bursa with anechoic fluid with posterior acoustic enhancement. The cortex of the olecranon process is hyperechoic and associated with acoustic shadowing. Long-standing effusions may result in debris in the dependent part of the bursa or fibrous adhesions. Echogenic material within the bursa may indicate an inflammatory, hemorrhagic, or infectious etiology. Power Doppler often shows increased flow within the bursal synovial lining in inflammatory conditions. References Sayegh ET, Strauch RJ. Treatment of olecranon bursitis: a systematic review. Arch Orthop Trauma Surg. 2014;134:1517–1536. Tran N, Chow K. Ultrasonography of the elbow. Semin Musculoskelet Radiol. 2007;11:105–116.

Cross-Reference Musculoskeletal Imaging: The Requisites, 4th ed, 114–116.

Figure S15-1  Transverse sonogram over the olecranon process shows distention of the olecranon bursa with anechoic fluid (asterisk) associated with posterior acoustic shadowing. The echogenic material within the bursa is consistent with fibrous adhesions adjacent to the bone (arrow).

32

CASE 16

Figure 16-1

Figure 16-2

HISTORY:  An 18-year-old man with limited range of motion.

3. What should you consider when a lesion suddenly undergoes painful enlargement? A. Acute pseudobursitis B. Hemorrhage C. Malignant transformation D. Overgrowth of the cartilage cap

1. What entities would be included in the differential diagnosis? (Choose all that apply.) A. Diaphyseal aclasis B. Ollier disease C. Myositis ossificans D. Multiple hereditary exostoses E. Trevor disease 2. How do MOST of these patients present clinically? A. Pathologic fracture B. Painless periarticular mass C. Claudication D. Chronic pain

. What is the risk for sarcomatous transformation? 4 A. Less than 1% B. 2% to 5% C. 25% D. Greater than 50%

See the Supplemental Figures section for additional figures and legends for this case.

33

ANSWERS CASE 16

growth until the skeleton is fully matured. Patients tend to be of short stature. A family history is identified in 70% of cases.

Hereditary Multiple Exostoses

Bony Changes

1. A and D. Diaphyseal aclasis, another name for multiple hereditary exostoses, is characterized by numerous sessile and pedunculated osteochondromas. Trevor disease is characterized by an osteochondroma-like bone growth that occurs in the epiphysis of young people and interferes with joint motion.

Although some osteochondromas may be pedunculated, most are broad-based and sessile like those seen in this patient. Because these osteochondromas frequently involve a large circumference of the metaphyseal region, they can mimic a bone dysplasia (Figure S16-3). Undertubulation of the ends of the long bones often results in a broadened shaft, hence the term diaphyseal aclasis. When the lower extremities are asymmetrically affected, it can produce a compensatory scoliosis. Each patient may have few to several hundred lesions. Lesions growing into the joint can result in significant mechanical erosive changes (Figure S16-4). The formation of pseudobursas occurs in areas of friction (Figure S16-5). The risk for sarcomatous transformation is about 2% to 5%, although the risk for proximal lesions has been reported to be as high as 10%.The risk for any one particular lesion is the same as for an isolated osteochondroma (Figure S16-6).

2. B. The most common clinical presentation is an incidental mass that is palpated in a young child. Pathologic fractures are uncommon since many of the lesions are sessile in nature. Pain is not typical of this condition unless associated with trauma, pseudobursitis, or malignant transformation. 3. C. The most likely consideration in a skeletally mature patient is transformation into an osteosarcoma or chondrosarcoma. Occasionally, a pseudobursa may form over an exostosis where there is repeated friction, and this bursa may become inflamed. This would cause pain and swelling but it would not enlarge the osteochondroma. 4. B. The risk for sarcomatous transformation is about 2% to 5%, although the risk for proximal lesions has been reported to be as high as 10%.

Comment Clinical Features This patient demonstrates classic findings of hereditary multiple exostoses, also known as diaphyseal aclasis, an uncommon autosomal-dominant metaphyseal hyperplastic disorder characterized by the development of multiple osteochondromas. Most patients present with the discovery of single or multiple painless masses near the joints (Figures S16-1 and S16-2), usually in the first decade of life.The lesions typically form and enlarge during

Figure S16-1  Lateral radiograph shows widening of the metaphyseal regions of the distal femur and proximal tibia from broad-based sessile osteochondromas and smaller pedunculated lesions (arrows).

34

Treatment Treatment varies with the clinical symptoms. Local resection is advocated when patients develop mechanical problems, particularly in ambulation. In cases of sarcomatous transformation, aggressive surgical management is recommended. References Pierz KA, Stieber JR, Kusumi K, Dormans JP. Hereditary multiple exostoses: one center’s experience and review of etiology. Clin Orthop Relat Res. 2002;401:49–59. Stieber JR, Dormans JP. Manifestations of hereditary multiple exostoses. J Am Acad Orthop Surg. 2005;13:110–120.

Cross-Reference Musculoskeletal Imaging: The Requisites, 4th ed, 447–449.

Figure S16-2  Ankle radiographs show mechanical erosive changes in the distal fibula from a sessile tibial osteochondroma (arrow) growing into the distal tibiofibular syndesmosis.

CASE 17

Figure 17-2 Figure 17-1

HISTORY:  This 24-year-old man presented with mild neurologic symptoms. 1.  What should be included in the differential diagnosis? (Choose all that apply.) A. Achondroplasia B. Hypochondroplasia C. Thanatophoric dysplasia D. Asphyxiating thoracic dysplasia E. Pseudoachondroplasia 2. What would be the risk for developing the homozygous form of the disease? A. 100% B. 50% C. 25% D. 0%

. In this condition, one skull abnormality is significant. Why? 3 A. Frontal bossing; vision disturbance B. Enlarged skull; propensity for subdural hematomas C. Small foramen magnum; hydrocephalus D. Receding nasal bridge; sinus disease . What is a “trident” hand and when is it most prominent? 4 A. Divergence of the middle and ring fingers in infants B. Divergence of the middle and ring fingers in adults C. Divergence of the index and middle fingers in infants D. Divergence of the index and middle fingers in adults

See the Supplemental Figures section for additional figures and legends for this case.

35

ANSWERS CASE 17 Achondroplasia 1. A, B, and E. This patient is a rhizomelic dwarf with shortening of the proximal limb and has all the classic findings of heterozygous achondroplasia. Hypochondroplasia is a milder form of rhizomelic dwarfism that can occasionally appear similar to achondroplasia. Pseudoachondroplasia is an autosomal-dominant dwarfism that is similar but the skull is normal.Thanatophoric dysplasia causes severe rhizomelia and early death. Asphyxiating thoracic dysplasia causes rhizomelia but also occurs with brachymelia, that is, the long bones are shorter and wider than normal. 2. C. The risk would be the same as for having a normal offspring, or 25%. 3. C. The most important skull abnormality is a small anteriorly displaced foramen magnum, because obstruction of the basal cisterns and aqueduct contributes to communicating hydrocephalus. 4. A. A trident hand occurs in infants with achondroplasia but resolves by childhood.

Figure S17-2  The humerus is short and curved with flared metaphyses (arrows). This is a characteristic appearance in this form of rhizomelic dwarfism.

Comment Clinical Expression and Cause Achondroplasia is a bone dysplasia characterized by rhizomelic, short-limbed dwarfism. Eighty to ninety percent of cases occur as a sporadic mutation of the FGFR3 gene, and the remaining cases are inherited as autosomal dominant.The essential abnormality is a defect in enchondral bone formation affecting all bones formed in cartilage. It is the most common form of dwarfism, with bone deformities that are evident at birth. Intelligence and life span are normal.

Skull and Spine The most important skull abnormality is a small anteriorly displaced foramen magnum, because obstruction of basal cisterns and aqueduct contributes to communicating hydrocephalus.

Other findings include an enlarged skull, frontal bossing, prognathism with a broad mandible, and shortening of the skull base. In the spine, the spinal canal is narrowed in the transverse dimension with progressive narrowing of the interpediculate distance. The vertebral bodies are also shortened in the anteroposterior dimension and show prominent posterior scalloping (Figure S17-1); shortening of the pedicles contributes to significant spinal stenosis (Figures S17-2 and S17-3). The lumbosacral lordosis is exaggerated, resulting in a horizontal sacrum. In some patients, a progressive gibbus deformity at the thoracolumbar spine may compress the spinal cord.

Pelvis and Appendicular Skeleton In the tubular bones, ossification of the periosteum exceeds cartilaginous ossification, causing the new bone to extend beyond the margins of the growth plate. The bone appears short and square with cupped ends (see Figure S17-2). Classic findings are evident in the pelvis (Figure S17-4), where there is shortening of the iliac bones and narrowing of the sacrum. The morphologic changes in the innominate bones result in small and deep greater sciatic notches, squaring of the iliac wings, and flattening of the acetabular angles.This constellation of findings results in the typical “champagne glass” appearance.

Infant Imaging The vertebral bodies may be bullet shaped in infancy. The ribs are shortened and the acetabular roofs are horizontal. Reference Lemyre E, Azouz EM, Teebi AS, et al. Bone dysplasia series. Achondroplasia, hypochondroplasia and thanatophoric dysplasia: review and update. Can Assoc Radiol J. 1999;50:185–197.

Cross-Reference Musculoskeletal Imaging: The Requisites, 4th ed, 632–633.

Figure S17-1  The vertebral bodies show prominent posterior ­scalloping (arrows).

36

CASE 18

Figure 18-1

Figure 18-2

HISTORY:  This 20-year-old woman has had chronic pain and weakness of her right groin.

. What lesions are associated with axial melorheostosis? 3 A. Lipomas of bone and soft tissues B. Hepatosplenomegaly and skin pustules C. Fibromas and fibrous dysplasia D. Fibrolipomatous lesions and arteriovenous malformations

1. What conditions should be included in the differential diagnosis? (Choose all that apply.) A. Tumoral calcinosis B. Sclerosing osteomyelitis of Garre C. Melorheostosis D. Myositis ossificans E. Parosteal osteosarcoma 2. What is the hallmark of this condition? A. Subperiosteal hemorrhage B. Progressive hyperostosis of bone along sclerotomes C. Deposition of fibrous tissue beneath periosteum D. Reactive periostitis to muscle inflammation

4. What is Voorhoeve disease? A. Osteopoikilosis B. Osteopathia striata C. Multiple hamartomas syndrome D. Osteochondromatosis

See Supplemental Figures section for additional figures and legends for this case.

37

ANSWERS CASE 18 Melorheostosis 1. C. Melorheostosis is by far the most likely consideration. It is characterized by the deposition of dense bone along the anterolateral cortex of otherwise normal-appearing ilium and femur with associated periarticular soft tissue ossifications. Tumoral calcinosis and myositis ossificans affect the soft tissues but not the bone. Chronic osteomyelitis affects the bone, but not the soft tissues, but may be considered in cases isolated to one bone.This condition may also mimic parosteal osteosarcoma when focal, but would not be appropriate in this case. 2. B. The hallmark is progressive hyperostosis along a sclerotome distribution. 3. D. Patients with melorheostosis in the axial skeleton may also present with fibrolipomatous lesions and arteriovenous malformations. 4. B. Voorhoeve disease is another name for osteopathia striata, which is manifested by dense linear striations in the metaphyses of bones.

Comment Clinical Information Melorheostosis, a rare nonheritable mesodermal disorder of unknown etiology, usually involves one bone or several bones distributed along the axis of a limb or along the distribution of a nerve. The hallmark of this process is progressive hyperostosis of bone. Melorheostosis usually is recognized in infancy and progresses during childhood and adult life. Chronic

Figure S18-2  Frog-leg lateral image showing that the bone along the femur occupies only one side of the bone (arrows).

progressive pain in an affected limb is typical, and joint stiffness and decreased range of motion contribute to atrophy and weakness of the surrounding musculature. Associated soft tissue abnormalities include joint contractures, fibrosis and edema of the subcutaneous tissue, and varices.

Pathology Histologically, the sclerotic bone is composed of a mixture of immature and mature bone. Interlacing osteoid and thickened trabeculae eventually obliterate the haversian system. Fibrous tissue can be seen within the marrow spaces surrounding areas of new bone proliferation.

Imaging Features Dense linear hyperostosis resembling flowing candle wax is the radiographic hallmark of this disease (Figures S18-1, S18-2, and S18-3). The hyperostosis may advance to the joint margin or even protrude into the joint. Paraarticular soft tissue ossification is uncommon but may be seen in more severely affected persons (see Figures S18-1 and S18-2).Thickening of the cortex and adjacent underlying trabeculae is best demonstrated on computed tomography, whereas medullary involvement is best depicted on magnetic resonance imaging (Figure S18-4). In this case, the findings in the bones and soft tissues are so characteristic that there really is no differential diagnosis. References

Figure S18-1  Frontal radiograph of the hip shows dense bone along the lateral aspect of the ilium (yellow arrows) and amorphous periarticular soft tissue ossifications (red arrows).

38

Greenspan A, Azouz EM. Bone dysplasia series. Melorheostosis: Review and update. Can Assoc Radiol J. 1999;50:324–330. Yu JS, Resnick D, Vaughan L, et al. Melorheostosis with an ossified soft tissue mass: MR features. Skeletal Radiol. 1995;24:367–370.

Cross-Reference Musculoskeletal Imaging: The Requisites, 4th ed, 629–631.

CASE 19

Figure 19-1

Figure 19-2

HISTORY:  A 13-year-old male with left hip pain.

. What is the Klein sign? 3 A. Line bisecting femoral neck divides the femoral head epiphysis in half B. Line drawn along lateral femoral neck intersects the femoral head C. Irregularity of the growth plate D. Focal osteopenia of the metaphysis

1.  What should be included in the differential diagnosis? (Choose all that apply.) A. Femoroacetabular impingement B. Femoral head epiphyseolysis C. Salter 1 injury D. Renal osteodystrophy E. Avascular necrosis (AVN) 2. All of these factors allow slipped capital femoral epiphysis to develop EXCEPT? A. Increase in thyroxine B. Rapid skeletal development C. Increased muscle strength D. Femoral neck develops more varus angulation

4.  Regarding slipped capital femoral epiphysis, what is the resulting deformity in the majority of cases? A. Normal development B. AVN C. Varus deformity with a short, broad femoral neck D. Acute chondrolysis

See the Supplemental Figures section for additional figures and legends for this case.

39

ANSWERS CASE 19 Slipped Capital Femoral Epiphysis 1. B, C, and D. The differential diagnosis for the abnormally positioned femoral epiphysis includes trauma producing a Salter 1 injury, slipped capital femoral epiphysis (SCFE; epiphyseolysis), rickets, and renal osteodystrophy. It may also occur with childhood radiation and hypothyroidism. When the physis closes, the deformity may lead to cam-type femoroacetabular impingement. The epiphysis is normal so AVN is not a consideration. 2. A. Factors that have allowed this condition to develop include rapid skeletal growth, increased muscular strength, and increased varus angulation of the femoral neck. All of these factors accelerate between the ages of 10 and 16 years. A decrease in thyroxine has been linked to this disorder. 3. B. The Klein line is a line drawn along the cortex of the lateral femoral neck. It should cut off about 20% of the lateral portion of the femoral head in a normal situation. This patient has a positive sign, since the line does not intersect the femoral head epiphysis. 4. C. Varus angulation with a short and broad femoral neck is the resultant deformity in the majority of cases. AVN may be seen in about 10% of individuals, usually after attempts at epiphyseal reduction. Acute chondrolysis is a rare event and may mimic infection.

Comment General Information SCFE is a childhood disorder of the hip characterized by posterior and inferior displacement of the proximal femoral epiphysis. It is most common during rapid skeletal growth at 10 to 16 years of age. It is usually a unilateral process, although 20% to 30% of cases are bilateral. Histologically, the abnormality occurs through the zone of hypertrophy in the growth plate. Clinically, patients present with poorly localized pain and a limp. Boys are affected 2 to 3 times more often than girls, although girls tend to present at an earlier age.

Figure S19-2  Frog-leg view shows posteromedial slippage of the epiphysis exposing a step-off between the epiphysis and metaphysis (arrow) and displacement away from the Klein line.

Key Imaging Feature The key to making the diagnosis is having both anteroposterior and frog-leg lateral radiographs available (Figures S19-1 and S19-2). A line drawn tangent to the lateral femoral neck cortex (Klein line) should intersect the femoral epiphysis laterally.This patient demonstrates classic findings. The cartilaginous growth plate is widened and there is irregularity and rarefaction of the metaphysis (Figure S19-3). Close scrutiny of the frog-leg lateral projection reveals posterior slippage of the epiphysis. As slippage progressed, the femoral epiphysis displaced medially, concomitantly widening the growth plate and moving away from the Klein line.

Classification SCFE may be classified according to the duration of symptoms or the degree of slippage. Patients are considered to have acute slips when symptoms have been present for less than 3 weeks. When symptoms last more than 3 weeks, it indicates a more chronic condition, associated with the development of characteristic radiographic abnormalities. The most severe consequence of SCFE is chondrolysis caused by pannus-like granulation tissue that erodes the articular cartilage. This complication predisposes the patient to secondary osteoarthritis. About 10% of patients develop AVN (Figure S19-4). Reference Gill KG. Pediatric hip: pearls and pitfalls. Semin Musculoskeletal Radiol. 2013;17:328–338.

Cross-Reference Musculoskeletal Imaging: The Requisites, 4th ed, 200–201.

Figure S19-1  Anteroposterior (AP) radiograph of the left hip shows widening of the growth plate, focal rarefaction of the metaphysis (arrows), and medial subperiosteal resorption of the femoral neck.

40

CASE 20

Figure 20-1

HISTORY:  A 12-year-old boy with pain in his shoulder. 1.  What should be included in the differential diagnosis? (Choose all that apply.) A. Langerhans cell histiocytosis B. Chondroblastoma C. Osteomyelitis D. Giant cell tumor (GCT) E. Aneurysmal bone cyst 2. Regarding chondroblastomas, what percentage show matrix calcification on computed tomography (CT)? A. None B. 5% to 25% C. 30% to 50% D. 70% to 90%

Figure 20-2

3. All of the following are typical of a chondroblastoma on magnetic resonance imaging (MRI) EXCEPT? A. Periosteal reaction B. Joint invasion C. Fluid-fluid level D. Peritumoral edema . What feature is NOT characteristic of a chondroblastoma? 4 A. Eccentrically located in the epiphysis B. Does not extend into the metaphysis C. Proximal humerus is most common location D. May involve tarsal or carpal bones

See the Supplemental Figures section for additional figures and legends for this case.

41

ANSWERS CASE 20 Chondroblastoma 1. A, B, and C. Lesions that affect the epiphysis or apophysis include chondroblastoma, Langerhans cell histiocytosis, infection, and occasionally, GCT of bone, clear cell chondrosarcoma, and osteoblastoma. In this case, GCT is not considered because of the dense sclerotic rim surrounding the lesion. 2. C. Calcifications may be evident in 30% to 50% of cases on CT. 3. B. Joint invasion is a rare finding with chondroblastoma but joint effusions are very common. About 30% to 50% show periosteal reactive changes, and peritumoral edema is common due to the hypervascularity associated with this lesion. Chondroblastoma is in the differential diagnosis of lesions with fluid-fluid levels on MR images. 4. B. A chondroblastoma may extend into the metaphysis with partial fusion of the epiphyseal plate. The most common location is the proximal humerus followed by the proximal femur, distal femur, and proximal tibia.The carpus and tarsus may be involved because these are apophyseal equivalent bones.

Comment Clinical Considerations Chondroblastoma is a benign tumor of bone characterized by chondroblasts and multinucleated giant cells. Nearly 90% of patients present between the ages of 5 and 25, and there is a twofold male predilection. The most common complaint is pain referring to a joint. Tenderness, swelling, decreased range

Figure S20-2  External rotation shows that the lesion extends to involve the greater tuberosity as well (arrow).

of motion, numbness, muscle atrophy, and weakness may be discovered on physical inspection. The femur (33%), humerus (20%), and tibia (20%) are preferred sites of involvement, although about 10% of chondroblastomas occur in the bones of the hands and feet, especially in the talus and calcaneus.

Imaging Features The radiographic hallmark of this tumor is a well-defined osteolytic lesion that is centrally or eccentrically located within the epiphysis or apophysis of a bone (Figures S20-1 and S20-2). A thin sclerotic rim surrounds the lesion separating it from the normal marrow. Calcifications may be evident in 30% to 50% of cases on CT. Periostitis is notable in one third of all cases and is often seen in the metaphysis on MRI. Aggressive features, such as joint invasion and extension into the adjacent soft tissue and bones and marked enhancement, simulate a more aggressive lesion on MR than on radiographs (Figure S20-3). A fluid-fluid level is often observed. Remember that the tuberosities of the humerus are apophyseal regions (Figure S20-4).

Differential Diagnosis The differential diagnosis includes infection and eosinophilic granuloma, although these lesions usually have their epicenters in the metaphysis of the bone.A GCT generally occurs in the epiphysis of a bone, but lacks matrix calcification or a sclerotic rim. References Douis H, Saifuddin A. The imaging of cartilaginous bone tumours. I. Benign lesions. Skeletal Radiol. 2012;41:1195–1212. Wootton-Gorges SL. MR imaging of primary bone tumors and tumorlike conditions in children. Magn Reson Imaging Clin North Am. 2009;17:469–487. Figure S20-1  Internally rotated view of the shoulder shows a geographic lesion located primarily within the epiphysis of the humerus. A well-defined sclerotic rim surrounds the lesion and matrix calcifications are evident centrally (arrow).

42

Cross-Reference Musculoskeletal Imaging: The Requisites, 4th ed, 452–455.

CASE 21

Figure 21-1

Figure 21-2

HISTORY:  A 49-year-old man with chronic elbow pain.

. What is corneal melt? 3 A. Inflammatory infiltration of the sclera with rheumatoid nodules B. Psoriatic scleritis C. Reiter syndrome D. Degenerative ocular episcleritis from connective tissue disease

1.  What would you include in the differential diagnosis? (Choose all that apply.) A. Hemophiliac arthropathy B. Psoriatic arthritis C. Rheumatoid arthritis (RA) D. Gout E. Neuropathic arthropathy 2. What percentage of patients with RA are eventually serum rheumatoid factor (RF) positive? A. Less than 50% B. 66% C. 85% D. Greater than 99%

4. Felty syndrome includes all of the following EXCEPT: A. RA B. Leukopenia C. Splenomegaly D. Encephalopathy

See the Supplemental Figures section for additional figures and legends for this case.

43

ANSWERS CASE 21 Rheumatoid Arthritis 1. A, C, and E. The key findings are synovial inflammation with soft tissue swelling and effusion as well as an erosive arthritic process that has virtually destroyed the articular surfaces in the knee. The differential considerations would include hemophilia, RA, neuropathic arthropathy, and occasionally, septic arthropathy. Psoriasis and gout are incorrect because the knee is not a typical target site for either arthropathy. 2. C. The serum RF will be present in 85% to 95% of patients with RA but is seen in less than 40% of patients evaluated initially with the disease. RF may be falsely positive in elderly patients. 3. A. Corneal melt refers to scleritis in patients with RA characterized by inflammatory infiltration of the sclera with associated nodular changes that are pathologically identical to rheumatoid nodules, which can lead to disruption of the globe. 4. D. Felty syndrome is an autosomal-dominant genetic disorder characterized by RA, leukopenia, and splenomegaly. Affected patients are susceptible to certain infections. Encephalopathy is not part of this disorder.

Comment RA occurs in 1% of the population and is 3 times more common in women.The peak onset of the disease is in the fourth to sixth decades of life. The earliest pathologic abnormality of RA is acute synovitis, which is associated with synovial congestion and edema that cause hypertrophy and villous transformation of the synovium.

Figure S21-2  Lateral view shows a large effusion (arrows).

Pathology Villous hypertrophy forms papillary fronds measuring 1 to 2 mm, and this may be evident on magnetic resonance imaging (MRI). The irregularity of the synovial tissue is most prominent at the edges of the articular cartilage. Pannus and toxic debris in the synovial fluid cause cartilaginous and osseous destruction. Histologically, cellular infiltration occurring diffusely or in small nodular aggregates, called Allison-Ghormley nodules, may be evident in the superficial portion of the synovium.

Imaging Findings The characteristic radiographic features of RA include soft tissue swelling, periarticular osteoporosis, early joint space narrowing, articular erosions, and marginal erosions (Figures S21-1 and S21-2) caused by the extension of pannus into synovial pockets that expose regions that do not contain a protective cartilaginous layer (bare areas). In the hands, metacarpophalangeal joint involvement is followed by that of the proximal interphalangeal joints (Figure S21-3).An important observation is the lack of reactive bone changes despite marked joint narrowing (Figures S21-4 and S21-5). Although symmetry is the hallmark of RA, several exceptions are notable. Early in the disease process, involvement may be monoarticular or pauciarticular in 5% to 20% of patients. In the hip, diffuse narrowing can result in acetabular protrusion (Figure S21-6). MRI and sonography are both valuable for assessing active synovitis and remission. Reference Vasanth LC, Pavlov H, Bykerk V. Imaging of rheumatoid arthritis. Rheum Dis Clin North Am. 2013;39:547–566.

Cross-Reference Figure S21-1  Frontal radiograph of the knee shows marked osteopenia and marginal erosive changes (yellow arrows). Soft tissue pannus appears dense in the medial aspect of the knee (red arrow).

44

Musculoskeletal Imaging: The Requisites, 4th ed, 290–298.

CASE 22

Figure 22-1

HISTORY:  This is a 22-year-old who was involved in a motor vehicle accident. 1.  What should be included in the differential diagnosis? (Choose all that apply.) A. Hangman’s fracture B. Effendi type 2 C2 fracture C. Congenital spondylolysis of C2 D. Bilateral C2 pedicle fractures 2. What does acute anterior displacement of C2 on C3 indicate? A. Disk protrusion B. Injury of the posterior longitudinal ligament C. Spondylosis D. Bilateral facet joint dislocation

3. What mechanism of injury is MOST likely? A. Shearing force B. Axial C. Hyperflexion of the head D. Acute hyperextension of the head 4. What percentage of people with this injury present with neurologic deficits? A. 10% B. 25% C. 50% D. 90%

See the Supplemental Figures section for additional figures and legends for this case.

45

ANSWERS CASE 22 Hangman’s Fracture 1. A, B, and D. The findings on the radiograph are prevertebral soft tissue swelling, fractures through the neural arch of C2 bilaterally with posterior displacement of the lamina and spinous process, and anterolisthesis and angulation of C2 on C3.These findings are compatible with an acute Effendi type 2 hangman’s fracture. 2. B. Traumatic spondylolisthesis occurs from rupture of the posterior longitudinal ligament, which occurs with a hyperflexion injury mechanism. There is usually asymmetric widening of the disk, anterior translation by greater than 3 mm, and greater than 10 degrees of angulation. 3. D. The majority of hangman’s fractures are the result of acute hyperextension of the head on the neck, although some of these fractures are the product of hyperflexion and axial compression. 4. A. About 10% of patients with hangman’s fractures present with neurologic deficits.

Comment Mechanism of Injury A bilateral fracture of the neural arch of C2 is called a hangman’s fracture. It is usually caused by a hyperextension injury, although flexion-compression and flexion-distraction injuries occasionally may cause this fracture. The vast majority are caused by motor vehicle accidents, with an unrestrained driver’s or passenger’s head striking the dashboard. Because there is no encroachment of the spinal canal, neurologic deficits are uncommon in this injury and when present are generally not permanent.

Table 22-1 

TYPES OF HANGMAN’S FRACTURE

Type 1 • 3 mm translation and >10 degrees angulation • These fractures are apparently caused by hyperflexion and are unstable • Fracture is manifested by pars fracture, anterior displacement of C2 body, and disruption and asymmetric widening of C2-C3 disk space as well as soft tissue swelling • C2-C3 disk and posterior longitudinal ligament are disrupted • Anterior longitudinal ligament usually remains intact Type 3 • Includes all characteristics of type 2 fracture as well as bilateral interfacetal dislocation • May require open reduction of facet dislocation and halo immobilization for the pedicle injury • Has angulation, translation, and also unilateral or bilateral facet dislocation at C2-C3

Effendi Types Hangman’s fractures have been classified by Effendi et al. and later modified by Levine-Edwards into three types, based on the relationship of C2 on C3. In type 1 fractures, there is no forward displacement of C2 on C3, and the fractures involve only the posterior elements (Figure S22-2). In type 2 fractures, there is anterior displacement of C2 on C3 exceeding 3 mm and with a 15-degree angulation (Figures S22-1 and S22-2). In type 2a, there is severe angulation without translation. In type 3 fractures, there is anterior displacement of anterior fragment and facet dislocation, or a locked facet, at C2-C3 (Figure S22-3).Type 3 fractures typically have a flexion component to their injury. Generally, both types 2 and 3 require halo immobilization. Surgical stabilization is recommended for types 2a and 3 fractures (Figure S22-4).

Associated Fractures Other fractures of the cervical spine also are common in patients with a hangman’s fracture. Fractures of the C1 arch occur in 15% of cases, fractures of the body of C2 occur in another 15% of cases (Figure S22-5), and 10% of patients also have a fracture in the thoracic spine. References Bransford RJ,Alton TB, Patel AR, et al. Upper cervical spine trauma. J Am Acad Orthop Surg. 2014;22:718–729. Li XF, Dai LY, Lu H, et al. A systematic review of the management of hangman’s fractures. Eur Spine J. 2006;15:257–269.

Cross-Reference Musculoskeletal Imaging: The Requisites, 4th ed, 170–171.

Figure S22-1  Lateral radiograph of the cervical spine shows vertical fractures (yellow arrow) through the pedicles of the C2 vertebra with distraction resulting in posterior displacement of the lamina and spinous process. The C2 body is subluxed anteriorly and the C2-C3 disk space appears narrowed due to the angular deformity (red arrow).

46

CASE 23

Figure 23-1

Figure 23-2

HISTORY:  Wrist pain in a 24-year-old construction worker.

. What anomaly of the lunate vascular supply increases the risk? 3 A. 5% have a single vessel volarly or dorsally B. 10% have no feeding vessel C. 20% have an aberrant vessel D. 30% have a single vessel volarly and dorsally

1.  What should be included in the differential diagnosis? (Choose all that apply.) A. Lunatomalacia B. Kienböck disease C. Posttraumatic lunate osteonecrosis D. Paget disease of the lunate 2. What is the MOST COMMON cause of this condition? A. Idiopathic B. Repetitive trauma C. Perilunar dislocation D. Atherosclerosis

4. When indicated, surgical procedures for Kienböck disease include the following EXCEPT: A. Radial shortening B. Carpal fusion C. Lunate resection D. Ulnar shortening

See the Supplemental Figures section for additional figures and legends for this case.

47

ANSWERS CASE 23 Kienböck Disease 1. A, B, and C. Kienböck disease, or lunatomalacia, is true necrosis of the lunate that occurs from macrotrauma, repetitive microtrauma, or ulnar minus wrist variance. Paget disease rarely occurs in the carpus or tarsus. 2. B. The most common cause of this condition is believed to be repetitive trauma, although a single traumatic event with a lunate fracture can also cause necrosis. Patients with an ulnar minus variance are at risk for developing Kienböck disease as well. 3. D. About 70% of people have multiple vessels feeding the lunate either volarly or dorsally. In the other 30%, only a single vessel is present volarly and dorsally, which increases their vulnerability to avascular necrosis. 4. D. Radial shortening and ulnar lengthening are the most common surgical procedures. Occasionally, the lunate is excised or replaced, or the carpus is fused. Ulnar shortening is not a typical procedure, because many patients already have a short ulna.

Comment

Figure S23-2  Oblique view shows no pathologic tilt of the lunate (asterisk).

Risk Factors Isolated lunate fractures are uncommon; however, this carpal bone is susceptible to Kienböck disease—avascular necrosis of the lunate. The loss of the blood supply to the lunate has been attributed to primary fracture, repetitive trauma causing microfractures, and traumatic injury to the ligaments that supply blood to the lunate. However, there is also a statistical association among patients with an ulnar minus variance. The morphology of the lunate may also influence the severity, with type 1 lunates being more vulnerable to a coronal fracture and scaphoid flexion deformity than type 2 lunates.

Imaging Diagnosis Initially, the lunate may appear normal, but with time the bone becomes increasingly more sclerotic (Figures S23-1 and S232). Magnetic resonance imaging (MRI) is useful in diagnosing the disease process earlier in patients with central wrist pain, because signal intensity changes can be observed before radiographic changes occur. As necrosis continues, there is eventual loss of height, fragmentation, and subsequent collapse of the lunate.

Classification Lichtman and colleagues devised a staging classification that identifies four different stages. In stage I, the radiographs are normal but MRI may show areas of altered signal intensity or a subchondral fracture. In stage II, the density of the bone increases. In stage III, the subchondral bone collapses.Two subtypes exist in stage III. In stage IIIA, there is lunate collapse but normal scaphoid rotation (Figure S23-3). In stage IIIB, the scaphoid rotation is fixed and there may be advanced capitate collapse (Figure S23-4). In stage IV, arthritic changes develop throughout the carpus. Reference Arnaiz J, Piedra T, Cerezal L, et al. Imaging of Kienböck disease. AJR Am J Roentgenol. 2014;203:131–139.

Cross-Reference Musculoskeletal Imaging: The Requisites, 4th ed, 145, 351.

Figure S23-1  Frontal radiograph of the hand shows increased density in the lunate (yellow arrow) and widening of the scapholunate interval (red arrow). The lunate morphology is otherwise normal. This patient had an ulnar neutral variance.

48

CASE 24

Figure 24-1

Figure 24-2

HISTORY:  This 63-year-old man had an emergent procedure and then developed hip pain.The radiographs are 10 days apart.

3. What organisms are responsible for the majority of septic arthritis cases in the United States? A.  Pseudomonas and Klebsiella B.  Staphylococcus and Streptococcus C.  Staphylococcus and gonococcus D.  Escherichia coli and Streptococcus

1. What conditions should be included in the differential diagnosis? (Choose all that apply.) A. Posttraumatic lysis B. Synovial inflammatory arthropathies C. Neuroarthropathy D. Septic arthritis 2. If this patient has a fever, what is the BEST next step? A. Leukocytosis evaluation B. Joint aspiration C. Blood culture D. Bone biopsy

. What is characteristic of tuberculous septic arthritis? 4 A. Development of avascular necrosis B. Rapid loss of joint space C. Joint ankylosis D. Much slower cartilage destruction

See the Supplemental Figures section for additional figures and legends for this case.

49

ANSWERS CASE 24 Septic Arthritis 1. A and D. Infection can result in rapid lysis of bone, characteristically involving both sides of the joint. It usually is associated with surrounding soft tissue swelling and constitutional symptoms such as fever and leukocytosis. Posttraumatic bone lysis can be a consideration if there was an unsuspected impaction injury of the femoral head. A synovial inflammatory arthritis (i.e., calcium pyrophosphate dihydrate deposition syndrome) and neuroarthropathy would not change so dramatically over 10 days. 2. B. The only specific test is to aspirate the joint and send the fluid for culture as well as a Gram stain. If the joint is infected, the white blood count will surely be elevated. Although a blood culture should be performed as well, it is not as specific as a joint aspiration and is often negative when patients are already on antibiotics. 3. C. Staphylococcus and gonococcus are the infectious agents responsible for the majority of septic arthropathy cases. 4. D. With tuberculous infections, the destruction of cartilage and the development of erosive changes occur more slowly than in with Staphylococcus infections.

Comment

Figure S24-2  Radiograph 10 days later shows progression of femoral head collapse (red arrow) and additional marginal erosions (yellow arrows).

Clinical Considerations Infection of a joint is a serious problem. It is a condition that should be diagnosed rapidly, particularly in immunocompromised patients. Staphylococcus and gonococcus are the infectious agents responsible for the majority of cases of septic arthropathy. Clinically, patients complain of pain, erythema, soft tissue swelling, and an effusion. Patients may present with leukocytosis and fever. Septic arthritis should be considered when inflammation is observed after an invasive procedure has been performed around a joint.This patient had a coronary

angiogram with a left femoral artery access site prior to developing hip pain.

Imaging Findings This patient demonstrates classic signs of advanced disease (Figures S24-1 and S24-2). The initial findings were related to synovial inflammation with focal periarticular osteopenia, but cartilage destruction was already evident in the weight-bearing portion of the joint. Osseous destruction is often rapid once a pannus has formed (Figure S24-3). The joint became irregularly narrowed as the pannus penetrated through the cartilage or into the recesses of the joint, producing central and marginal erosions. In the appendicular skeleton, periostitis in the periarticular region of the infected bone is a common finding, although often subtle.

Course of Action An aspiration is the most important test that should be performed in patients suspected of having an infected joint. Although this procedure may not always identify the bacterial agent, it will demonstrate the presence of leukocytes, an elevated protein count, and a low sugar level that are diagnostic of a septic arthropathy. Magnetic resonance imaging is useful for confirming the marrow edema and in further delineating the extent of involvement of the soft tissues. Bone scintigraphy will show uptake in areas of increased bone metabolism in the delayed image (Figure S24-4) as well as hyperemia in the blood pool phase of the study. Cross-Reference Musculoskeletal Imaging: The Requisites, 4th ed, 485-486.

Figure S24-1  Frontal hip radiograph shows superior joint narrowing with marginal erosions (arrows). An incision and drainage was recently performed after a joint aspiration confirmed infection.

50

CASE 25

Figure 25-1

Figure 25-2

HISTORY:  A 42-year-old man presents with shoulder pain that is exacerbated by abduction and internal rotation on physical examination.

3. Why is it important to keep the transducer perpendicular to the tendon? A. Because the orientation of the tendon fibers can influence the echogenicity B. Because atrophy may not be detected C. Because the orientation is easily lost in the shoulder D.  Because even slight angulation can create artifactual hypoechoic to anechoic defects

1. What is the BEST diagnosis? A. Calcific tendinitis B. Biceps tendon dislocation C. Rotator cuff tear D. Greater tuberosity fracture . What is a “cartilage interface” sign? 2 A. Hyperechoic cartilage from fibrillation B. Acoustic shadowing from cartilage deposition C. Cartilage protruding through a defect in the rotator cuff D. Two parallel hyperechoic lines underneath a cuff tear

. What does a “sagging peribursal fat” sign mean? 4 A. Effacement of the subacromial fat B. Hyperechoic fat replacing the rotator cuff in chronic tears C.  Herniation of peribursal fat into a cuff defect with compression D. Displacement of subacromial fat from synovitis in the bursa

See the Supplemental Figures section for additional figures and legends for this case.

51

ANSWERS CASE 25 Rotator Cuff Tear (Supraspinatus) 1. C. The sonographic findings are consistent with a defect in the supraspinatus insertion associated with retraction of the tendon. Calcific tendinitis can cause cuff tears, but the calcium deposits reflect sound waves and appear as a hyperechoic focus with acoustic shadowing. When in the tendon, the calcification creates enlargement of the tendon, not attenuation. 2. D. The parallel hyperechoic lines underneath the cuff are caused by the interface between the joint fluid and the hyaline cartilage that is produced by increased through-transmission of the ultrasound beam through the rotator cuff defect and the humeral cortex. It indicates a cuff tear. 3. D. Accurate diagnosis requires careful scanning techniques with proper positioning of the transducer at all times. 4. C. Herniation of the peribursal fat into the rotator cuff tear during compression is consistent with the “sagging peribursal fat” sign.

Comment Sonographic Findings This patient has a full-thickness tear of the supraspinatus tendon, which allows communication between the glenohumeral joint and the subacromial bursa. Sonography of the shoulder is an excellent and rapid method for evaluating abnormalities of the rotator cuff. The supraspinatus tendon appears hyperechoic and fibrillar, positioned directly on the humerus. A thin anechoic rim of cartilage covers the hyperechoic bone cortex. The deltoid muscle, which is hypoechoic, is just deep to the subcutaneous fat. Beneath it is a thin anechoic bursa with surrounding hyperechoic peribursal fat. Medially, the supraspinatus muscle is interposed between the trapezius muscle and the scapula. Nonvisualization of the tendon indicates a very large tear with a retracted tendon. Atrophy of the muscle may increase its overall echogenicity owing to increased fat content.

Figure S25-2  Longitudinal sonogram shows a hypoechoic region in the rotator cuff (yellow arrows) from the bursa to the articular surface of the humeral head. A cartilage interface sign is shown (red arrow). Compression caused the peribursal fat to sag into the rotator cuff defect.

Sonographic Signs Full-thickness rotator cuff tears appear as hypoechoic or anechoic defects in which fluid has replaced the torn tendon (Figure S25-1). This fluid accentuates the through-transmission of the ultrasound beam, accentuating the cartilage that appears hyperechoic. This creates the appearance of a double cortex, with two hyperechoic parallel lines, or the cartilage interface sign (Figure S25-2). Compression may produce herniation of the peribursal fat into the tendon gap, called the sagging peribursal fat sign.

Magnetic Resonance Imaging Magnetic resonance imaging is effective for evaluating the rotator cuff. Reportedly, it has 92% sensitivity and 93% specificity (Figure S25-3) for full-thickness tears and 64% sensitivity and 92% specificity for partial-thickness tears. Reference Nazarian LN, Jacobson JA, Benson CB, et al. Imaging algorithms for evaluating suspected rotator cuff disease: Society of Radiologists in Ultrasound consensus conference statement. Radiology. 2013;267:589–595.

Cross-Reference Musculoskeletal Imaging: The Requisites, 4th ed, 92–98.

Figure S25-1  Transverse sonogram shows absence of the rotator cuff tendon where the supraspinatus tendon should be (yellow arrows). A normal tendon would have a hyperechoic appearance similar to the infraspinatus tendon (asterisk). The biceps tendon (red arrow) is located at the rotator cuff interval.

52

CASE 26

Figure 26-1

HISTORY:  A 37-year-old woman with persistent low back pain for several years. 1.  What would be included in the differential diagnosis? (Choose all that apply.) A. Ankylosing spondylitis B. Infection C. Ulcerative colitis D. Reiter disease E. Psoriatic spondyloarthropathy 2. Why is the iliac side more severely affected than the sacral side? A. The cortex is thinner on the iliac side B. The iliac side of the joint has fibrocartilage C. The cortex is denser on the sacrum D. The iliac cartilage is thinner and has clefts

Figure 26-2

3. What disease is MOST LIKELY to develop ankylosis of the sacroiliac (SI) joints? A. Psoriasis B. Rheumatoid arthritis C. Pyrophosphate arthropathy D. Gout . How is a Ferguson view obtained? 4 A. Anteroposterior (AP) radiograph with cephalad angulation by 25 to 30 degrees B. AP radiograph with caudal angulation by 25 to 30 degrees C. Posteroanterior (PA) radiograph with cephalad angulation by 25 to 30 degrees D. PA radiograph with caudal angulation by 25 to 30 degrees

See the Supplemental Figures section for additional figures and legends for this case.

53

ANSWERS CASE 26 Sacroiliitis 1. B, D, and E. The differential diagnosis for asymmetric or unilateral sacroiliitis includes psoriatic spondyloarthropathy, Reiter disease, and infection. Because this is a woman, Reiter disease would be considered much less likely since 98% of those affected are male. Ankylosing spondylitis (which is much more common in men) and ulcerative colitis typically present as symmetric sacroiliitis. 2. D. The cartilage is thinner on the iliac side but also contains clefts that facilitate the formation of erosions. In addition, the sacral side is structurally more similar to a symphysis, and the iliac side is structurally more similar to a synovial joint. 3. A. Conditions that can fuse the SI joints include ankylosing spondylitis, psoriasis, Reiter syndrome, colitic spondylitis, and, sometimes, infection. 4. A. The Ferguson view is obtained by directing the x-ray beam in an anterior to posterior direction with the camera angled cephalad by 25 to 30 degrees.

Figure S26-2  Computed tomography shows the same findings but note that the erosions affect the iliac side more severely (arrows) than the sacral side. Compare these findings with the normalappearing sacroiliac joint on the left with smooth cortical margins present on both sides of the joint.

Comment Pathologic Considerations The radiograph and computed tomography (CT) show erosive changes in the right SI joint (Figures S26-1 and S26-2). The principal observation when evaluating sacroiliitis is the distribution of the disease. Typically, the synovial portion of the SI joint, found in the inferior one half to two thirds of the joint on frontal radiographs, is more severely involved than the ligamentous portion (Figure S26-3). The Ferguson view allows for optimal radiographic inspection of this area. Erosions involving the synovial articulation affect the iliac side more severely than the sacral articulation because of architectural differences between the two sides.

Symmetric Sacroiliitis In ankylosing spondylitis, the SI joint constitutes a classic site of initial involvement, and involvement is generally characterized as bilateral and symmetric in distribution. Sacroiliitis associated

with inflammatory bowel disease is usually bilateral and symmetric in distribution and cannot be differentiated from ankylosing spondylitis.

Asymmetric Sacroiliitis Psoriatic arthropathy and Reiter syndrome result in osseous erosions and bony sclerosis similar to those seen in ankylosing spondylitis, although ankylosis is less common.The distribution, however, may be bilateral and symmetric but is more often bilateral and asymmetric, or unilateral, which helps to differentiate it from ankylosing spondylitis.

Differential Diagnosis Osteoarthritis may involve one or both joints, and is manifested by joint space narrowing, osteophyte formation, and eburnation, but erosions are conspicuously absent. When only one SI joint appears abnormal, one must always consider the possibility of infection when the cortex is irregular, particularly if there is a history of intravenous drug abuse. In this situation, it is best to aspirate the joint, although magnetic resonance (MR) imaging and CT may be noninvasive alternatives. Both CT and MR imaging (Figures S26-4 and S26-5) allow for direct inspection of the joint surface and are better able to detect cartilaginous and osseous destruction than conventional radiography. References Amrami KK. Imaging of the seronegative spondyloarthropathies. Radiol Clin North Am. 2012;50:841–854. Egund N, Jurik AG.Anatomy and histology of the sacroiliac joints. Semin Musculoskelet Radiol. 2014;18:332–339.

Cross-Reference Musculoskeletal Imaging: The Requisites, 4th ed, 318–322.

Figure S26-1  Anteroposterior view of the pelvis shows erosions surrounded by reactive sclerosis in the right sacroiliac (SI) joint (arrows). This results in the appearance of an irregular cortical contour affecting both sides of the right SI joint.

54

CASE 27

Figure 27-1

HISTORY:  An 88-year-old man with acute right hip pain after a fall. 1. Based on the radiograph ONLY, what is included in the differential diagnosis? (Choose all that apply.) A. Chondrosarcoma B. Paget disease C. Lymphoma D. Osteoblastic metastases 2. What would a sudden rise in the serum alkaline phosphatase level mean? A. Active infection B. Sarcomatous degeneration C. Acute fracture D. Recurrence of lytic phase

Figure 27-2

3. What does a mosaic pattern refer to in patients with this condition? A. Loss of bony trabeculation B. Deposition of sclerosis within areas of fat C. Leading edge of osteolysis D.  Patternless arrangement of coarse and thickened trabeculae 4.  Does osteoporosis circumscripta involve the inner table, outer table, both, or neither? A. Inner table B. Outer table C. Both D. Neither

See the Supplemental Figures section for additional figures and legends for this case.

55

ANSWERS CASE 27 Paget Disease 1. B, C, and D. The main observation is a diffuse increase in the density of the right innominate bone associated with marked thickening of the cortex.The findings are characteristic of Paget disease, but osteoblastic metastasis can have a similar appearance. Primary bone lymphoma has a spectrum of radiographic presentations, from a mixed sclerotic appearance seen with the non-Hodgkin type to a sclerotic lesion seen more typically with the Hodgkin type. The computed tomographic image excludes metastasis and lymphoma from consideration, however. 2. B. When Paget disease is widely disseminated, the serum alkaline phosphatase level may be elevated. An acute elevation, however, heralds malignant transformation. 3. D. The definition of a mosaic pattern is a patternless arrangement of coarsened and enlarged osseous trabeculae. 4. B. Osteoporosis circumscripta generally involves the outer table.

Figure S27-2  Cortical thickening and coarse trabeculation (arrows) appear more conspicuous on computed tomography, which often is definitive.

Comment Different Phases Paget disease is characterized by destruction (lytic phase) of bone followed by attempts at repair (reparative phase). The cause of this disease is unknown. It is most common in middleaged people, affecting twice as many men as women. The lytic phase may predominate early in the disease process, but more frequently there is a combination of destruction and repair.The osteosclerotic phase is characterized by osteoblastic activity (Figures S27-1 and S27-2). In the quiescent phase, bone resorption and cellular activity are absent. Histologically, lesions are characterized by fibrosis and marked vascularity. The haversian canals are abnormally enlarged, resulting in poor distinction between cortical and medullary bone. This is not seen in diffuse osteoblastic metastasis (Figure S27-3).

Imaging Characteristics About 10% to 35% of patients present with monostotic disease. In the long bones, it begins at the ends of the bones and moves

toward the diaphysis. The leading edge represents the osteolytic phase and has been described as having a “blade of grass” appearance. A mixed and sclerotic phase follows, resulting in a pattern of disorganization with bone expansion, thickening of the cortex, and trabecular coarsening, referred to as a “cotton wool” appearance in the skull (Figure S27-4).The bones are soft and prone to fracture (Figure S27-5). In the spine, involvement may be monostotic or polyostotic. The thickened cortex produces a classic “picture frame” appearance. The lumbar spine and sacrum are the most common sites of involvement in the vertebral column. Bone scintigraphy typically shows markedly increased uptake (Figure S27-6) but may be normal in burnedout lesions or show peripheral uptake in the inactive phase of the disease. Reference Theodorou DJ, Theodorou SJ, Kakitsubata Y. Imaging of Paget disease of bone and its musculoskeletal complications: review. AJR Am J Roentgenol. 2011;196(6 suppl):S64–S75.

Cross-Reference Musculoskeletal Imaging: The Requisites, 4th ed, 397–403.

Figure S27-1  Anteroposterior pelvis shows diffusely sclerotic changes in the right innominate bone (arrows) with coarsening of the trabeculation and thickening of the cortex. The left innominate bone is normal and depicts the appropriate cortical thickness.

56

CASE 28

Figure 28-1

Figure 28-2

HISTORY:  A 35-year-old man with shoulder pain and a prior history of injury.

3.  What factor is associated with the highest risk of redislocation? A. Number of dislocations B. Mechanism of injury C. Associated fractures D. Age during initial dislocation

. What was the prior shoulder injury? 1 A. Greater tuberosity fracture B. Full-thickness rotator cuff tear C. Acromioclavicular (AC) joint separation D. Anterior shoulder dislocation 2. What is the MOST COMMON mechanism of injury responsible for an anterior shoulder (glenohumeral) dislocation? A. Direct impaction onto the posterior shoulder B. Hyperabduction with external rotation of an extended arm C. Posterior subluxation on a flexed arm D. Hyperabduction with internal rotation of an extended arm

4. All of the following are characteristic fractures associated with an anterior shoulder dislocation EXCEPT: A. Hill-Sachs lesion B. Bankart fracture C. Lesser tuberosity fracture D. Greater tuberosity fracture

See the Supplemental Figures section for additional figures and legends for this case.

57

ANSWERS CASE 28 Anterior Shoulder Dislocation 1. D. The key observation is a Hill-Sachs lesion depicted on the internally rotated view. The greater tuberosity footplate appears normal, so rotator cuff pathology or fracture is not demonstrated.The AC joint appears anatomically aligned. 2. B. The most common mechanism of injury is hyperabduction of an externally rotated and fully extended arm. Direct impaction to the posterior shoulder can also produce an anterior dislocation, but it is significantly less common. 3. D. The patient’s age at first dislocation is the most important risk factor associated with recurrence. 4. C. Lesser tuberosity fractures suggest a posterior glenohumeral joint dislocation. Hill-Sachs lesions, osseous Bankart lesions, and greater tuberosity fractures are important associated fractures in anterior shoulder dislocations.

Comment Mechanism of Injury The glenohumeral joint is the most commonly dislocated joint in the skeleton. Nearly 95% result in anterior displacement of the humeral head with respect to the glenoid fossa. An indirect force, such as a fall on an outstretched arm, is the most common mechanism of injury. The main factor that determines a patient’s risk for a redislocation is the age of the patient during the first dislocation, with a more than 90% recurrence rate if the first dislocation occurs before 20 years of age.

Types of Anterior Dislocations Four types of anterior dislocations have been described: subcoracoid, subclavicular, subacromial, and intrathoracic.

Imaging Features There are several important radiographic signs. An impaction fracture of the posterolateral aspect of the humeral head

Figure S28-2  Internally rotated view of the humerus shows a linear defect in the posterolateral aspect of the head (arrows) consistent with a large Hill-Sachs lesion.

(Hill-Sachs lesion) (Figures S28-1 to S28-4), fracture of the anteroinferior glenoid rim (osseous Bankart lesion) (Figures S28-5 and S28-6), and greater tuberosity fractures are important observations that may be detected on the initial radiographs. Computed tomography may be useful in confirming fractures and quantifying the size, location, and degree of displacement, particularly with glenoid rim fractures and Hill-Sachs lesions. The extent of glenoid bone loss as well as more medially located Hill-Sachs lesions are both associated with increased engagement on physical examination. Soft tissue injuries to the labrum, anterior capsule, and subscapularis tendon are deferred to magnetic resonance imaging. Remember, the physiologic trough can mimic a Hill-Sachs lesion on cross-sectional imaging; however, the physiologic trough does not extend to the superior-most centimeter of the humeral head, that is, the initial three or four axial images of the humerus. Reference Gyftopoulos S, Albert M, Recht MP. Osseous injuries associated with anterior shoulder instability: what the radiologist should know. AJR Am J Roentgenol. 2014;202:W541–W550.

Cross-Reference Musculoskeletal Imaging: The Requisites, 4th ed, 77–85, 98–105.

Figure S28-1  Anteroposterior shoulder radiograph shows no specific abnormality. The greater tuberosity footplate is normal (arrow).

58

CASE 29

Figure 29-1

Figure 29-2

HISTORY:  A 44-year-old patient with a chronic disease.

3.  What causes increased bone resorption in patients with hyperparathyroidism? A. Increased osteoblastic activity B. Increased osteoclastic activity C. Increased fibroblastic activity D. Increased osteocytic activity

1.  What should be included in the differential diagnosis? (Choose all that apply.) A. Sacroiliitis B. Hyperparathyroidism C. Renal osteodystrophy D. Secondary tumoral calcinosis E. Osteomalacia 2. In patients with this condition, what does osteitis fibrosa cystica refer to? A. An osteoclastoma filled with parathyroid cells B. A bone cyst secondary to aluminum toxicity C. A bone cyst filled with amyloid D. A necrotic brown tumor undergoing liquefaction

4. What is the most common cause of primary hyperparathyroidism? A. Parathyroid adenoma B. Parathyroid hyperplasia C. Parathyroid carcinoma D. Multiple endocrine adenomatosis type 2

See the Supplemental Figures section for additional figures and legends for this case.

59

ANSWERS CASE 29 Hyperparathyroidism 1. B, C, and D. Subchondral bone resorption caused the widening of the sacroiliac joints and lysis of the distal clavicles. The end plates of the vertebral bodies were slightly dense, consistent with renal osteodystrophy. Metastatic calcifications are present in the left shoulder from chronic renal failure and although other conditions can present like this, the observed imaging findings restrict the differential diagnosis. Osteomalacia was not a consideration because the patient was not osteopenic. 2. D. Osteitis fibrosa cystica is a brown tumor (osteoclastoma) that has become cystic because of necrosis and liquefaction. 3. B. Increased osteoclastic activity is the main cellular component of increased bone resorption in patients with hyperparathyroidism. 4. A. Solitary (80%) or multiple (7%) adenomas are the most common cause of primary hyperparathyroidism, followed by hyperplasia (10% to 15%), carcinoma (2% to 4%), nonparathyroid tumors, and multiple endocrine neoplasia (MEN) syndromes.

Comment Clinical Information and Types Hyperparathyroidism refers to a group of disorders characterized by the presence of increased circulating parathyroid hormone (PTH). The presence of excess PTH results in increased osteoclastic resorption of the bone. Osteocytic osteolysis is responsible for the initial calcium release from the bone. In primary hyperparathyroidism, the parathyroid gland overproduces PTH. Patients present with weakness, lethargy, bone pain, polydypsia, and polyuria. Other associated abnormalities include nephrolithiasis, gastrointestinal ulcers, and pancreatitis. Common causes of the primary form of the disease include

Figure S29-2  Close-up of a chest radiograph shows widening of the acromioclavicular joint space caused by distal clavicular subchondral bone resorption (arrow) and prominent calcific deposits around the left shoulder. The open arrow indicates that it was a standing chest radiograph.

an adenoma, hyperplasia of the gland, parathyroid carcinoma, tumors that secrete PTH-like substances, and type 2 MEN (Sipple syndrome). Secondary hyperparathyroidism occurs in response to chronic hypocalcemia, usually from renal glomerular disease and, sometimes, malabsorption. Tertiary hyperparathyroidism refers to an autonomous gland that has escaped the regulatory effects of serum calcium after a prolonged period of stimulation, typically occurring in patients on chronic hemodialysis.

Imaging Characteristics Bone resorption is the radiologic hallmark of hyperparathyroidism.This patient shows typical changes related to subchondral bone resorption (Figures S29-1 to S29-3). The most distinctive type of bone resorption, however, is subperiosteal, particularly in the phalanges of the hand (Figure S29-4). Other types of resorption include cortical, trabecular, and subligamentous. Osteosclerosis is seen most frequently in secondary hyperparathyroidism, and the rugger jersey spine is characteristic (Figure S29-5). This patient also had an expansile lesion in the right 10th rib consistent with a brown tumor. Brown tumors can weaken the bone, increasing the risk for pathologic fractures (Figure S29-6). Metastatic calcifications are common in the arterial walls and periarticular soft tissues. About 15% of patients also show evidence of chondrocalcinosis. Reference Boswell SB, Patel DB, White EA, et al. Musculoskeletal manifestations of endocrine disorders. Clin Imaging. 2014;38:384–396.

Cross-Reference Musculoskeletal Imaging: The Requisites, 4th ed, 374–376, 381–385. Figure S29-1  Anteroposterior radiograph of the lumbar spine shows classic findings of bone resorption of the sacroiliac joints, r­ esulting in a widened appearance (arrows). Note the expansile lesion in the right tenth rib consistent with a brown tumor (asterisk).

60

CASE 30

Figure 30-1

Figure 30-2

HISTORY:  A 28-year-old woman with chronic arthritis.

3. What is the SINGLE most likely cause of the lesion? A. Corticosteroid use B. Vasculitis C. Gaucher disease D. Trauma

1.  What histopathologic factors contribute to the development of this disorder? (Choose all that apply.) A. Thrombosis or embolism B. Decreased bone marrow pressure C. Disruption of the vascular supply D.  Weakened bone marrow from exposure to cytotoxic factors 2. Which of the following constitutes the BEST diagnosis? A. Insufficiency fracture B. Impaction fracture C. Osteochondral defect D. Avascular necrosis

4. On T2-weighted (T2W) magnetic resonance imaging (MRI), what would a thin region of high signal intensity beneath the cortex represent? A. Subchondral collapse B. Subchondral edema C. Granulation tissue D. Reparative bone

See the Supplemental Figures section for additional figures and legends for this case.

61

ANSWERS CASE 30 Avascular Necrosis 1. A, C, and D. Important etiologic considerations for avascular necrosis include thrombosis and embolism, disruption of a primary vascular supply, and weakening of the bone marrow from exposure to cytotoxic factors. 2. D. The radiographs show a region of sclerosis in the anterosuperior aspect of the femoral head and a focus of articular collapse where there was subchondral lucency typical of avascular necrosis. An insufficiency usually does not involve this large an area of the articular surface, and an impaction injury usually results in a fracture of the head, not collapse. Osteochondral defects do not usually occur in this joint. 3. A. This patient was on intermittent corticosteroids to help control her arthritis when it became active. 4. A. An arcuate area of subchondral high signal intensity represents a fracture cleft filled with fluid, which appears bright on T2W images.

Comment Etiology Avascular necrosis indicates bone death, most commonly due to ischemia. Etiologic factors that contribute to its development in the humeral head include trauma, hematologic conditions (systemic lupus erythematosus, Gaucher disease), Cushing syndrome or exogenous corticosteroid administration, alcoholism, pancreatitis, pregnancy, and caisson disease.

Radiographic Features The Ficat classification is most widely used for radiographic assessment. Stage 0 shows a normal radiograph. Stage 1 may show mild osteopenia. Stage 2 shows mixed osteopenia and sclerosis without a subchondral lucency, whereas stage 3 shows

Figure S30-2  Frog-leg lateral radiograph shows the residual subchondral lucency in the anterior margin of the collapsed articular surface (arrow).

a crescent sign or subchondral lucency and eventual collapse (Figures S30-1 and S30-2). Stage 4 is end stage with secondary arthritic changes.

Magnetic Resonance Imaging Features MRI is efficacious in assessing patients with joint pain and with risk factors for developing avascular necrosis. In the acute phase of the disease, bone marrow edema involving the femoral head may be the only notable finding. In the absence of any risk factors, osteonecrosis is only one of a number of other potential diagnoses. As the ischemic process worsens, it becomes more demarcated as an irregular rim of low signal intensity on T1W images (Figure S30-3).  A “double line” sign is depicted as paired rims of high and low signal intensities on T2W images, identifying the interface between viable and dying bone marrow. The high signal intensity rim is indicative of granulation tissue, and the adjacent low signal intensity rim reflects cellular debris, fibrous tissue, and reactive trabecular bone. When a subchondral fracture occurs, the cleft fills in with fluid and is depicted as a thin region of high signal intensity beneath the low signal intensity cortex on T2W images (Figure S30-4). References Lee JA, Farooki S, Ashman CJ, Yu JS. MR patterns of involvement of humeral head osteonecrosis. J Comput Assist Tomogr. 2002;26:839– 842. Malizos KN, Karantanas AH, Varitimidis SE, Dailiana ZH, Bargiotas K, Maris T. Osteonecrosis of the femoral head: etiology, imaging, and treatment. Eur J Radiol. 2007;63:16–28.

Cross-Reference Musculoskeletal Imaging: The Requisites, 4th ed, 346–351.

Figure S30-1  Frontal radiograph shows a well-demarcated focus of sclerosis (yellow arrows) in the femoral head and an area of collapse in the superior articular surface (red arrow).

62

CASE 31

Figure 31-1

Figure 31-2

HISTORY:  A 27-year-old fell on his hand.

3. What structure stabilizes the medial triangular fragment that does not displace? A. Transverse ligament B. Dorsal oblique ligament C. Volar oblique ligament D. Ulnar collateral ligament

1. What is your diagnosis? A. Rolando fracture B. Bennett fracture C. Epibasal fracture D. Salter-Harris type 3 fracture 2. When lateral subluxation of the metacarpal shaft fragment occurs, what is causing it? A. Adductor pollicis longus muscle B. Adductor pollicis brevis muscle C. Abductor pollicis longus muscle D. Abductor pollicis brevis muscle

. What is the preferred treatment for this injury? 4 A.  Closed reduction and fixation to the trapezium with K-wires B. Longitudinal traction and cast immobilization C. Open reduction with screws across the fracture D. Thumb-spica cast fixation for 6 weeks

See the Supplemental Figures section for additional figures and legends for this case.

63

ANSWERS CASE 31 Bennett Fracture 1. B. The radiographs show a typical Bennett fracture, an intraarticular fracture of the base of the first metacarpal. A Rolando fracture is a three-part fracture in this area of the bone. 2. C. The action of the abductor pollicis longus muscle ­distracts the fracture while the flexor pollicis brevis muscle, by its more distal attachment, augments the displacement. The adductor pollicis longus muscle flexes the metacarpal. 3. C. The volar (medial) fragment is anchored by the strong volar oblique ligament, holding in place its articulation with the trapezium. 4. A. The preferred treatment is closed reduction and fixation with percutaneous Kirschner wires. The wires do not need to cross the fracture or directly fix the metacarpal to the smaller fragment. Fixation of the base of the metacarpal to the trapezium is sufficient. If this cannot be accomplished, open reduction and fixation with wires is recommended.

Comment Basilar Fracture Types The first metacarpal bone is the second most frequently fractured metacarpal bone after the fifth metacarpal. Nearly 80% of fractures of this bone involve the base. Basilar fractures can be divided into four types: epibasal, Bennett, Rolando, and comminuted. Epibasal fractures are extraarticular fractures through the first metacarpal base and can be transversely or obliquely oriented. Comminuted fractures result in numerous bone fragments and frequently have more than one intraarticular fracture extension.A Bennett fracture is defined as an intraarticular twopart fracture of the base of the first metacarpal (Figures S31-1 and S31-2). It is the most common thumb fracture, accounting for about one third of all fractures of the first metacarpal bone. A Rolando fracture is similar, but it produces a three-part

Figure S31-2  Lateral view of the hand affords a frontal view of the thumb metacarpal, and the fracture (arrow) is more difficult to visualize. This is why dedicated thumb radiographs are preferred for this injury.

fracture (Figure S31-3). A comminuted fracture has more than three osseous fragments.

Key Anatomic Features When evaluating radiographs of the first metacarpal, recall the anatomy of the first carpometacarpal joint. The deep volar oblique ligament, which is intracapsular in location, is the most important stabilizer of this joint. It inserts on the ulnar articular margin of the medial volar beak of the first metacarpal base. When a Bennett or Rolando fracture occurs, the medial volar beak (Figures S31-4 and S31-5) remains attached to this ligament while the rest of the metacarpal base becomes displaced radially due to the pull of the abductor pollicis longus muscle. That is what makes this fracture potentially unstable. References Liverneaux PA, Ichihara S, Hendriks S, et al. Fractures and dislocation of the base of the thumb metacarpal. J Hand Surg Eur Vol. 2015;40:42–50. Peterson JJ, Bancroft LW. Injuries of the fingers and thumb in the athlete. Clin Sports Med. 2006;25:527–542.

Cross-Reference Musculoskeletal Imaging: The Requisites, 4th ed, 159–160.

Figure S31-1  Frontal radiograph of the right hand shows an intraarticular fracture (arrow) at the base of the first metacarpal bone.

64

CASE 32

Figure 32-1

Figure 32-2

HISTORY:  A 23-year-old woman has chronic swelling in the distal calf.

3. What percentage of patients with fibrous dysplasia have the monostotic form? A. 10% B. 40% C. 70% D. 95%

1. What is your diagnosis? A. Lymphoma B. Gaucher disease C. Fibrous dysplasia D. Diaphyseal dysplasia . In patients with this condition, what is cherubism? 2 A. Association with soft tissue myxomas B. Involvement of the tibia with osteoblastic activity C. Precocious puberty and café au lait spots D. Multilocular cysts in the mandible

4.  How often does fibrous dysplasia undergo malignant transformation? A. Never B. Rarely C. In 10% of cases D. In 25% of cases

See the Supplemental Figures section for additional figures and legends for this case.

65

ANSWERS CASE 32 Fibrous Dysplasia 1. C. The expansile changes in the bone, the cyst like changes in the diaphysis with sclerotic border, and the ground-glass appearance with irregular calcifications in both the tibia and fibula make this an “Aunt Minnie” diagnosis. Lymphoma may show some of these features, but when diffuse it also shows some aggressive features. Look for the Erlenmeyer deformity in Gaucher disease and “hair-on-end” periostitis from extraosseous spread if the marrow changes are severe. In diaphyseal dysplasia, the marrow space would be constricted by the thickened cortex. 2. D. Cherubism is seen in familial fibrous dysplasia with involvement of the mandible. 3. C. The monostotic form is present in 70% of cases. 4. B. Malignant transformation of fibrous dysplasia is extremely rare but can occur. Osteosarcoma and spindle cell sarcoma have both been described. External radiation may be a contributory factor.

Comment Pathology Fibrous dysplasia is a hamartomatous fibro-osseous metaplastic disorder caused by a defect in the germ plasm involving the proliferation and maturation of fibroblasts. The marrow becomes replaced by fibrous tissue. The variable amount of osteoid accounts for a wide range of radiographic densities in the bone.

Figure S32-1  The distal tibia and fibula show variable bony expansion, bowing deformities, and areas of endosteal scalloping (arrows). Note the uniform ground-glass opacities (asterisk) within the medullary cavity.

Clinical Features Clinically, patients are young and generally present in the first or second decade of life with pain or pathologic fractures. Patients with polyostotic disease frequently have café au lait spots (30% to 50%) and may present with endocrine dysfunction, particularly precocious puberty (referred to as McCune-Albright syndrome). About 90% of patients who have polyostotic disease have involvement in only one limb or one side of the body. The monostotic form, seen in 70% of patients, has an affinity for the long bones and has a wide spectrum of radiographic appearances, ranging from relatively lucent to having scattered, heterogeneous areas of sclerosis.

Imaging Findings The characteristic appearance is uniform ground-glass density seen centrally within the medullary cavity (Figure S32-1). The endosteal surface may be scalloped or mixed with sclerosis (Figure S32-2). Expansion of the bone is common, as are bowing deformities. Lesions that have been present for a long period of time may demonstrate spotty calcification or a well-defined sclerotic rim (Figure S32-3), or a mixed appearance with focal lytic areas surrounded by patchy areas of sclerosis with deformation of the bone morphology (Figure S32-4). Pathologic fractures, due to weakening of the bone, can elicit a periosteal reaction, but otherwise periostitis is not a typical feature of this condition. On magnetic resonance imaging, lesions are isointense with areas of hypointensity on T1-weighted images (Figure S32-5) and heterogeneously hyperintense on T2-weighted images (Figure S32-6), and may show a patchy enhancement pattern.

66

Figure S32-2  The demarcation between normal bone and abnormal bone in the tibia (arrow) is more apparent in the lateral view.

References DiCaprio MR, Enneking WF. Fibrous dysplasia. Pathophysiology, evaluation, and treatment. J Bone Joint Surg Am. 2005;87:1848–1864. Shak ZK, Peh WC, Koh WL, Shek TW. Magnetic resonance imaging appearances of fibrous dysplasia. Br J Radiol. 2005;78:1104–1115.

Cross-Reference Musculoskeletal Imaging: The Requisites, 4th ed, 460–465.

CASE 33

Figure 33-1

HISTORY:  A 65-year-old patient fell and presents with hip pain. . What is the BEST diagnosis? 1 A. Subcapital femur fracture B. Femoroacetabular impingement C. Stress fracture D. Avascular necrosis 2. Using the Garden classification, how would this injury be classified? A. Garden I B. Garden II C. Garden III D. Garden IV

3. All of these vessels supply the femoral head EXCEPT: A. Retinacular arteries B. Medial femoral circumflex artery C. Superficial femoral artery D. Artery of the ligamentum teres 4. What is the sensitivity of magnetic resonance (MR) imaging for detecting femoral neck fractures? A. Less than 10% B. 33% C. 66% D. Greater than 99%

See the Supplemental Figures section for additional figures and legends for this case.

67

ANSWERS CASE 33 Subcapital Femur Fracture 1. A. The best diagnosis is a valgus-impacted subcapital fracture of the femoral neck. 2. A. This would be considered a Garden I fracture. 3. C. The femoral artery bifurcates into the deep femoral artery and the (superficial) femoral artery. The branches of the deep femoral artery, also known as the deep artery of the thigh, supply the femoral head, including the medial and lateral circumflex arteries which give rise to the retinacular arteries.The obturator artery is the main supply of the artery of the ligamentum teres.

the medial circumflex artery (medial epiphyseal artery) and lateral circumflex artery (lateral epiphyseal artery) converge to form a vascular ring that supplies numerous ascending cervical arterial vessels.The artery of the ligamentum teres also contributes blood supply to the femoral head epiphyseal region. The leading factors contributing to the development of avascular necrosis of the femoral head are disruption of the epiphyseal arteries or the vessels that supply these terminal vessels, and an unstable reduction of a fracture. Angulated and displaced neck fractures tether these end arteries, disrupting the blood supply (Figure S33-2).

Garden Classification

4. D. For all practical purposes, MR imaging is 100% sensitive for detecting femoral neck fractures in patients who present with nondisplaced fractures that are radiographically occult.

The most common classification for intracapsular hip fractures is the Garden classification (four stages): I, incomplete or valgus-impacted fracture (Figure S33-1); II, complete fracture without displacement (Figure S33-3); III, complete fracture with partial displacement and varus angulation (Figure S33-4); and IV, complete fracture with complete displacement (Figure S33-5).

Comment

Reference

Pathologic Considerations

Yu JS. Hip and femur trauma: imaging of trauma to the extremities. Semin Musculoskelet Radiol. 2000;4:205–220.

Subcapital fractures occur just distal to the articular margin of the femoral head. Fractures that occur through the neck are referred to as transcervical fractures, and those that occur at the junction of the neck and the shaft are referred to as basicervical. Two important potential complications of subcapital fractures include nonunion and avascular necrosis of the femoral head. The more proximal the fracture line, the greater the incidence of both complications.

Vascular Supply The femoral head is supplied by three terminal arterial sources. The main arterial supply of the adult femoral head originates from the medial and lateral femoral circumflex arteries, which may arise from either the femoral artery or the deep femoral artery. At the base of the femoral neck, terminal branches of

Figure S33-1  Frontal radiograph shows a fracture line (arrows) that extends from below the head of the femur laterally to the medial cortex. Note that there is mild valgus angulation from lateral impaction.

68

Cross-Reference Musculoskeletal Imaging: The Requisites, 4th ed, 168-171.

CASE 34

Figure 34-1

Figure 34-2

HISTORY:  A 45-year-old woman presents with neck pain.

3. What condition is associated with the “picture frame” sign? A. Gastric carcinoma B. Lymphoma C. Paget disease D. Hemangioma

1.  What should be included in the differential diagnosis? (Choose all that apply.) A. Plasmacytoma B. Lymphoma C. Hemangioma D. Metastases E. Paget disease 2. Which ONE of the following neoplasms is LEAST LIKELY to produce blastic metastases? A. Chordoma B. Prostate C. Breast D. Carcinoid

4. What condition is associated with the “sandwich vertebra” sign? A. POEMS syndrome B. Paget disease C. Sickle cell disease D. Osteopetrosis tarda

See the Supplemental Figures section for additional figures and legends for this case.

69

ANSWERS CASE 34 Ivory Vertebra 1. B, C, D, and E. The differential diagnosis for an ivory vertebra includes blastic metastases, lymphoma, hemangioma, chronic infection, and Paget disease. 2. A. A chordoma is largely lytic in appearance, although it may have peripheral calcifications and produce secondary bone sclerosis in the tumor periphery. Prostate metastasis is purely osteoblastic; breast and carcinoid are frequently osteoblastic. 3. C. Paget disease produces the “picture frame” sign characterized by coarse trabeculation, thickened cortices, and vertebral body enlargement. 4. D. Osteopetrosis tarda is associated with this sign, referring to increased density occurring at the end plates. Sickle cell disease produces the “bone-in-bone” sign.

Comment Diagnostic Considerations Metastasis is the most common tumor of the spine. Metastasis to the axial skeleton occurs in nearly one half of patients who have been discovered to have osseous metastasis on bone scintigraphy, most frequently involving the thoracic and lumbar vertebral bodies. When a vertebral body is heterogeneously or homogeneously sclerotic, it is referred to as an ivory vertebra (Figures S34-1 and S34-2). In adults, the differential diagnosis includes osteoblastic metastases such as prostate, breast, and carcinoid; lymphoma; Paget disease; and occasionally, infection and hemangioma. A characteristic finding in metastasis is involvement of the pedicles, which can result in a “dense pedicle” sign (Figure S34-3). In children, lymphoma, osteosarcoma, and osteoblastoma, as well as metastases from neuroblastoma and medulloblastoma, should be considered.

Figure S34-2  Sagittal T2-weighted magnetic resonance image shows diffuse low signal intensity changes within the marrow of the C7 vertebral body (arrow). There is no pathologic fracture.

Differential Diagnosis When the process is more diffuse, affecting numerous vertebral bodies, other conditions such as mastocytosis, tuberous sclerosis, myelofibrosis, renal osteodystrophy, fluorosis, and osteopetrosis should be considered. Not all conditions that appear to mimic an ivory vertebra are caused by osteoblastic metastasis, however. Paget disease often is mistaken for an ivory vertebra, but careful observation for enlargement of the vertebral body as well as coarsening of the trabeculation often reveals the diagnosis (Figure S34-4).The same is true of a vertebral body hemangioma. Reference Graham TS.The ivory vertebra sign. Radiology. 2005;235:614–615.

Cross-Reference Musculoskeletal Imaging: The Requisites, 4th ed, 497.

Figure S34-1  Lateral cervical spine radiograph shows a very dense C7 vertebral body (arrow) and degenerative disk disease at the C6-C7 level. This patient had breast cancer.

70

CASE 35

Figure 35-1

Figure 35-2

HISTORY:  A 32-year-old slipped on the ice and fell on the upper extremity.

3.  Using the Frykman classification, what type would this patient have? A. Type 2 B. Type 4 C. Type 6 D. Type 8

1.  What would be included in the differential diagnosis? (Choose all that apply.) A. Colles fracture B. Chauffeur fracture C. Smith fracture D. Reverse Colles fracture E. Volar Barton fracture 2. After reducing a distal radius fracture, all of the following should be observed EXCEPT: A. Less than 10 degrees of palmar tilt B. Radial-sided inclination C. Maintenance of radial length D. Congruity of the articular surface

4.  Low-energy trauma in elderly patients most commonly results in what type of fracture? A. Dorsally impacted comminuted radius fracture B. Volarly impacted comminuted radius fracture C. Dorsally impacted comminuted ulnar fracture D. Volarly impacted comminuted ulnar fracture

See the Supplemental Figures section for additional figures and legends for this case.

71

ANSWERS CASE 35 Distal Radius (Smith) Fracture 1. C, D, and E. This patient has a type 3 Smith fracture with an intraarticular fracture of the distal radius with volar displacement.This is the same as a volar Barton fracture or a reverse Colles fracture. 2. B. All of the choices are goals except radial-sided inclination. The goal is to obtain ulnar-sided inclination. 3. B. In the Frykman classification, a type 4 is a radius fracture that extends to the radiocarpal joint and is associated with an ulnar styloid fracture. 4. A. The most common low-energy wrist fracture in elderly patients is a Colles fracture, which is characterized by dorsal impaction and is often comminuted.

Comment Clinical Features Distal radius fractures are extremely common injuries.They are 10 times more likely to occur than carpal fractures and may have a wide spectrum of appearances. A Smith fracture is similar to a Colles fracture except for volar displacement and/or angulation (Figures S35-1 and S35-2); hence these fractures are frequently referred to as reverse Colles fractures. The mechanism is likely related to a fall on the dorsum of the hand. The fracture can be intraarticular or extraarticular. The lateral view is the key to diagnosis, showing that the distal radial fragment displaces anteriorly with palmar angulation of the articular surface. The Thomas classification describes three types: type 1 is a transverse extraarticular fracture, type 2 involves an intraarticular fracture that crosses into the dorsal surface, and type 3 is a volar intraarticular fracture of the radiocarpal joint. These fractures are frequently associated with ulnar styloid fractures and may also be associated with injury to the extensor tendons.

Figure S35-2  Lateral radiograph shows soft tissue swelling and displaced pronator quadratus fat stripe (asterisk). The fracture produces anterior displacement of the volar lip of the distal radius typical of a Smith fracture or a volar Barton fracture (arrow).

Fracture Types A Colles fracture is a fracture of the distal radius characterized by dorsal displacement and/or angulation (Figure S35-3). A Barton fracture is a fracture of the distal radius associated with dislocation of the radiocarpal joint, and can involve either the volar or dorsal cortex. Note that the volar type of a Barton fracture is the same as a type 3 Smith fracture. The Frykman classification is still widely used for describing fractures of the distal radius and ulnar styloid. In this classification, the odd numbers refer to the fracture of the radius: type 1 is an extraarticular fracture, type 3 is a fracture that extends to the radiocarpal joint, type 5 is a fracture extending to the sigmoid notch, and the type 7 fracture extends to both the radiocarpal and distal radioulnar joints. If the radius fracture is associated with an ulnar styloid fracture, the fracture becomes the even-numbered counterpart to the radius fracture (Figure S35-4). Reference Porrino JA Jr, Maloney E, Scherer K, et al. Fracture of the distal radius: epidemiology and premanagement radiographic characterization. AJR Am J Roentgenol. 2014;203:551–559.

Cross-Reference Musculoskeletal Imaging: The Requisites, 4th ed, 138–140.

Figure S35-1  Posteroanterior view of the right wrist shows an intraarticular fracture of the distal radius extending to the scaphoid facet (yellow arrow). The length of the bone is diminished, giving rise to an ulnar-plus configuration. There is also an ulnar styloid fracture (red arrow).

72

CASE 36

Figure 36-1

HISTORY:  A 22-year-old skateboarder with shoulder injury. 1.  What should be included in the differential diagnosis? (Choose all that apply.) A. Type 3 acromioclavicular (AC) joint separation B. Type 4 AC joint separation C. Type 5 AC joint separation D. Type 6 AC joint separation . What is a “floating” clavicle? 2 A. Clavicle fracture with concomitant AC joint separation B. Any complete AC joint dislocation C. Simultaneous dislocation of the AC and sternoclavicular joints D. AC separation with a subclavicular hematoma

Figure 36-2

. What is the purpose of weight-bearing views? 3 A. To discriminate between type 1 and type 2 injuries B. To discriminate between type 2 and type 3 injuries C. To discriminate between type 3 and type 4 injuries D. To discriminate between type 5 and type 6 injuries 4. In children and adolescents, what is a potential associated injury? A. Avulsion of the lesser tuberosity B. Avulsion of the greater tuberosity C. Avulsion of the acromion process D. Avulsion of the coracoid process

See the Supplemental Figures section for additional figures and legends for this case.

73

ANSWERS CASE 36 Acromioclavicular Joint Separation 1. A and B. In type 3 injuries, the AC and coracoclavicular ligaments are completely disrupted and there is clavicular migration exceeding 5 mm or 50% of the bone width. In type 4 injuries, the AC and coracoclavicular ligaments are completely disrupted and the clavicle dislocates posteriorly. Depending on the projection, it may appear superiorly displaced as well. In type 5 injuries, the distal clavicle is markedly displaced superiorly. 2. C. A “floating” clavicle is caused by the simultaneous dislocation of the AC and sternoclavicular joints. 3. B. Weighted views of the shoulder are helpful in discriminating between type 2 and type 3 AC injuries. 4. D.In young people, the coracoclavicular ligaments may remain intact during injury to the shoulder and instead may avulse the coracoid process at its base. This injury typically occurs in patients younger than 25 years because fusion of the coracoid ossification center can occur as late as 21 to 25 years of age.

Comment Tossy Classification and Imaging Findings The majority of AC joint separations or dislocations occur from direct impaction to the point of the shoulder. They constitute about 9% of shoulder girdle injuries.The most widely used classification is the Tossy classification, which identifies three different types.The radiographs appear normal in a type 1 separation. Clinically, soft tissue swelling about the joint and an effusion may be evident and indicate a mild strain of the AC ligament. In a type 2 injury, there is disruption of the AC ligament and possible strain of the coracoclavicular ligaments, such that the clavicle migrates superiorly less than 5 mm or 50% of the width of the clavicle on weight-bearing views. In type 3 injuries, the AC and coracoclavicular ligaments are completely disrupted and there is clavicular migration exceeding 5 mm or 50% of the bone width, as seen in this patient (Figures S36-1 and S36-2). On magnetic resonance imaging (MRI), the coronal plane is useful in depicting a strain or disruption of the superior AC ligament (Figures S36-3 and S36-4), and the sagittal plane is optimal for showing disruption of the coracoclavicular ligaments (Figure S36-5). Interstitial edema in the surrounding connective tissues and muscles corresponds to a more severe

Figure S36-1  Anteroposterior view of the shoulder shows a widened acromioclavicular joint and superior migration of the distal clavicle with respect to the acromion process (arrow).

74

Figure S36-2  Non–weight-bearing view of the clavicle confirms that the clavicle is displaced superiorly by 100% of the width of the shaft, exposing the entire articular surface (arrow).

injury mechanism. Weight-bearing radiographs are useful in discriminating between type 2 and type 3 injuries.

Rockwood Revision Rockwood described three additional injury patterns. In type 4 injuries, the clavicle is displaced posteriorly into or through the trapezius muscle (Figure S36-6). In type 5 injuries, the clavicle migrates superiorly more than in a type 3 separation. In type 6 injuries, the clavicle dislocates inferiorly below the coracoid or acromion processes.

Complications Acutely, postoperative infection is the most important concern, but loss of surgical reduction is the most common immediate complication. The two most important chronic complications of this injury are secondary osteoarthritis and distal clavicular osteolysis. Reference Melenevsky Y,Yablon CM, Ramappa A, et al. Clavicle and acromioclavicular joint injuries: a review of imaging, treatment, and complications. Skeletal Radiol. 2011;40:831–842.

Cross-Reference Musculoskeletal Imaging: The Requisites, 4th ed, 76–77.

CASE 37

Figure 37-1

Figure 37-2

HISTORY:  A 16-year-old boy presents with knee pain.

3. What is the MOST reliable finding on MR arthrography that indicates an unstable fragment? A. Imbibition of contrast around entire fragment B. Enhancement of the bone marrow adjacent to fragment C. A fissure in the articular cartilage D. Contrast entering a cleft in the bone

1. What entities can be included in the differential diagnosis? (Choose all that apply.) A. Impaction fracture B. Insufficiency fracture C. Traumatic osteochondral fracture D. Osteochondritis dissecans 2. Regarding the idiopathic type, how many patients have bilateral lesions? A. Less than 1% B. 10% C. 25% D. Greater than 50%

4. How would you grade this lesion? A. Grade 2 B. Grade 3 C. Grade 4 D. Grade 5

See Supplemental Figures section for additional figures and legends for this case.

75

ANSWERS CASE 37 Osteochondritis Dissecans 1. C and D. Osteochondritis dissecans is caused by idiopathic devascularization of the non–weight-bearing portion of the femoral condyle; approximately 80% of lesions involve the medial femoral condyle. A rotational or shearing injury may occasionally produce a subchondral fatigue fracture, which can then become fragmented and mimic osteochondritis dissecans and can affect either of the femoral condyles. Impaction fractures usually occur in the articular surface of the lateral femoral condyle because the pivot-shift mechanism is responsible for the majority of these injuries. Insufficiency fracture occurs in older people. 2. C. Approximately 25% of patients with the idiopathic type of osteochondritis dissecans have bilateral lesions. 3. A. Fluid imbibition between the osteochondral fragment and the rest of the condyle is the most reliable indicator of instability. 4. A. Grade 2 is characterized by subchondral fracture lines with intact overlying articular cartilage, although it may also be associated with cartilage fissuring.

Comment

Figure S37-1  T1-weighted magnetic resonance image in a patient with an extended osteochondritis dissecans (asterisk). The lesion involves the nonarticulating surface of the medial femoral condyle as well as a small portion of its weight-bearing surface (arrow).

Clinical Considerations Osteochondritis dissecans refers to a condition that is characterized by the development of a devascularized fragment of bone, usually in the non–weight-bearing surface of the medial femoral condyle.The etiology is unknown but is speculated to be the result of repeated minor trauma during an age when the epiphysis reaches skeletal maturation. Osteochondritis dissecans affects males two to three times more frequently than females, and most patients present in adolescence (mean age of 15 years).

Imaging Features and Staging Radiographically, the lesion has a well-demarcated rim of sclerosis that surrounds the abnormal marrow and extends to the articular cortex. The size of the osteochondral lesion and the thickness of the surrounding sclerotic rim are helpful when staging a lesion: grade 1, a subchondral bone lesion (Figure S37-3) measuring 1 to 3 cm in diameter with intact overlying articular cartilage; grade 2, subchondral fracture lines with intact overlying articular cartilage (Figures S37-1 and S37-2), although fissures may be present as well; grade 3, partial detachment of the osteochondral defect with a large cleft extending into the subchondral bone (Figure S37-4), with the lesion easily displaced on arthroscopy; grade 4, lesions that are in situ loose fragments in which the overlying articular cartilage is completely disrupted (Figure S37-5); and grade 5, a loose body and an empty crater (Figure S37-6).

MR Imaging Findings MR arthrography is the most accurate technique for establishing the stability of a lesion. Conventional T2-weighted images may depict high signal intensity surrounding the osteochondral fragment, but it may be difficult to differentiate granulation tissue from free fluid. Look for cartilage clefts at the edges of the lesion. Focal cystic lesions deep to the lesion are associated with instability of the fragment, as is fluid imbibition between the fragment and the rest of the condyle.

76

Figure S37-2  Sagittal T2-weighted image shows the demarcation of the lesion surrounded by bone marrow edema (arrows). There was no cartilage defect, making this a grade 2 lesion.

Displacement of the fragment is a reliable radiographic finding of instability. Reference Grimm NL, Weiss JM, Kessler JI, et al. Osteochondritis dissecans of the knee: pathoanatomy, epidemiology, and diagnosis. Clin Sports Med. 2014;33:181–188.

Cross-Reference Musculoskeletal Imaging: The Requisites, 4th ed, 53–58.

CASE 38

Figure 38-1

Figure 38-2

HISTORY:  An asymptomatic 38-year-old patient with systemic lupus erythematosus.

3. What is the most reliable ACUTE finding on magnetic resonance (MR) imaging? A. Articular collapse B. Double line sign C. Bone marrow edema D. Low signal intensity periphery

1.  What conditions would you consider? (Choose all that apply.) A. Fungal infection B. Avascular necrosis C. Corticosteroid use D. Radiation osteitis E. Multifocal bone infarcts . All of the following etiologic factors are accepted EXCEPT: 2 A. Thrombosis or intraluminal obstruction B. Vascular compression C. Physical disruption of vessel D. Vasodilation

. How early can a bone infarct be detected on MR imaging? 4 A. 1 to 2 hours B. 6 to 12 hours C. 24 to 48 hours D. 3 to 4 days

See the Supplemental Figures section for additional figures and legends for this case.

77

ANSWERS CASE 38 Multifocal Bone Infarcts 1. B, C, and E. This patient exhibits the classic findings of multiple bone infarcts characterized by radiolucent areas surrounded by serpiginous sclerosis within the medullary cavity of the affected bones.The most common cause of this presentation is a patient who is on chronic corticosteroid medication. 2. D. The cause of bone infarction is significant reduction or cessation of the blood supply that can be caused by thrombosis, vascular compression, vasospasm, disruption of the feeding vessels, or any combination of these factors. Vasodilation increases blood flow. 3. C. Bone marrow edema surrounding the infarcting bone is the most reliable sign of an acute event. An inflammatory response surrounds the devitalized bone and is characterized by edema surrounding the lesion. 4. B. The ischemic changes in the bone are apparent within 6 to 12 hours of the event.

Comments Clinical Considerations A bone infarct is an area of bone that has undergone ischemic death of the cellular elements of both the bone and marrow. Upon interruption of the vascular supply, the hematopoietic elements are the first to undergo anoxic death in 6 to 12 hours, followed by bone cells (osteocytes, osteoclasts, and osteoblasts) within 12 to 48 hours. The marrow lipoid cells subsequently die within 2 to 5 days. In general, lesions in the metaphysis and diaphysis of bone are called infarcts, while lesions in the subchondral region of the epiphysis are referred to as avascular necrosis.

Figure S38-1  Frontal radiograph shows numerous infarcts in the distal femur and proximal tibia characterized by serpiginous sclerosis surrounding radiolucent areas (asterisk).

Corticosteroid Medication The association between osteonecrosis of bone and corticosteroid administration is important and likely related to the total intake. The maximum daily dose may be the most important factor, although no known safe dosage exists.A short-term, highdose regimen is more likely to cause an infarction than lowerdose regimens of longer duration.There are several hypotheses for its cause. A widely accepted theory suggests that corticosteroids induce adipose metamorphosis in the liver and hyperlipidemia. This leads to fat embolization and vascular occlusion in the bone. Another theory suggests that corticosteroid administration produces a hypercoagulable state of blood, which is associated with vasculitis. Yet another theory suggests that corticosteroids induce osteoporotic trabecular fractures which compress the subchondral vascularity.

Imaging Findings The classic radiographic appearance of a bone infarct in long bones is an area of radiolucency surrounded by a serpiginous rim of sclerosis (Figures S38-1 and S38-2). Acutely, periostitis may be evident with very few other associated findings (Figure S38-3). In flat bones, patchy areas of mixed sclerosis and lucency are characteristic. On magnetic resonance imaging, acute findings are bone marrow edema surrounding an irregular region of mixed signal intensity. As the lesion becomes more chronic, the edema subsides and the peripheral rim becomes more sharply delineated (Figure S38-4). A change in

78

Figure S38-2  Lateral view shows a subtle area of sclerosis in the subchondral region of the medial femoral condyle (arrow).

its contour may indicate malignant transformation (Figures S38-5 and S38-6). Reference Salesi M, Karimifar M, Mottaghi P, et al. A case of SLE with bilateral osteonecrosis of femoral heads and bone infarct in distal of femur. Rheumatol Int. 2010;30:527–529.

Cross-Reference Musculoskeletal Imaging: The Requisites, 4th ed, 306.

CASE 39

Figure 39-1

Figure 39-2

HISTORY:  A 17-year-old boy presents with increasing ankle pain.

. What are the MOST COMMON bacteria seen in osteomyelitis? 3 A.  Staphylococcus aureus B.  Streptococcus pyogenes C.  Salmonella D.  Pseudomonas

1. What entities can be included in the differential diagnosis? (Choose all that apply.) A. Langerhans cell histiocytosis B. Osteomyelitis C. Giant cell tumor D. Osteosarcoma E. Ewing sarcoma 2. In children, which site is LEAST likely for hematogenous osteomyelitis? A. Femur B. Tibia C. Humerus D. Spine

4. In adults, what percentage of osteomyelitis is considered to be of hematogenous source? A. Less than 5% B. 10% C. 20% D. 30%

See the Supplemental Figures section for additional figures and legends for this case.

79

ANSWERS CASE 39 Hematogenous Osteomyelitis 1. A, B, and E. The patient is nearly skeletally mature. The entities to consider include Langerhans cell histiocytosis (eosinophilic granuloma), osteomyelitis, metastasis, and occasionally, Ewing sarcoma. 2. D. In children, the most likely site of hematogenous infection is within one of the long bones, such as the femur, tibia, or humerus. These bones have extensive blood circulation, making them more vulnerable to invasion by bacteria. But in adults, the long bones are not as well vascularized as the vertebral bodies, which do receive an abundant blood flow; thus, hematogenous osteomylelitis in adults is likely to affect the spine. 3. A. S. aureus is the most common organism involved in osteomyelitis in general. However, in patients with hematogenous spread of infection, nearly 95% are infected with this bacteria. Pseudomonas is an important consideration in intravenous drug abusers. Salmonella is an important consideration in patients with sickle cell disease. 4. C. The pathogenesis of about 20% of adult patients with osteomyelitis is hematogenous.

Comments

Figure S39-1  Frontal radiograph shows a large lytic lesion in the metaphysis of the tibia (asterisk) with adjacent soft tissue swelling (arrows). The epiphysis was not involved.

Pathophysiology Hematogenous osteomyelitis is generally regarded as a disease of childhood. Clinically, the disease is associated with a sudden onset of fever and focal inflammation. In many instances, there is no systemic sign of infection and suspicion is aroused only by the presence of soft tissue swelling, diminished range of motion, or a joint effusion. Bacteria (group B streptococci or S. aureus) are the major cause of osteomyelitis in neonates and infants, while S. aureus is overwhelmingly the cause in children. The bacteria may be introduced by vascular catheterization, monitoring devices, and repeated venipuncture, eventually settling in the terminal capillaries of the metaphysis. Infection in drug addicts is often from organisms in the Pseudomonas, Klebsiella, and Enterobacteriaceae groups.

Vascular Supply Hematogenous osteomyelitis affects the tubular bones of the lower extremity in 75% of cases, largely owing to the vascular anatomy. In infants, some of the metaphyseal vessels penetrate the growth plate and form anastomoses to the epiphyseal vessels, resulting in osteomyelitis that involves the end of the bone and joint. In older children, terminal vessels occur as loops with sluggish blood flow in the metaphyses that do not cross the growth plate, so that the metaphysis is the most common site of infection.

Imaging Findings Radiographically, the initial infectious process may be insidious. As the infection progresses, destructive changes in the bone develop, as does soft tissue swelling. The main observation in children with hematogenous osteomyelitis is single or multiple osteolytic areas in the metaphysis (Figures S39-1 to S39-3). Periostitis may occur as well if the cortex is penetrated. Magnetic resonance imaging depicts areas of intense bone marrow edema surrounding the site of infection. Langerhans cell histiocytosis, and occasionally, Ewing sarcoma

80

Figure S39-2  Lateral radiograph shows another well-defined lytic lesion located anterior to the larger lesion (arrow).

(Figure S39-4) may have similar clinical and radiographic presentations. Reference Karmazyn B. Imaging approach to acute hematogenous osteomyelitis in children: an update. Semin Ultrasound CT MR. 2010;31:100–106.

Cross-Reference Musculoskeletal Imaging: The Requisites, 4th ed, 473–485.

CASE 40

Figure 40-1

Figure 40-2

HISTORY:  This is a 23-year-old African-American patient with an underlying condition.

3. All of the following conditions typically cause periarticular calcifications EXCEPT: A. Hyperparathyroidism B. Hypervitaminosis D C. Milk-alkali syndrome D. Gout

1.  What should be included in the differential diagnosis? (Choose all that apply.) A. Idiopathic tumoral calcinosis B. Hyperparathyroidism C. Metastatic calcification D. Collagen vascular disease E. Hypothyroidism 2. In patients with the idiopathic form, what clinical information is most useful? A. Age and race of patient B. Blood urea nitrogen and creatinine levels C. Serum calcium and phosphorus D. Serum uric acid level

4. These joints are commonly involved in idiopathic tumoral calcinosis EXCEPT: A. Shoulders B. Elbows C. Hips D. Knees

See the Supplemental Figures section for additional figures and legends for this case.

81

ANSWERS CASE 40 Tumoral Calcinosis 1. A, B, C, and D. The differential diagnosis would include scleroderma, hyperparathyroidism, renal osteodystrophy, hypervitaminosis D, milk-alkali syndrome, dermatomyositis, polymyositis, and sarcoidosis. Hypothyroidism is characterized by delayed skeletal maturation. 2. A. Idiopathic tumoral calcinosis is rare, affects young people, and is predominantly seen in blacks. Patients usually have normal serum calcium plus phosphorus and no evidence of renal, metabolic, or collagen vascular disease. 3. D. Gout does not usually cause soft tissue calcifications. 4. D. The knees are an unusual site of involvement in patients with the idiopathic form of the disease.

Comments Clinical Considerations Idiopathic tumoral calcinosis is a rare condition characterized by mass like deposits of calcium salts around joints. The etiology of this disorder is a defect in phosphate processing in the proximal renal tubules. One third of cases are familial and transmitted as an autosomal dominant disorder. The deposition of these calcific masses is painless and typically involves the shoulders, hips, and elbows. Patients affected are usually young, presenting between the ages of 6 and 25 years, and there is a strong affinity for blacks. In general no serum abnormality is evident, although rarely alkaline phosphatase and phosphate levels may be elevated.

Imaging Features Radiographically, calcium deposits are extracapsular in location and appear as radiodense masses surrounding a joint (Figures S40-1 to S40-3). The masses tend to enlarge from small, calcified nodules to large, solid, lobulated calcific lesions with smooth peripheral margins. The bony structures beneath

Figure S40-2  The lateral radiograph shows the multilobular appearance (arrows) characteristic of idiopathic tumoral calcinosis.

these masses usually are normal, that is, without erosion or osseous destruction. Occasionally, a fluid-fluid level may be evident in cystic lesions (Figure S40-4), referred to as a sedimentation sign, which may be more conspicuous on computed tomography.

Physical Examination When the calcific masses are large, the range of motion of the adjacent joint may be limited and the overlying skin may break down, forming a sinus that drains a viscous, chalky material.

Differential Diagnosis Periarticular calcified soft tissue masses may be seen in patients with scleroderma (especially the digits), dermatomyositis, and systemic lupus erythematosus. Hypervitaminosis D and milkalkali syndrome are not common but remain important considerations, emphasizing the importance of a complete medical history. Calcific tendinitis may cause periarticular calcifications, but usually these are small and localized in the tendons or bursal cavities. Calcification of a gouty tophus is an unusual finding and usually reflects a coexisting abnormality of calcium metabolism. Reference Olsen KM, Chew FS.Tumoral calcinosis: pearls, polemics, and alternative possibilities. Radiographics. 2006;26:871–885.

Cross-Reference Musculoskeletal Imaging: The Requisites, 4th ed, 336–337.

Figure S40-1  Frontal foot radiograph shows dense homogeneous calcific deposits (arrows) around the first metatarsophalangeal joint.

82

CASE 41

Figure 41-1

Figure 41-2

HISTORY:  A 62-year-old transplant patient presented with symptoms in several joints.

3. What is the population that is generally affected? A. Men B. Middle-aged adults C. African-Americans D. Diabetics

1.  What should be included in the differential diagnosis? (Choose all that apply.) A. Vascular insufficiency B. Hypertrophic osteoarthropathy C. Thyroid acropachy D. Pachydermoperiostosis E. Multifocal nodular periostitis 2. In multifocal nodular periostitis, what information is needed for diagnosis? A. Medication history B. Age of patient C. Distribution of disease D. Gender

. What is likely to happen with the cessation of voriconazole? 4 A. Stabilization of the periostitis B. Progression to chronic symptoms C. Increased bone fragility D. Resolution of the periostitis

See the Supplemental Figures section for additional figures and legends for this case.

85

ANSWERS CASE 41 Multifocal Nodular Periostitis 1. B and E. There is diffuse nodular periostitis. Pain renders hypertrophic osteoarthropathy as a potential diagnosis.Thyroid acropachy causes swelling and digital clubbing but not pain, and the periostitis is more common in the small tubular bones. Vascular insufficiency affects the lower extremities. Pachydermoperiostosis affects young boys, with a predilection for African-Americans. 2. A. Multifocal periostitis is a complication of the antifungal medication voriconazole and produces nodular periostitis in both short and long tubular bones as well as the axial skeleton. 3. B. Most people affected are middle-aged adults with chronic voriconazole exposure. Therefore, anyone with a fungal disease can be at risk, including transplant patients, immunocompromised patients, and patients on chemotherapy. 4. D. The treatment is cessation of the antifungal therapy. This will result in resolution of symptoms and the radiographic findings.

Comment Clinical Considerations Voriconazole is a medication used to treat fungal infections. It is a strong second-generation triazole antifungal agent. It is commonly used in immunocompromised patients to treat invasive aspergillosis and candidemia, particularly in lung transplant patients. The radiologic and clinical manifestations of drugrelated periostitis are an important cause of multifocal pain and arthralgias. It is most frequently observed in middle-aged patients with chronic voriconazole exposure.

Figure S41-2  In the elbow, chunky periosteal changes are evident in the distal humerus (arrows).

Pathogenesis The mechanism of periostitis is unclear, but one hypothesis is that fluoride toxicity could be the etiology. Fluoride induces bone formation by stimulating osteoblasts. The nodular type of periostitis seen in voriconazole is similar to the description of periostitis deformans in the setting of fluoride excess. Cessation of the drug results in reduction of symptoms and resolution of the periostitis.

Imaging Features Radiographically, multifocal nodular periostitis is characterized by nodular or bulky periosteal changes along the shafts of both short (Figure S41-1) and long tubular bones (Figure S41-2); the periostitis may be extensive at times. It may also affect the axial skeleton (Figure S41-3), which is useful in differentiating this condition from other causes of diffuse periostitis. Scintigraphy demonstrates radionuclide uptake in active areas of periostitis (Figure S41-4).

Differential Diagnosis The differential diagnosis includes hypertrophic osteoarthropathy, pachydermoperiostosis, thyroid acropachy, and venous stasis. Hypertrophic osteoarthropathy (Figure S41-5) is another cause of painful periostitis; however, nodular periostitis tends to be much more extensive in distribution and also affects the axial skeleton. In thyroid acropachy, the periostitis is asymptomatic and occurs principally in the small bones of the hands and feet. It does not involve the long tubular bones. Other clinical findings such as exophthalmos and pretibial myxedema generally are present as well. Periosteal reaction in venous stasis (Figure S41-6) tends to occur in the lower extremities. Reference

Figure S41-1  Wrist radiograph shows nodular periostitis in the radius/ulna (arrows) as well as the second (arrow), fourth, and fifth metacarpal bones.

86

Chen L, Mulligan ME. Medication-induced periostitis in lung transplant patients: periostitis deformans revisited. Skeletal Radiol. 2011;40:143–148.

Cross-Reference Musculoskeletal Imaging: The Requisites, 4th ed, 358-359.

CASE 42

Figure 42-1

Figure 42-2

HISTORY:  A 35-year-old with a chronic condition.

3. What is a relatively early indicator of neuroarthropathy in the foot in a patient with diabetes? A. Joint subluxation B. Atherosclerosis C. Marginal erosions D. Insufficiency fractures

1.  What should be included in the differential diagnosis? (Choose all that apply.) A. Syringomyelia B. Diabetes C. Scleroderma D. Syphilis E. Spina bifida 2. All of the following are characteristic features of the hypertrophic form of this condition EXCEPT: A. Decreased density B. Distention of joint C. Intraarticular debris D. Dislocation

4. What causes the changes of neuroarthropathy in the feet of diabetics? A.  Loss of neurologic response to pain and lack of proprioception B. Small vessel thrombotic disease C. Poor joint nutrition D. Increased osteoclastic bone resorption leading to bone fragility

See the Supplemental Figures section for additional figures and legends for this case.

87

ANSWERS CASE 42 Neuroarthropathy (Atrophic) 1. A, D, and E. The finding shown is typical of atrophic neuropathic arthropathy. In the shoulder, considerations are syrinx, neurosyphilis, spinal cord tumor, myelomeningocele, and congenital indifference to pain. Diabetes, leprosy, peripheral nerve injury, collagen vascular disease, and synovial inflammatory arthropathies cause peripheral neuroarthropathy. 2. A. Recall the five D’s of hypertrophic neuroarthropathy: increased density, joint distention, intraarticular and periarticular debris, joint disorganization, and joint dislocation. 3. D. Fractures constitute a relatively early manifestation of neuroarthropathy, although they may occur in the absence of trauma or unusual activity. In fact, unrecognized fractures occur in as many as 22% of diabetic patients with neuropathy. 4. A. The loss of the protective sensations of pain and proprioception leads to chronic destabilization of the joint, thus producing abnormal loading and repetitive injury. The end result is malalignment and degeneration of the joint and eventual disorganization and destruction of the articulation.

Comment Pathogenesis Neuropathic osteoarthropathy, or neuroarthropathy, is a complex complication of neuropathic disease that can progress to severe joint destruction. The pathogenesis of neuropathic disorders remains debatable, but many experts consider repetitive trauma and altered sympathetic control of blood flow to be

Figure S42-2  The Grashey view also shows that there was lysis of the distal clavicle (arrow). There was no prior surgery.

significant factors. Loss of the protective sensations of pain and proprioception leads to chronic destabilization of the joint.The resultant changes in the articular cartilage cause fibrillation and degeneration and eventual erosion of the cortical surfaces. Joint effusion and soft tissue swelling are common clinical features.

Imaging Presentations Neuroarthropathy has three presentations: hypertrophic, atrophic, and mixed. In atrophic neuroarthropathy (Figures S42-1 and S42-2), which constitute 40% of neuropathic joints, there is resorption of the bone, producing a well-demarcated transition between the lysed and remaining bone.This presentation is most common in the shoulder, knee, and hip.There is usually absence of prior fracture, the bone density at the margin of lysis is normal, and there is no indication of surgery.This presentation of neuroarthropathy is particularly common in the upper extremity and is associated with syringomyelia or peripheral nerve injury. In the lower extremity, the hip and the knee are common locations of neuroarthropathy, which is more likely to be caused by either tabes dorsalis or syringomyelia. In the hip (Figure S42-3), soft tissue swelling caused by an effusion and capsular hypertrophy, and intraarticular debris, are important clues to the diagnosis. Involvement of the foot and ankle is nearly always mixed or hypertrophic and caused by diabetes mellitus or alcoholism. Weight bearing is considered an essential factor for the development of hypertrophic neuroarthropathy (Figure S42-4). However, only 20% of neuropathic joints are purely hypertrophic. Reference Figure S42-1  Shoulder radiograph shows diffuse osteopenia and resorption of the humeral head producing a well-demarcated transition between the lysed bone and the shaft (arrows). This patient had a syrinx.

88

Dogan BE, Sahin G, Yagmurlu B, Erden I. Neuroarthropathy of the extremities: magnetic resonance imaging features. Curr Probl Diagn Radiol. 2003;32:227–232.

Cross-Reference Musculoskeletal Imaging: The Requisites, 4th ed, 311-314.

CASE 43

Figure 43-1

Figure 43-2

HISTORY:  A 24-year-old presents with knee pain after a motor vehicle collision.

1.  What should be included in the differential diagnosis? (Choose all that apply.) A. Intraarticular body B. Meniscal ossicle C. Focal nodular synovitis D. Intercondylar eminence fracture E. Tibial spine avulsion fracture

. What finding is most specific for an intraarticular fracture? 3 A. Hoffa fat pad edema B. Joint effusion C. Pain on bearing weight D. Lipohemarthrosis

2. Tibial avulsion fractures related to the anterior cruciate ligament (ACL) have a higher incidence in which population? A. Children B. Men

C. Women D. Elderly

4. What percent of posterior cruciate ligament (PCL) ruptures involve a bony avulsion? A. None B. About 5% C. About 15% D. About 25%

See the Supplemental Figures section for additional figures and legends for this case.

89

ANSWERS CASE 43 Intercondylar (Tibial) Eminence Fracture 1. A, D, and E. The ossific density represents an avulsion fracture of the anteromedial aspect of the tibial eminence, disrupting the medial spine. It may mimic an intraarticular body, because the anterior recess is a location where they commonly reside. 2. A. Avulsion of the ACL is one the most common injuries in children. It results from traumatic traction by the ACL on the anterior aspect of the intercondylar eminence. It is considered uncommon in adults and requires a high-velocity mechanism of injury, such as a motor vehicle injury or fall from a great height. 3. D. A lipohemarthrosis indicates that there is marrow fat in a joint effusion. 4. B. About 6% of PCL tears are associated with an avulsion fracture.

Comment Incidence An ACL avulsion fracture occurs almost exclusively at the tibial eminence. This injury has a higher incidence in the pediatric population. The etiology in children is either a fall or an injury that induces forceful hyperextension of the knee, or a direct blow upon the distal femur with the knee in flexion.The increased tension on the ACL often results in partial tearing of the ligament prior to the development of the tibial eminence fracture.

Figure S43-2  The lateral radiograph shows a lipohemarthrosis (asterisk) and an elongated fragment of bone in the anterior aspect of the knee joint (arrow).

Imaging Features Four types of ACL avulsion fractures have been categorized based on the Meyers-McKeever classification. In a type I injury, there is an incomplete avulsion fracture with no (or minimal) displacement in the anterior aspect of the tibial eminence. In type II, the fragment is anteriorly elevated but the fracture is not complete. In type III, there is complete separation of the fragment (Figures S43-1 and 43-2). Type IIIa affects only the ACL attachment, and type IIIb involves the majority of the eminence. In type IV, the fragment is rotated or comminuted. The anteroposterior (AP) view may show a disruption of the tibial eminence, but characterization of the fragment is best performed on the lateral radiograph. Magnetic resonance imaging is useful for characterizing the fracture fragment and especially for differentiating between type II and type III fractures (Figure S43-3).

Differential Diagnosis PCL avulsion fractures are less common and almost always seen in adults.The fracture arises from the posterior tibial attachment of the PCL and requires high velocity, such as that in a motor vehicle accident. It produces a triangular or pyramidal fragment of bone, which usually displaces superiorly into the joint (Figure S43-4). Both ACL and PCL avulsion fractures can mimic an intraarticular body. Reference Miller LS, Yu JS. Radiographic indicators of acute ligament injuries of the knee: a mechanistic approach. Emerg Radiol. 2010;17:435–444. Figure S43-1  Anteroposterior radiograph of the left knee shows a subtle lucency at the base of the medial tibial spine (arrow). No other abnormality is noted.

90

Cross-Reference Musculoskeletal Imaging: The Requisites, 4th ed, 237-239.

CASE 44

Figure 44-2 Figure 44-1

HISTORY:  A 24-year-old presents with lower extremity weakness. 1.  What conditions would you consider? (Choose all that apply.) A. Sickle cell disease B. Osteomalacia C. Thalassemia D. Osteoporosis E. Sarcoidosis 2. What does right upper quadrant abdominal pain often indicate in patients with this condition? A. Hepatitis B. Colitis C. Cholecystitis D. Pancreatitis

3. All of the following are manifestations of extramedullary hematopoiesis EXCEPT: A. Hair-on-end skull B. Posterior mediastinal mass C. Round paraspinal lesions D. Bone-in-bone appearance 4. What should be the primary consideration when a patient with this condition presents with acute cord compression? A. Hemorrhage into the cord B. Epidural extramedullary hematopoiesis C. Compression fracture of a vertebra D. Disk herniation

See the Supplemental Figures section for additional figures and legends for this case.

91

ANSWERS CASE 44 Thalassemia Major 1. C. The skull radiograph shows extramedullary hematopoiesis. The significant findings in the pelvis are widened medullary spaces with thinning of the cortices, coarsening of the trabeculae from marrow hyperplasia, and diffuse osteopenia. 2. C. Patients with thalassemia major are susceptible to forming gallstones. 3. D. Manifestations of extramedullary hematopoiesis in patients with thalassemia major include “hair-on-end” appearance of the skull, posterior mediastinal masses, and paraspinal lesions outside the thorax. A bone-in-bone appearance is associated with medullary infarcts. 4. B. Extramedullary hematopoiesis lesions in the epidural space can cause cord compression, particularly in the thoracic and lumbar spine.

Comment Pathologic Considerations Thalassemia syndromes are a group of disorders of hemoglobin synthesis characterized by a decreased production of either the alpha or beta polypeptide chains of the hemoglobin molecules, as a result of markedly decreased amounts of globin messenger ribonucleic acid. Beta-thalassemia major is the most common. Decreased or absent production of beta chains leads to decreased synthesis of total hemoglobin, producing severe hypochromic anemia. Inclusion bodies in erythrocytes from excess alpha chains lead to hemolysis of the erythrocytes and ineffective hematopoiesis. This is associated with marrow hyperactivity, with excess production of fetal hemoglobin.

Imaging Findings Marrow hypertrophy and extramedullary hematopoiesis result in hepatosplenomegaly, skull (Figure S44-1) and facial deformities, pathologic fractures, and growth retardation. The osseous structures also appear osteopenic, with thin cortices and coarse trabeculation (Figures S44-2 to S44-4). Medullary expansion resulting in widening of the ribs is a common finding (Figure S44-5).

Figure S44-2  Anteroposterior radiograph of the pelvis shows characteristic findings of thalassemia major with diffuse osteopenia, thinning of the cortices, and coarsening of the trabeculation in the innominate bones (asterisk) and proximal femora.

Extramedullary Hematopoiesis The appearance of intrathoracic extramedullary hematopoiesis is that of a paraspinal mass in the posterior mediastinum at the level of the middle to lower thorax (Figure S44-6). The process may be either unilateral or bilateral. These masses generally appear rounded or lobulated and do not contain calcifications. In thalassemia, the hair-on-end appearance of the skull from marrow expansion occurs with maxillary hypertrophy and forward displacement of the incisors, producing a characteristic “rodent facies” appearance. Patients with polycythemia, myelofibrosis, leukemia, Hodgkin disease, and other marrow replacement processes also occasionally demonstrate a need for extramedullary hematopoiesis. Reference Dinan D, Epelman M, Guimaraes CV, Donnelly LF, Naqasubramanian R, Chauvin NA.The current state of imaging pediatric hemoglobinopathies. Semin Ultrasound CT MR. 2013;34:493–515.

Cross-Reference Musculoskeletal Imaging: The Requisites, 4th ed, 566-569.

Figure S44-1  Lateral skull radiograph shows a hair-on-end appearance (arrows) with perpendicular spicules of bony trabeculation from increased hematopoietic activity in the medullary cavity.

92

CASE 45

Figure 45-2

Figure 45-1

HISTORY:  A 22-year-old fell from a 40-foot scaffold. 1.  What should be included in the differential diagnosis? (Choose all that apply.) A. L1 transverse process fracture B. Fracture of the L1 lamina C. Superior end plate fracture L1 D. L1-L2 facet joint subluxation E. Rupture of the T12-L1 posterior longitudinal ligament 2.  What percentage of spine fractures are Chance-type fractures? A. Less than 2% B. 10%

C. 20% D. 30%



3. Regarding Chance fractures, where do most occur? A. T10-11 B. T12-L1 C. L2-L3 D. L4-L5 . What is the “empty hole” sign? 4 A. Disruption of the neural foramen on lateral view B. Loss of pedicle margin on anteroposterior (AP) view C. Enlargement of the interlaminar space D. Widening of the interspinal distance on AP view

See the Supplemental Figures section for additional figures and legends for this case.

93

ANSWERS CASE 45 Chance Fracture 1. A, C, D, and E. This patient has a Chance fracture at the T12-L1 level associated with a fracture of the right L1 transverse process, superior end plate compression fracture of L1, and anterior facet joint subluxation with disruption of the posterior longitudinal ligament at T12-L1. 2. B. This fracture used to be more common with lap belt– type restraints but still accounts for about 10% of all spine fractures. 3. C. About half of Chance fractures occur at the L2 and L3 levels. 4. D. The “empty hole” sign is created when the interspinal distance is increased, giving the appearance of a large hole in the lamina of the vertebra in the AP view.

Comment Variations and Mechanism of Injury The Chance fracture was initially described as a horizontal fracture that began in the spinous process and neural arch, extended into the vertebral body, and exited through the superior end plate anterior to the neural foramen. Since then, several patterns have been described ranging from purely osseous, potentially involving all three columns, to purely ligamentous (Figures S45-1 to S45-5). Compression of the vertebral body is not a major finding when observed. The mechanism of injury is related to the anterior position of the fulcrum of force during forced flexion of the torso. The restraint of a lap seatbelt or a fall onto the abdomen results in only tensile forces on the spine, so that the primary injury is distraction.

Figure S45-2  Lateral view shows anterior subluxation of T12 and a narrowed disk space. There is minimal compression of the superior end plate of L1 (arrow).

Diagnostic Signs A good lateral view generally is diagnostic, but often this cannot be obtained in a patient who has sustained multiple traumas. The diagnosis of a Chance-type injury can be achieved through careful inspection of an AP radiograph of the thoracolumbar spine. Pertinent observations include an increased interspinal distance (empty hole sign), splitting of the pedicle and/or transverse processes, fracture of the spinous process, fracture of the lamina, widening of the disk space or facet joints, and widening of the intercostal space. Computed tomography (CT) with sagittal reformatted images is helpful for preoperative planning. Classic CT signs include naked facet sign from facet distraction and dissolving pedicle sign as sequential axial CT images traverse the pedicle fracture.

Clinical Considerations About 40% of patients with this type of injury have associated intra-abdominal injuries, most commonly involving the bowel and mesentery, both of which may be subtle on imaging. Reference Bernstein MP, Mirvis SE, Shanmuganathan K. Chance-type fractures of the thoracolumbar spine: imaging analysis in 53 patients. AJR Am J Roentgenol. 2006;187:859–868.

Cross-Reference Musculoskeletal Imaging: The Requisites, 4th ed, 153–154.

Figure S45-1  AP radiograph shows widening of the interspinous space between the spinous process of T12 and L1 (asterisk). There is a fracture of the right L1 transverse process (arrow).

94

CASE 46

Figure 46-2 Figure 46-1

HISTORY:  A 25-year-old with a birth anomaly. 1. What is the differential diagnosis? (Choose all that apply.) A. Sacral agenesis B. Spinal dysraphism C. Caudal regression syndrome D. Tethered spinal cord 2. All of the following have been suggested as etiologic factors for caudal regression syndrome EXCEPT: A. Maternal diabetes mellitus B. Congenital dermal rest C. Genetic predisposition D. Vascular hypoperfusion

3. All of the following are genitourinary manifestations of caudal regression syndrome EXCEPT: A. Anal atresia B. Malformed external genitalia C. Neurogenic bladder or renal aplasia D. Absent colon 4. What is sirenomelia? A. Absence of a portion of one of the lower extremities B. Absence of a portion of both lower extremities C. Congenital absence of the lower extremities D. Congenital fusion of the lower extremities

See the Supplemental Figures section for additional figures and legends for this case.

95

ANSWERS CASE 46 Caudal Regression Syndrome 1. A, B, and C. The main finding is a midline closure defect of the sacrum or spinal dysraphism. This can be a manifestation of sacral agenesis or caudal regression syndrome with fusion of the caudal-most vertebrae. Although often associated with a tethered cord, that is not always present and patients should undergo imaging of their entire neuraxis to search for this anomaly. 2. B. A congenital dermal rest refers to a spine dermoid, a multilocular cystic tumor lined by squamous epithelium containing skin appendages. This condition is not related to caudal regression syndrome or sacral agenesis. 3. D. Clinically, findings depend on the severity of disease expression and include leg weakness, neurogenic bladder if two sacral segments are missing, bowel and bladder incontinence, anorectal atresia, malformed external genitalia, renal aplasia, and pulmonary hypoplasia. 4. D. Sirenomelia, or mermaid syndrome, refers to the congenital fusion of the lower extremities.

Figure S46-2  Axial computed tomographic image shows the spinal dysraphism (arrow) characteristic of this syndrome.

Imaging Findings

Comment Clinical Syndrome The caudal regression syndrome is a spectrum of caudal axial skeletal and associated neurologic and soft tissue defects caused by an insult to the developing caudal mesoderm and ectoderm during the first trimester. Defects include partial to complete sacral agenesis, spina bifida (Figures S46-1 and S46-2), spinal stenosis, an angular and wedge-shaped conus medullaris, tethering of the cord, and a presacral meningocele. Clinically, findings depend on the severity of disease expression and include leg weakness, bowel and bladder incontinence, anorectal atresia, renal aplasia, and pulmonary hypoplasia. Patients with sirenomelia, congenital fusion of the lower extremities, have presented with caudal regression syndrome, but this is likely incidental and not part of the syndrome.

Development of the sacrum in this syndrome is variable and may only affect the distal segments (Figure S46-3), half of the sacrum, or nearly all of its components. Fusion anomalies are not uncommon. In the absence of the sacrum, the ilia articulate with the lumbar spine, drawing these bones to the midline so that the pelvis appears narrowed. Scoliosis is common and seen in over 50% of cases. Reference Boulas MM. Recognition of caudal regression syndrome. Adv Neonatal Care. 2009;9:61–69.

Cross-Reference Musculoskeletal Imaging: The Requisites, 4th ed, 509–511.

Figure S46-1  Anteroposterior radiograph of the pelvis shows a hypoplastic sacrum and a large midline fusion defect of L5 and the sacral segments (arrow).

96

http://radiologyebook.com

CASE 47

Figure 47-1

Figure 47-2

HISTORY:  A 20-year-old man presents with a hand mass.

3.  What is the rate of sarcomatous degeneration in these patients? A. None B. 1% to 25% C. 25% to 50% D. 50% to 75%

1. What is the BEST diagnosis? A. Osteosarcomatosis B. Fibrous dysplasia C. Ollier disease D. Hereditary exostoses E. Enchondromatosis

. What is Maffucci syndrome? 4 A. Multiple osteochondromas and soft tissue hemangiomas B. Enchondromatosis and soft tissue hemangiomas C. Multiple osteochondromas and osseous hemangiomas D. Enchondromatosis and osseous hemangiomas

2. What is going on in this patient’s fifth finger? A. Sarcomatous degeneration B. Pathologic fracture C. Infection D. Enlarging preexisting enchondroma

See the Supplemental Figures section for additional figures and legends for this case.

http://radiologyebook.com

97

ANSWERS CASE 47 Ollier Disease 1. C and E. The main finding is numerous cartilaginous lesions with punctate calcifications deforming the medullary cavities of the majority of phalanges and metacarpals, resulting in endosteal scalloping. This is typical of Ollier disease or enchondromatosis. Osteosarcomatosis typically involves the metaphysis of long bones producing osteoblastic lesions that appear sclerotic. Hereditary multiple exostoses results in the formation of pedunculated and sessile osteochondromas near joints due to involvement of the metaphyseal regions of tubular bones. 2. A. Sarcomatous degeneration with an area of cortical infiltration in the middle phalanx. 3. C. Sarcomatous transformation is high in these patients, in the range of 25% to 50%, with an average reported rate of 30%. In young adults, osteosarcoma is a common lesion, while in older patients, chondrosarcoma and fibrosarcoma are typical lesions. 4. B. Maffucci syndrome is characterized by enchondromatosis, nonhereditary type, with multiple cavernous hemangiomas in the soft tissues. Figure S47-2  Lateral radiograph shows that the cortex is disrupted by a soft tissue mass in the middle phalanx of the fifth finger (arrow).

Comment General Information Enchondromatosis, or Ollier disease, is a condition characterized by the presence of numerous enchondromas, particularly in the hands, such as in this patient. It is a nonhereditary abnormality with a predilection for one side of the body. The source of the disease is abnormal cartilage growth, arising from either the growth plate or metaplastic cartilage from the periosteum.

Imaging Findings Radiographically, the osseous lesions may be located in the medullary cavity of the metacarpals, phalanges (Figures S47-1 and S47-2), or the long bones (Figure S47-3). In the hand, multiple enchondromas produce thinning, expansion, and occasionally disappearance of the cortex, with focal calcifications within the lesions. In the long bones, the intraosseous lesion may be extensive in the diaphysis and metaphysis of the bone, occupying all areas of the bone including the medullary cavity and cortex (Figure S47-4). Computed tomography is useful in depicting chondroid matrix formation (Figure S47-5). In skeletally immature patients, involvement of the epiphysis may cause growth disturbances.

Malignant Transformation The consideration of malignant transformation should be raised when there is progressive destruction and breakthrough of the cortex with infiltration of the adjacent soft tissues beyond the cortical limits on sequential radiographs (Figure S47-6). Reportedly, this can occur in 30% to 50% of cases. The risk of malignancy increases the longer the patient has the disease, and most often the transformation is into a chondrosarcoma, although this often is influenced by the age of the patient. Reference Muthusamy S, Conway SA, Temple HT. Five polyostotic conditions that general orthopedic surgeons should recognize (or should not miss). Orthop Clin North Am. 2014;45:417–429.

Cross-Reference Musculoskeletal Imaging: The Requisites, 4th ed, 386–387.

Figure S47-1  Frontal radiograph shows medullary expansion by cartilaginous tumors in numerous phalanges and metacarpal bones. There is endosteal scalloping at numerous sites with thinning of the cortex (yellow arrows). There are several lesions with chondroid matrix formation (red arrows).

98

http://radiologyebook.com

CASE 48

Figure 48-2 Figure 48-1

HISTORY:  A 19-year-old sustained a foot injury. 1. What should be included in the differential diagnosis? (Choose all that apply.) A. Second metatarsal fracture B. Os intermetatarseum C. Medial cuneiform fracture D. Lisfranc ligament disruption E. Intraarticular body 2. What is a simple maneuver that may confirm the diagnosis when equivocal? A. Perform a magnetic resonance (MR) scan B. Perform a computed tomography (CT) scan C. Obtain radiographs of the other foot D. Perform weight-bearing views

3. What pattern of injury does this patient have? A. Divergent B. Partial incongruity-medial dislocation C. Partial homolateral D. Complete homolateral 4. What percentage of patients will develop arthritic changes to the tarsometatarsal (TMT) joints? A. Less than 5% B. 10% C. 25% D. 50%

See the Supplemental Figures section for additional figures and legends for this case.

http://radiologyebook.com

99

ANSWERS CASE 48 Lisfranc Fracture-Dislocation 1. A, C, and D. The main observation is a small ossific density between the middle cuneiform and the base of the second metatarsal bone. The diagnostic consideration based on the frontal view would include avulsion fractures of either the second metatarsal base or medial cuneiform, representing a rupture of the Lisfranc ligament, and an os intermetatarseum. Close inspection of the lateral view shows dorsal subluxation of the second metatarsal base and dorsal soft tissue swelling. 2. D. Weight-bearing or standing views are a useful maneuver to look for dorsal subluxation that would not be conspicuous on the non–weight-bearing lateral radiographs or for widening of the intermetatarsal or intercuneiform spaces on the frontal projection. 3. C. This patient has a partial homolateral dislocation pattern. 4. C. About 20% to 30% of patients will develop secondary osteoarthritis.

Comment Anatomic Considerations TMT fracture-dislocations are referred to as Lisfranc ligament injuries. Transverse metatarsal ligaments connect the bases of the second through fifth metatarsal bones, but this ligament does not exist between the first and second metatarsal bones. Instead, the base of the second metatarsal bone is attached to the medial cuneiform by an oblique ligament (Lisfranc ligament). Avulsion fractures of the second metatarsal base frequently occur at the enthesis of this ligament (Figures S48-1 and S48-2). Because there is greater support on the plantar surface by the plantar ligaments and tendons, most dislocations occur dorsally, but this may require a standing weight-bearing stress view to become evident. Soft tissue swelling in the dorsum

Figure S48-2  Lateral radiograph shows subtle dorsal subluxation of the second tarsometatarsal joint (arrow).

of the midfoot should increase your index of suspicion. If the space between the first and second metatarsal bases appears widened, then the diagnosis is unequivocal. Evaluation with MR imaging or CT is advocated if the diagnosis is equivocal or if reduction cannot be achieved due to entrapment of either a tendon or a fracture fragment (Figures S48-3 and S48-4).

Types of Injuries Lisfranc injuries may have two main patterns. A homolateral (convergent) pattern occurs when there is lateral displacement of the metatarsal bases. It is complete when it involves all five rays and partial if only one or two rays are involved (Figure S48-5). A divergent pattern occurs when the first metatarsal base displaces medially and the second metatarsal, or a combination of the second to fifth metatarsals, displaces laterally. Medial subluxation of the first ray may occur at the navicular-medial cuneiform articulation (Figure S48-6). Partial incongruity occurs when there is medial subluxation of the first metatarsal. Treatment is aimed at restoring the anatomy. If displacement is less than 2 mm, closed reduction is adequate. More significant fracture-dislocations require open reduction and internal fixation. Reference Nazarenko A, Beltran LS, Bencardino JT. Imaging evaluation of traumatic ligamentous injuries of the ankle and foot. Radiol Clin North Am. 2013;51:455–478.

Cross-Reference Musculoskeletal Imaging: The Requisites, 4th ed, 240–241.

Figure S48-1  Frontal view of the foot shows minimal widening of the space between the first and second metatarsal bases and a small avulsion fracture (arrow).

100

http://radiologyebook.com

CASE 49

Figure 49-1

Figure 49-2

HISTORY:  A 23-year-old was involved in a motor vehicle collision.

3. What is an analogous odontoid fracture type that has this appearance? A. Type 1 odontoid fracture with nonunion B. Type 2 odontoid fracture with nonunion C. Type 3 odontoid fracture with nonunion D. There is no fracture that looks this way

1.  What should be included in the differential diagnosis? (Choose all that apply.) A. Persistent os terminale B. Odontoid aplasia C. Odontoid fracture D. Os odontoideum 2. What determines atlantoaxial instability when an os odontoideum is observed? A. Size of ossicle B. Juxtaposition of the transverse ligament of C1 to the ossicle C. Presence of C1 anterior arch hypertrophy D. Number of ossicles

4.  All of the following occur with flexion and extension EXCEPT: A. Dislocation of the lateral masses of C1 and C2 B. The ossicle is fixed to the atlas and moves with it C. There is narrowing of the spinal canal with flexion D. The spinolaminar line is normal in extension

See the Supplemental Figures section for additional figures and legends for this case.

http://radiologyebook.com

101

ANSWERS CASE 49 Os Odontoideum 1. A, C, and D.  This patient has a hypoplastic odontoid with an ossicle (os odontoideum) superior to it. Therefore, it would be appropriate to also consider a persistent os terminale and an odontoid fracture in the proper clinical setting. 2. B. Stability is dependent on the level of the cleft between the ossicle and the odontoid process, and on the degree of development of the dens. If the transverse ligament is juxtaposed to the ossicle, then the odontoid process is unable to form a stable relationship with the atlas, a configuration that is conducive to atlantoaxial instability. 3. B. Some patients may actually have a type 2 odontoid fracture with nonunion. 4. A.  The lateral masses may undergo subluxation but not dislocation. All of the other statements are correct.

Comment Developmental Considerations An os odontoideum is a small bone that exists above a hypoplastic odontoid process (Figures S49-1 and S49-2). It has a characteristic appearance: a small, round or oval, well-corticated ossicle located above the tip of the odontoid process or more cephalad near the basion of the skull. When evaluating this anomaly, one clue is to search for coexisting abnormalities of the atlas, including hypoplasia of the posterior neural arch and hypertrophy of the anterior arch. Most experts agree that an os odontoideum represents a congenital anomaly (overgrowth of the os terminale secondary to hypoplasia of the odontoid), although some cases do represent a posttraumatic condition from an unrecognized fracture through the odontoid growth plate before the age of 5 years.

Figure S49-2  Coronal reformatted computed tomographic image of the upper cervical spine shows the multipartite deformity of the odontoid process (arrow).

Types There are two types. In the orthotopic type, there is a normal position with a wide gap between C2 and the ossicle. In the dystopic type, the ossicle is displaced.

Stability Factors Stability depends on where the cleft occurs between the ossicle and the odontoid process, and on the development of the dens. If the transverse ligament is juxtaposed to the ossicle, then the odontoid process is unable to form a stable relationship with the atlas. In severe cases of instability, the caliber of the spinal canal can be significantly compromised. Assessment of stability is performed with flexion and extension lateral radiographs (see Figures S49-1 and S49-3). It is important to quantify the degree of motion of the ossicle and changes in the caliber of the spinal canal. Currently, computed tomography (CT) (Figure S49-4) or magnetic resonance imaging is advocated for further evaluation of the integrity of the transverse ligament. References Matsui H, Imada K, Tsuji H. Radiographic classification of os odontoideum and its clinical significance. Spine. 1997;22:1706–1709. Rozzelle CJ, Aarabi B, Dhall SS, et al. Os odontoideum. Neurosurgery. 2013;72(suppl 2):159–169.

Cross-Reference Musculoskeletal Imaging: The Requisites, 4th ed, 144–145.

Figure S49-1  Lateral radiograph of the atlantoaxial region shows an os odontoideum (arrow). Note the truncated appearance of the odontoid process below the level of the C1 anterior arch (asterisk).

102

http://radiologyebook.com

CASE 50

Figure 50-1

Figure 50-2

HISTORY:  A 25-year-old man serving in the military presents with leg pain.

3.  What is the expected scintigraphic appearance of this condition? A. No activity B. Transverse linear activity C. Fusiform cortical activity D. Circumferential activity

1. What is the differential diagnosis based on the radiograph? (Choose all that apply.) A. Longitudinal stress fracture B. Osteoid osteoma C. Normal nutrient vessel D. Enthesopathy E. Remote fracture 2. Where do most longitudinal stress fractures occur? A. Tibial diaphysis B. Femoral diaphysis C. Tibial metaphysis D. Femoral neck

. What does the tibial “dreaded black line” sign indicate? 4 A.  Diminished density in the cortex of a bone from hyperemia B. Periosteal elevation at the site of a stress fracture C. Transverse fracture across the anterior tibial shaft D. Nonunion of a prior fracture

See the Supplemental Figures section for additional figures and legends for this case.

http://radiologyebook.com

103

ANSWERS CASE 50 Longitudinal Stress Fracture 1. A and B. The radiograph shows a vertical lucency in the posterior cortex of the tibia associated with adjacent periosteal and endosteal reactive changes, contributing to thickening of the cortex. Although a stress fracture is the most likely consideration, an osteoid osteoma cannot be entirely excluded on the appearance alone. Enthesopathy and remote fracture are less likely but could be considered if the vertical lucency was not conspicuous.The magnetic resonance (MR) image confirms the diagnosis, however. 2. A. Longitudinal stress fractures occur nearly exclusively in the diaphysis of the tibia and femur, with the vast majority involving the tibia. 3. C. These stress fractures show fusiform longitudinal activity along the involved cortex. 4. C. The “dreaded black line” refers to a lucency in the anterior tibial shaft indicating the presence of a transverse fracture line. Figure S50-2  Axial proton-density-weighted magnetic resonance image performed 2 days later shows the break in the cortex as a vertically oriented defect in the bone (arrow).

Comment Risk Factors Exercise-induced stress reactions and stress fractures involving the tibia are common and may account for up to 75% of all stress fractures. Detection is important to prevent complications and to lead to early recovery. The pathogenesis of stress fracture is not well understood, but there is stimulation of bone remodeling, initially beginning with increased osteoclastic bone resorption. As the stimulus persists, an imbalance between bone resorption and bone replacement leads to weakening of the bone. Weight bearing, muscle actions, and fatigue may all be contributing factors. Accelerated intracortical remodeling causes microscopic fractures, diminished density, and formation of resorption cavities that may coalesce and ultimately produce failure of the cortical or trabecular bone.

Fredericson Classification Medial stress syndromes of the tibia may be graded by magnetic resonance imaging using the Fredericson classification system. In grade 0, the bone is normal. In grade 1, periosteal edema is noted along the medial tibial cortex. In grade 2, there is periostitis and endosteal edema seen on T2-weighted (T2W) images. In grade 3, the marrow edema is obvious in both T1W and T2W images. In grade 4a, there are intracortical signal intensity changes. In grade 4b, there is a linear region of cortical disruption (Figures S50-1 and S50-2).

Imaging Findings This patient is unusual because the fracture is very evident radiographically. Radiographs are often normal, and 50% remain so throughout treatment. Often, the longitudinal fracture may occur near the nutrient foramen, particularly in the femur. On scintigraphy, fusiform activity along the involved cortex is characteristic (Figure S50-4). A longitudinal stress fracture is depicted on computed tomography as a linear lucency in the cortex associated with periosteal and endosteal callus. Reference Kijowski R, Choi J, Shinki K, Del Rio AM, De Smet A. Validation of MRI classification system for tibial stress injuries. AJR Am J Roentgenol. 2012;198:878–884.

Cross-Reference Musculoskeletal Imaging: The Requisites, 4th ed, 9–13.

Figure S50-1  Lateral radiograph shows fusiform thickening of the posterior cortex of the proximal tibial shaft with a longitudinal lucency oriented vertically within the cortex (arrows).

104

http://radiologyebook.com

CASE 51

Figure 51-2

Figure 51-1

HISTORY:  A 44-year-old man with polyarthralgia. 1. Based on the radiograph, what is the differential diagnosis? (Choose all that apply.) A. Rheumatoid arthritis B. Gout C. Tuberculous arthropathy D. Tophaceous pseudogout 2. What is the time interval between the clinical onset of gout and its typical radiographic manifestations? A. 6 to 8 years B. 4 to 6 years C. 2 to 4 years D. Less than 1 year

3. What do the soft tissue masses in the hand represent? A. Pannus formation B. Deposits of uric acid crystals C. Soft tissue swelling D. Heberden nodes 4. Regarding gout, what is the most specific characteristic of a tophus on magnetic resonance imaging (MRI)? A. Low signal intensity on T1-weighted (T1W) and high signal intensity on T2W images B. High signal intensity on T1W and low signal intensity on T2W images C. Low signal intensity on both T1W and T2W images D. High signal intensity on both T1W and T2W images

See the Supplemental Figures section for additional figures and legends for this case.

http://radiologyebook.com

105

ANSWERS CASE 51 Gout 1. B and D. This patient has typical findings of gouty arthritis in the hand radiograph with juxtaarticular soft tissue masses associated with erosions that have overhanging edges. Occasionally, pyrophosphate arthropathy may present in a similar fashion and therefore is referred to as tophaceous pseudogout. 2. A. Many years separate the onset of gout and its radiographic findings. Most experts agree that it is at least 6 to 8 years, although some consider the time interval to be 7 to 12 years. 3. B. A tophus is a deposit of sodium urate monohydrate crystals. 4. C. A tophus may have several imaging manifestations on MR imaging, but the key to interpreting gout is to identify a lesion that is low in signal intensity on both T1W and T2W images.

Figure S51-2  Coronal T2-weighted magnetic resonance image shows a low signal intensity mass (arrow) representing a gouty tophus.

Comment

S51-1 to S51-3). Calcifications within the tophi are not uncommon, increasing the density of the tophus (Figure S51-4). Joint narrowing generally occurs later in the disease.The most recognizable locations are the first metatarsophalangeal joint of the foot and the joints of the hands.

Pathologic Considerations

Advanced Imaging

Gout is a condition caused by the deposition of urate monohydrate crystals in tissues.There are two forms of gout: an idiopathic form and one that is secondary to other disorders.The idiopathic form is characterized by an overproduction of uric acid due to abnormal renal excretion of urate, caused by a deficiency of the enzyme phosphorylribosyltransferase. There are numerous causes of secondary gout. In some diseases, the increased production of uric acid is caused by an excessive breakdown of nucleoproteins (polycythemia vera, myelofibrosis, leukemia, multiple myeloma, anemias, psoriasis, and glycogen storage disease), whereas in others it is on the basis of renal failure.

On MR imaging, gout can sometimes mimic a neoplastic process, because it can provoke a significant periosteal reaction and bone marrow edema. Destruction of the bone and the presence of juxtaarticular masses (which do enhance) can appear ominous. The key to interpreting gout on MRI is to identify a lesion that is low in signal intensity on both T1W and T2W images (Figure S51-5). Recent advances in dual energy computed tomography have broadened its application with high sensitivity and specificity (Figure S51-6).

Radiographic Findings The radiographic findings in this patient are characteristic of this disease.Tophi may occur in the soft tissues, synovium, cartilage, and bones. Bone erosions usually have a peripheral rim of sclerosis and are characterized by “overhanging” edges (Figures

References Girish G, Glazebrook KN, Jacobson JA. Advanced imaging in gout. AJR Am J Roentgenol. 2013;201:515–525. Yu JS, Chung C, Recht M, Dailiana T, Jurdi R. MR imaging of tophaceous gout. AJR Am J Roentgenol. 1997;168:523–527.

Cross-Reference Musculoskeletal Imaging: The Requisites, 4th ed, 294–297.

Figure S51-1  Radiograph of the hand shows prominent soft tissue tophi (yellow arrows) associated with erosions that are characterized by typical overhanging edges and sclerotic rims (red arrows).

106

http://radiologyebook.com

CASE 52

Figure 52-1

Figure 52-2

HISTORY:  A 33-year-old with shoulder pain.

3. All of these are subtypes of this abnormality EXCEPT: A. Subacromial B. Subglenoid C. Subspinous D. Subcoracoid

. What conditions would you consider? (Choose all that apply.) 1 A. Lesser tuberosity fracture B. Posterior shoulder dislocation C. Osteonecrosis of the humeral head D. Anterior shoulder dislocation E. Trough lesion 2. What percentage of glenohumeral joint dislocations are of this type? A. Less than 5% B. 25% C. 50% D. More than 75%

. How often is a lesser tuberosity fracture an associated finding? 4 A. Less than 5% of cases B. 10% of cases C. 25% of cases D. 50% of cases

See the Supplemental Figures section for additional figures and legends for this case.

http://radiologyebook.com

107

ANSWERS CASE 52 Posterior Shoulder Dislocation 1. A, B, and E. The first radiograph shows a posterior glenohumeral joint dislocation with the humeral head impacted against the posterior glenoid rim, creating a trough defect. The second radiograph was obtained after reduction and shows a lesser tuberosity fracture. 2. A. Less than 5% of glenohumeral joint dislocations are posterior dislocations. 3. D. Remember that the coracoid process is an anterior scapula structure.This location marks the most common subtype of anterior glenohumeral dislocations. 4. C. A lesser tuberosity fracture occurs in 25% of patients with a posterior glenohumeral joint dislocation.

Comment General Considerations A posterior dislocation of the glenohumeral joint is one of the most commonly misdiagnosed injuries in trauma patients.About 50% of these injuries are missed on initial radiographic evaluation. Several factors contribute to the errors in diagnosis. An adequate study may be impossible to perform because the shoulder is locked in internal rotation. Clinically, important findings that are indicative of a dislocation may be masked by a concomitant hematoma, muscle spasm, or associated fractures. Seizure is the most common cause of this injury, but electrocution is also an important cause and both may lead to bilateral dislocations.

Types There are three types of posterior shoulder dislocations; however, the subacromial type is by far the most common. Posterior subglenoid and subspinous dislocations are rare.

Imaging Findings Patients often present with their arm locked in internal rotation. Important radiographic observations include the trough

Figure S52-2  Axillary view shows a trough lesion (arrow) in the anterior surface of the humeral head and a lesser tuberosity fracture (asterisk).

sign (a vertical compression fracture of the anterior humeral head) (Figures S52-1 to S52-4), rim sign (widening of the joint beyond 6 mm), absent half moon sign (loss of humeral head or glenoid rim overlap) (Figure S52-5), Velpeau sign (superior subluxation of humeral head), and disruption of the scapulohumeral arch. Computed tomography (CT) or magnetic resonance imaging may depict a reverse osseous Bankart lesion (Figure S52-6), a tear of the posterior labrum, stripping of the posterior periosteum, and posterior humeral avulsion injuries of the glenohumeral ligaments. A lesser tuberosity fracture of the proximal humerus should raise suspicion for a potential posterior glenohumeral dislocation (see Figure S52-2). Patients with chronic dislocations, particularly in the elderly population, often develop a pseudoarthrosis as the trough lesion widens from mechanical erosion (see Figure S52-4). Reference Saupe N,White LM, Bleakney R, et al.Acute traumatic posterior shoulder dislocation: MR findings. Radiology. 2008;248:185–193.

Cross-Reference Musculoskeletal Imaging: The Requisites, 4th ed, 88–89.

Figure S52-1  Grashey radiograph shows that the humeral head is perched against the back of the glenoid rim, resulting in loss of the joint space and an impaction fracture of the humeral head (arrows).

108

http://radiologyebook.com

CASE 53

Figure 53-1

Figure 53-2

HISTORY:  A 31-year-old farmer with a long-standing medical problem. 1.  What should be included in the differential diagnosis? (Choose all that apply.) A. Involucrum B. Stress fracture C. Osteoid osteoma D. Sequestrum E. Chronic osteomyelitis 2. What would be the most specific imaging technique to further evaluate this patient? A.  Positron emission tomography–computed tomography (PET-CT) and tagged leukocyte scintigraphy B. Magnetic resonance (MR) imaging with contrast C. CT with contrast D. Bone scintigraphy

3. Define a sequestrum. A. New bone beneath elevated periosteum B. Necrotic fragment of bone C. Opening in the involucrum D. Tract communicating with surface . What is a Brodie abscess? 4 A. A fragment of dead bone B. Small perpendicular tunnels in the bone C. A sheath of new bone often around a sequestrum D. An intraosseous abscess

See the Supplemental Figures section for additional figures and legends for this case.

http://radiologyebook.com

109

ANSWERS CASE 53 Chronic Osteomyelitis 1. A, D, and E. This patient has classic manifestations of chronic osteomyelitis with a sequestrum, an involucrum, and a cloaca where the cortex appears thickened. 2. A. Infection can be confirmed with a combination of PET-CT and a tagged white blood cell scan. 3. B. A sequestrum is a necrotic fragment of bone in chronic osteomyelitis. It is more frequently observed in children than in adults. 4. D. A Brodie abscess is an intraosseous abscess seen in subacute osteomyelitis.

Comment General Considerations This is a straightforward case of chronic osteomyelitis (Figures S53-1 and S53-2).When a bone infection progresses to a chronic infection, it can be associated with significant periosteal bone formation, giving rise to a markedly thickened cortex of variable density. Often, the bony trabecular pattern alters also with an increased number and size of spongy trabeculae, which further adds to the density of the bone. An involucrum (reactive new bone beneath an elevated periosteum) may form around a sequestrum (a necrotic piece of bone), producing dense periosteal changes.  A cloaca is an opening in the involucrum, and this can communicate to the skin surface through a sinus tract. If this occurs for a period of years, the development of squamous

Figure S53-2  Lateral view shows an involucrum (arrow), which is reactive new bone underneath the elevated periosteum.

cell carcinoma in the sinus tract is an important consideration to keep in mind. Although the bone appears denser, it is structurally weakened by the infection and vulnerable to the formation of a pathologic fracture, especially if there is increased stress on the bone.

Brodie Abscess A Brodie abscess is a painful lesion characterized by a welldefined geographic, radiolucent focus surrounded by diffuse sclerosis (Figures S53-4 to S53-6). Two thirds of these lesions occur in the metaphysis of a bone, and the remaining occur in the diaphysis; although rarely the epiphysis can be involved. Although most lesions present as an intramedullary lytic lesion along the long axis of the bone, occasionally it is cortically based with associated periosteal reactive changes. A Brodie abscess represents a cavity in the bone lined by inflammatory granulation tissue and is filled with purulent or mucoid fluid. It is hypothesized that the bone abscess develops when the organism has a reduced virulence or the host demonstrates increased resistance to the infection. About 20% of patients with a Brodie abscess will eventually develop a sequestrum. Reference Pineda C,Vargas A, Rodriguez AV. Imaging of osteomyelitis: current concepts. Infect Dis Clin North Am. 2006;20:789–825. Figure S53-1  Frontal radiograph of femur shows fusiform thickening of the posteromedial cortex (asterisk) with a sequestrum (arrow) extruding from the medullary cavity through a cloaca.

110

Cross-Reference Musculoskeletal Imaging: The Requisites, 4th ed, 480–481.

http://radiologyebook.com

CASE 54

Figure 54-1

Figure 54-2

HISTORY:  A 40-year-old man presents with intense left hip pain. . What is the differential diagnosis? (Choose all that apply.) 1 A. Transient osteoporosis of the hip (TOH) B. Regional migratory osteoporosis C. Osteonecrosis D. Septic arthritis E. Insufficiency fracture 2. What condition has idiopathic TOH been linked to? A. Synovitis B. Autoimmune disorder C. Repetitive trauma D. Insufficiency fracture

3. All of the following are characteristic magnetic resonance imaging (MRI) findings EXCEPT: A. Low T1 and high T2 signal intensity in the marrow of the femoral head B. No enhancement after intravenous (IV) contrast C. Hypointense subchondral fracture line paralleling cortex D. Normal cortex and overlying cartilage 4. In TOH, when does the marrow edema resolve? A. Never B. 2 to 6 months C. 6 to 10 months D. After 1 year

See the Supplemental Figures section for additional figures and legends for this case.

http://radiologyebook.com

111

ANSWERS CASE 54 Transient Osteoporosis of the Hip 1. A, B, C, and E. The important radiographic observation is osteopenia of the affected hip joint. On MRI, it is characterized by bone marrow edema in the head and neck. The differential diagnosis is for conditions that cause regional osteopenia or bone marrow edema, which include avascular necrosis (AVN), insufficiency fracture, idiopathic TOH, and transient edema syndrome. In a septic joint, you would expect edema to affect the acetabulum and joint effusion with synovitis. 2. D. High-resolution MRI of the hip has suggested that the underlying process causing the regional osteopenia and bone marrow edema are very subtle subchondral subarticular stress and insufficiency fractures that were previously overlooked. 3. B. The edematous marrow typically shows heterogeneous enhancement with IV contrast. 4. C. The condition is self-limiting. In most cases of TOH, the bone marrow edema in the head and neck region resolves after 6 to 10 months.

Comment Epidemiology

Figure S54-2  Coronal short tau inversion recovery image shows intense bone marrow edema (asterisk) in the femoral head and neck. A small joint effusion is present.

the presence of edema (Figures S54-2 and S54-3). Administration of gadolinium solution generally shows diffuse enhancement of the abnormal marrow. A joint effusion is common. In some patients, TOH may be a manifestation of regional migratory osteoporosis, which affects multiple consecutive joints. Scintigraphy shows uniformly increased uptake at the affected femoral head extending into the femoral neck and intertrochanteric region.

TOH is a painful hip condition that most frequently affects middle-aged men and pregnant women. Overall, it is 3 times more common in males. It is characterized by a sudden onset of severe hip pain in the absence of trauma and affects people with no risk factors for AVN. It is a self-limiting process that usually reverses within 6 months of onset with conservative therapy but may last as long as 9 or 10 months. It may represent previously undetected stress or insufficiency fractures.

AVN is a progressive condition caused by interruption of the vascular supply to the femoral head. Acutely, it may mimic the appearance of TOH, but once there is necrosis of the bone it does not revert to normal marrow.

Imaging Findings

Reference

Radiographically, the classic appearance of an affected hip is diminished bone density involving the femoral head and trabeculation of the femoral neck (Figure S54-1). However, it may be normal in the symptomatic patient for the first 2 or 3 months. MRI depicts low signal intensity on T1-weighted images and high signal intensity on fluid-sensitive images within the marrow of the femoral head and neck, indicating

Klontzas ME, Vassalou EE, Zibis AH, Bintoudi AS, Karantanas AH. MR imaging of transient osteoporosis of the hip: an update on 155 hip joints. Eur J Radiol. 2015;84:431–436.

Differential Diagnosis

Cross-Reference Musculoskeletal Imaging: The Requisites, 4th ed, 311–312.

Figure S54-1  Anteroposterior radiograph of the pelvis shows marked osteopenia involving only the left femoral head and neck region (arrow). Note the contrast with the normal contralateral hip. The adjacent acetabulum is normal in density and the joint space appears normal.

112

http://radiologyebook.com

CASE 55

Figure 55-1

HISTORY:  This 44-year-old woman had radiographs of the ankle for heel pain. 1. What are the possible considerations in your differential diagnosis? (Choose all that apply.) A. Intraosseous lipoma B. Metastasis C. Calcaneal pseudocyst D. Giant cell tumor E. Simple bone cyst 2. Why does fat appear bright in signal intensity on T1-weighted (T1W) magnetic resonance (MR) images? A. Fat shortens T1 relaxation and therefore appears bright on T1W images B. Fat shortens T2 relaxation and therefore appears bright on T1W images C. Fat lengthens T1 relaxation and therefore appears bright on T1W images D. Fat lengthens T2 relaxation and therefore appears bright on T1W images

3.  How would you classify this lesion using the Milgram classification? A. Stage 1 B. Stage 2 C. Stage 3 D. Stage 4 4.  What is the most distinctive feature of an intraosseous lipoma? A. Reactive ossification at tumoral margins B. Central nidus of dystrophic calcification C. Pathologic fracture D. Soft tissue mass

See the Supplemental Figures section for additional figures and legends for this case.

http://radiologyebook.com

113

ANSWERS CASE 55 Intraosseous Lipoma 1. A, C, D, and E. An intraosseous lipoma is the only serious consideration if one considers the presence of dystrophic calcification in the center of the lesion. However, initially, a calcaneal pseudocyst, giant cell tumor of bone, and a simple bone cyst could be in the differential diagnosis, particularly in cases that are more radiolucent. 2. A. Fat shortens T1 relaxation and therefore appears bright on T1W images. 3. B. This lesion would be considered a stage 2.

fat in the bone marrow by a low signal intensity rim (Figure S552). Fat saturation is a good technique for confirmation of the tumor histology.The Milgram classification divides intraosseous lipomas into three stages: Stage 1 lesions are sharply delineated lipomas with homogeneous fat content; stage 2 lesions contain areas of central necrosis with dystrophic calcification or ossification; and stage 3 lesions are heterogeneous with variable thickened septations, cystic cavities, and variable degrees of calcification or ossification (Figures S55-3 and S55-4). Reference Murphey MD, Carroll JF, Flemming DJ, Pope TL, Gannon FH, Kransdorf MJ. From the archives of the AFIP: benign musculoskeletal lipomatous lesions. Radiographics. 2004;24:1433–1466.

Cross-Reference

4. B. A central nidus of dystrophic calcification in a lytic geographic lesion is nearly pathognomonic of an intraosseous lipoma.

Musculoskeletal Imaging: The Requisites, 4th ed, 413–414.

Comment Pathology and Distribution An intraosseous lipoma may be detected in patients of all ages. Histologically, this neoplasm is identical to an extraosseous lipoma, composed of mature adipose cells that are separated into lobules by fibrovascular septations. Two thirds of patients present with localized pain, which varies in duration. The most common location for this tumor is in the lower extremity, with nearly 40% of cases involving the femur, tibia, and fibula. The calcaneus is the most common tarsal bone involved, accounting for 15% to 30% of all cases of intraosseous lipomas.

Radiographic Findings A typical lesion appears well marginated, radiolucent, and surrounded by a thin rim of sclerosis (Figure S55-1). Lobulations are common at the margins of the lesion. In the calcaneus, it depicts a triangular configuration, typically residing in between the major trabeculations of the bone in the lateral projection and sharing a similar location with simple cysts. A central nidus of dystrophic calcification is common, which is nearly pathognomonic for this neoplasm.

Magnetic Resonance Imaging Findings Unequivocal diagnosis is readily provided by T1W MR images. The high signal intensity of the lipoma is demarcated from the

Figure S55-1  The lateral radiograph shows a well-circumscribed triangular lucency in the calcaneus with a very fine rim of surrounding sclerosis. Note the central nidus of dystrophic calcification (arrow).

114

http://radiologyebook.com

CASE 56

Figure 56-1

Figure 56-2

HISTORY:  A 33-year-old man with polyarthralgia.

3. What percentage of patients with psoriatic arthritis develop sacroiliac joint symptoms? A. Less than 5% B. 10% to 25% C. 30% to 50% D. More than 80%

1.  What should be included in the differential diagnosis? (Choose all that apply.) A. Psoriatic arthritis B. Rheumatoid arthritis C. Erosive osteoarthritis D. Reactive arthritis 2.  What percentage of people with psoriasis develop the arthropathy? A. Less than 1% B. Between 5% and 25% C. About 50% D. More than 75%

4. What is the main difference between the spondylitis of psoriatic arthritis and that of ankylosing spondylitis? A. Sacroiliac involvement is rare B. The distribution is more superior C. Does not involve the facet joints D. Does not involve annulus fibrosus

See the Supplemental Figures section for additional figures and legends for this case.

http://radiologyebook.com

115

ANSWERS CASE 56 Psoriatic Arthropathy 1. A and D. The findings are erosive arthritic changes in the first and third toes and a spondyloarthropathy. The differential diagnosis for these two entities is psoriatic arthritis and reactive (formerly Reiter disease) arthritis. 2. B. The reported incidence is between 2% and 6%, although as many as 25% of certain groups develop the clinical and radiographic manifestations of psoriatic arthritis. 3. C. About one third to one half of patients expressing the arthropathy of psoriasis develop sacroiliac joint pain. 4. D. The paravertebral ossification does not involve the annulus fibrosus, which is the essential target site of syndesmophytes of ankylosing spondylitis.Therefore they are referred to as parasyndesmophytes.

Comment Clinical Presentation Psoriatic arthritis is a synovial inflammatory arthropathy that is considered a seronegative rheumatoid variant. It generally affects patients with moderate to severe skin disease, but its strongest correlation is in patients who demonstrate nail abnormalities such as pitting, ridging, splintering, and thickening. This arthropathy has five presentations: distal interphalangeal joint polyarthritis (Figure S56-1), arthritis mutilans, rheumatoid arthritis–like symmetric polyarthritis, monoarthritis or asymmetric oligoarthritis, and sacroiliitis and spondylitis (Figure S56-2) mimicking ankylosing spondylitis. The arthropathy may antedate skin changes in 20% of cases. Scoring indices for dactylitis and enthesitis can assess disease activity.

Figure S56-2  Anteroposterior view of the lumbar spine shows typical parasyndesmophytes (arrows) and bilateral sacroiliitis (asterisk) characteristic of the spondyloarthropathy pattern.

Imaging Findings The radiographic hallmark of this disease is bone proliferation (Figure S56-3), distinguishing it from rheumatoid arthritis (the prototypical non–bone-forming synovial inflammatory arthropathy). Periostitis and joint ankylosis are also radiographic hallmarks. The joints of the hands and feet are common target sites. Key observations include soft tissue swelling that sometimes involves the entire digit (“sausage finger”) (Figure S56-4), normal mineralization, bony erosions that begin at the margins of the joint and progress centrally, destroying the entire articular surface area (“pencil-in-cup” deformity) (Figure S56-5), and resorption of the tufts of the distal phalanges. Bony proliferative changes may occur adjacent to these erosions but may also occur in the form of periostitis, joint ankylosis (Figure S56-6), and enthesopathy. In the axial skeleton, bilateral sacroiliitis is more frequent than unilateral involvement and may be either symmetric or asymmetric. In the spine, asymmetric or unilateral paravertebral ossifications affect the lower thoracic and upper lumbar spine.

Imaging Horizons Whole-body magnetic resonance imaging has been investigated as a novel imaging modality for evaluation of enthesitis in patients with psoriatic arthritis. Ultrasound also has been suggested as a potentially useful modality for investigation in patients who are not yet symptomatic by assessing dactylitis and nail morphology. Both techniques have appeal because patients would not be exposed to radiation. Reference O’Connor PJ. Crystal deposition disease and psoriatic arthritis. Semin Musculoskelet Radiol. 2013;17:74–79.

Cross-Reference Figure S56-1  Frontal radiograph shows destructive changes in the interphalangeal (IP) joint of the great toe and the third proximal IP joints (arrows) indicative of a synovial inflammatory process.

116

Musculoskeletal Imaging: The Requisites, 4th ed, 281–285.

http://radiologyebook.com

CASE 57

Figure 57-1

Figure 57-2

HISTORY:  A 29-year-old man presents with increasing shoulder weakness.

3. All of the following describe quadrilateral space syndrome EXCEPT: A. Impingement of the axillary nerve B.  Symptoms from posterior humeral circumflex artery compression C. Affects posterior deltoid and teres minor muscles D. One of the most common causes is fibrotic bands

. What is the differential diagnosis? (Choose all that apply.) 1 A. Suprascapular notch syndrome B. Decentering syndrome C. Quadrilateral space syndrome D. Parsonage-Turner syndrome 2. All of these structures form the boundaries of the quadrilateral space EXCEPT: A. Teres minor muscle B. Teres major muscle C. Long head of the triceps muscle D. Lateral scapular border

. What is the cause of decentering syndrome? 4 A. Chronic anterior labral tear and anterior instability B. Superior labral tear and rotator cuff tear C. Posterior labral tear and posterior instability D. Chronic deltoid atrophy

See the Supplemental Figures section for additional figures and legends for this case.

http://radiologyebook.com

117

ANSWERS CASE 57 Compressive Shoulder Neuropathy (Quadrilateral Space Syndrome) 1. B, C, and D. This patient has atrophy of the posterior deltoid and teres minor muscles, so quadrilateral space syndrome is the best answer; but decentering syndrome, which affects the teres minor muscle, and Parsonage-Turner syndrome, which affects any combination of shoulder muscles, are potential considerations. 2. D. The boundaries of the quadrilateral space are the teres minor muscle superiorly, teres major muscle inferiorly, long head of the triceps brachii muscle medially, and the humerus laterally. 3. B. Symptoms are the result of compression of the axillary nerve and not from the arterial occlusion. 4. C. Chronic traction injury to the teres minor nerve from a posterior labral tear and posterior glenohumeral joint instability or subluxation are the causes of decentering syndrome.

Comment Quadrilateral Space Syndrome Compressive neuropathies are an important cause of shoulder pain and disability.The axillary nerve and posterior humeral circumflex artery course through the quadrilateral space of the shoulder. The syndrome is characterized by pain that is exacerbated by abduction and external rotation. Paresthesia in the shoulder and upper arm and weakness may be present. In the absence of trauma, the syndrome is usually caused by fibrous bands, although any space-occupying process may elicit the syndrome. Magnetic resonance imaging is the preferred method for diagnosis. The acute appearance of denervation is high signal

Figure S57-2  Sagittal T1-weighted image shows that the posterior deltoid muscle (arrow) also appears atrophied in addition to the teres minor muscle (asterisk), characteristic of quadrilateral space syndrome.

intensity on T2-weighted images in the affected muscle. The altered signal intensity reflects an increase in the extracellular water space, which can be detected as early as 2 weeks from the time of insult. Atrophy of the teres minor and posterior deltoid muscles are late findings (Figures S57-1 and S57-2).

Suprascapular Notch Syndrome The suprascapular nerve is a mixed motor and sensory nerve that contains pain fibers to the shoulder and provides the motor supply to the supraspinatus and infraspinatus muscles of the rotator cuff. When affected, it is referred to as suprascapular notch syndrome. Some common causes of this syndrome include a ganglion, perilabral cyst, and tumors. The location of insult determines which muscles are affected. A lesion within the suprascapular notch affects innervation to both the supraspinatus and infraspinatus muscles (Figure S57-3), whereas a lesion below the spinoglenoid notch affects only the infraspinatus muscle innervation (Figure S57-4).

Differential Diagnosis Decentering syndrome is a cause of isolated teres minor atrophy and occurs in elderly men who have other pathologies such as rotator cuff tears and joint instability (Figure S57-5).The cause of atrophy is likely related to posterior decentering of the humeral head, which produces traction of the teres minor nerve. Parsonage-Turner syndrome, or brachial neuritis, affects any combination of shoulder muscles, and the key is involvement of two nerve distributions (Figure S57-6). Reference Martinoli C, Gandolfo N, Perez MM, et al. Brachial plexus and nerves about the shoulder. Semin Musculoskeletal Radiol. 2010;14:523–546. Figure S57-1  Coronal T1-weighted magnetic resonance image of the posterior shoulder shows atrophy of the teres minor muscle (asterisk).

118

Cross-Reference Musculoskeletal Imaging: The Requisites, 4th ed, 71, 81-82.

http://radiologyebook.com

CASE 58

Figure 58-1

Figure 58-2

HISTORY:  A 60-year-old man after surgery. First radiograph is postoperative, and the second is 6 weeks later.

3. After hardware loosening, what is the second most common cause of revision arthroplasty? A. Dislocation B. Hardware fracture C. Particle disease D. Infection

1. What is the differential diagnosis for the second radiograph? (Choose all that apply.) A. Mechanical wear B. Axial creep or molding C. Polyethylene fracture D. Dislocation E. Fracture of acetabular implant 2. Regarding total hip arthroplasties, all of the following are indicators of mechanical wear EXCEPT: A. Loose particles B. Asymmetric joint space C. Typical stress shielding D. Tilted locking ring

4. Regarding total hip arthroplasty, what is the current incidence of infections requiring revision arthroplasty? A. Less than 5% B. 10% C. 20% D. More than 25%

See the Supplemental Figures section for additional figures and legends for this case.

http://radiologyebook.com

119

ANSWERS CASE 58 Hip Surgery Complications 1. C. Given the time frame, the important considerations would include fracture of the polyethylene component and a break in the ceramic liner. However, if only the second radiograph is available, then mechanical wear and axial molding would also be viable considerations. 2. C. Typical stress shielding or disuse osteopenia is usually focal and involves the proximal femur. It occurs because of redistribution of mechanical forces due to the placement of the prosthesis and is an expected finding in about 25% of total hip replacements, especially those with larger stems. 3. A. Dislocation is the second most common reason (after implant loosening) for revision arthroplasty. 4. A. Improved sterility, operative technique, and patient care have resulted in a dramatic decrease of infections requiring revision of a hip arthroplasty, reportedly occurring in less than 5% of hip replacements.

Comment Polyethylene Fracture and Dissociation In the perioperative period, an asymmetric position of the head of the femoral implant within the acetabular cup usually indicates a problem with the polyethylene liner within the cup, although rarely it may be caused by a break in the acetabular ceramic backing.The polyethylene liner may dissociate from the cup presenting as a tilted or broken locking ring, or it may fracture causing a shift in the position of the femoral head, but this can be subtle (Figures S58-1 to S58-3). Increased pain is an important symptom.

Figure S58-2  Follow-up radiograph 6 weeks later shows that the superior joint space is narrowed (arrow) and the head nearly abuts the acetabular cup.

Mechanical Wear Friction can cause accelerated loss of the polyethylene material, releasing particles into the joint.The most common type of wear occurs from the femoral prosthetic head against the acetabular liner. There are two types of wear. Fatigue wear occurs from repetitive stressing of the joint (Figure S58-4). Interfacial wear is further divided into abrasive (surface asperity cuts into the opposing surface) and adhesive (particles from tears in the softer surface adhere to the other surface) wear.

Other Considerations Fortunately, postoperative complications are not common, though as many as 3% to 10% of complications can be sufficiently significant to require additional or repeat surgery. Infection, nerve injury, heterotopic ossification, and dislocation are notable complications. Hardware failure can occur in the perioperative period, and the radiographic manifestation depends on the area of failure (Figures S58-5 and S58-6). Other complications to look for include particle disease or loosening of the implants, depicted by a zone of lucency surrounding the implant that exceeds 2 mm in width.When the implant changes in position, the diagnosis is unequivocal. Incorrect placement of a femoral implant may result in a fracture of the femoral cortex. References Brooks PJ. Dislocation following total hip replacement: causes and cures. Bone Joint J. 2013;95-B(suppl 11A):67–69. Popescu D, Gallart X, Garcia S, Bori G, Tomas X, Riba J. Fracture of a ceramic liner in a total hip arthroplasty with a sandwich cup. Arch Orthop Trauma Surg. 2008;128:783–785. Figure S58-1  Postoperative radiograph of a total hip arthroplasty shows that the head of the femoral implant (asterisk) is located centrally within the acetabular cup (arrows).

120

Cross-Reference Musculoskeletal Imaging: The Requisites, 4th ed, 318–322.

http://radiologyebook.com

CASE 59

Figure 59-1

Figure 59-2

HISTORY:  A 15-year-old presents with a palpable lump in his hand.

3. On magnetic resonance (MR) imaging, what observation would favor a malignancy? A. Soft tissue mass B. Heterogeneous signal intensity C. Enhancement D. Peritumoral edema

1.  What should be included in the differential diagnosis? (Choose all that apply.) A. Periosteal chondroma B. Osteoid osteoma C. Periosteal chondrosarcoma D. Parosteal osteosarcoma E. Langerhans histiocytosis 2.  What feature suggests benignity chondrosarcoma? A. Size B. Periosteal reaction C. Calcification D. Lack of soft tissue mass

over

a

periosteal

4. All of the following differentiate bizarre parosteal osteochondromatous proliferation (BPOP) from periosteal chondroma EXCEPT: A. Exophytic, pedunculated mass B. Affects hands and feet C. Wide base D. Calcified matrix

See the Supplemental Figures section for additional figures and legends for this case.

http://radiologyebook.com

121

ANSWERS CASE 59 Periosteal (Juxtacortical) Chondroma 1. A, C, and E. The mass in the fourth proximal phalanx has produced saucerization of the cortex and matrix calcification within the lesion. Biopsy confirmed a periosteal chondroma. Periosteal chondrosarcoma and eosinophilic granuloma are to be considered because both can present with cortical scalloping. 2. A. The relatively small size (5 cm

Deep

High

No

IV

Any

Any

Any

Yes

The upper extremity and retroperitoneum are the next most common locations affected. Lesions in the retroperitoneum can become quite large before detection. Most patients are between 50 and 70 years old, although any age may be affected (rare in people younger than 20 years).

Imaging Features Radiographs may show a soft tissue mass or a destructive lesion if in bone. On computed tomography (CT), tumors show an attenuation value similar to muscle. MRI is the preferred imaging modality for assessing the location, size, and proximity to neurovascular structures (Figures S197-1 to S197-6). Typically lesions are large (5 to 20 cm) and well circumscribed, and demonstrate areas of necrosis. On T1, they usually are intermediate- to low-intensity masses with heterogeneously increased T2 signal intensity. Staging is based on size, depth, grade of lesion, and presence of metastasis. Reference Walker EA, Salesky JS, Fenton ME, Murphey MD. Magnetic resonance imaging of malignant soft tissue neoplasms in the adult. Radiol Clin North Am. 2011;49:1219–1234.

Cross-Reference Musculoskeletal Imaging: The Requisites, 4th ed, 409–411. Figure S197-1  Coronal T1-weighted image shows a large mass in the anterior thigh with signal intensities ranging from low signal intensity to high signal intensity (arrows).

400

CASE 198

Figure 198-1

HISTORY:  A newborn with deformities of the upper extremities. 1.  What should be included in the differential diagnosis? (Choose all that apply.) A. Radial hand B. Thrombocytopenia–absent radius (TAR) syndrome C. Fanconi anemia D. Holt-Oram syndrome E.  Vertebral, anal, cardiac, tracheoesophageal, renal, and limb (VACTERL) syndrome 2. Embryologically, what gives rise to the osseous structures in the upper extremity? A. External ectoderm B. Somatic mesoderm C. Lateral plate mesoderm D. Endoderm

3. What percentage of patients with radial longitudinal deficiency (RLD) have bilateral disease? A. 90% B. 60% C. 25% D. 5% 4. The abnormalities in VACTERL syndrome are derived from what embryologic germ layer? A. Endoderm B. Ectoderm C. Mesoderm D. Neural crest

See the Supplemental Figures section for additional figures and legends for this case.

401

ANSWERS or ventricular septal defects); VACTERL syndrome; and Fanconi anemia (pancytopenia).

CASE 198 Radial Longitudinal Deficiency

Imaging

1. A, B, C, and E. The main observations are absence of the radius and maldeveloped thumb. The differential includes all in this list, but Holt-Oram syndrome is associated with a triphalangeal thumb in contradistinction to the other syndromes, which have poorly developed thumbs.

Radiographically, the manifestations of RLD may vary from thumb hypoplasia with an intact radius to complete absence of the radius (Figure S198-1 and S198-2). However, thumb hypoplasia may occur independently as well. The ulna bows toward the radial side and is usually thickened. The thumb may be hypoplastic or absent. As the skeleton develops, carpal fusion or absence may become evident, and the humerus length may be shortened or deformed.

2. C. The cells from the lateral plate mesoderm form cartilage and bones during limb bud development between 4 and 8 weeks after fertilization. 3. B. About 60% of patients have bilateral involvement. 4. C. VACTERL syndrome refers to abnormalities in structures derived from the embryonic mesoderm.

Bauer AS, Bednar MS, James MA. Disruption of the radial/ulnar axis: congenital longitudinal deficiencies. J Hand Surg Am. 2013;38: 2293–2302.

Cross-Reference

Comment

Musculoskeletal Imaging: The Requisites, 4th ed, 551, 553.

Embryologic Overview Congenital hand anomalies are relatively rare, but important, because the 1-year mortality associated with live-born infants with congenital upper limb deformities is about 15%. Embryogenesis occurs between 4 and 8 weeks after fertilization, the time most anomalies form. The cartilage and bone arise from the lateral plate mesoderm during the limb bud development phase.

Clinical Overview The incidence of RLD, or radial clubhand, is about 0.4 to 1 per 10,000 live births. It may occur through both autosomal dominant and recessive genetic transmission. The congenital deformity is characterized by hypoplasia or aplasia of the radius. The deficiency usually involves the thumb. The wrist deviates markedly toward the radial side, owing to the absence of the normal buttress afforded by the radius. The disorder is bilateral in 60% of cases. The most frequently reported syndromes associated with RLD are TAR syndrome; Holt-Oram syndrome (atrial

L

R

Figure S198-1  Frontal radiographs of both arms of a newborn show absence of the radii (arrows) and deformation of the thumbs.

402

Reference

CASE 199

Figure 199-1

Figure 199-2

HISTORY:  A 58-year-old dialysis patient presents with chronic knee and wrist pain.

. All of the following characterize an amyloidoma EXCEPT: 3 A. Massive destruction of bone B. May be AL-type or AA-type C. Can occur purely in the soft tissues D. Potential for metastasis

1.  What could be included in the differential diagnosis? (Choose all that apply.) A. Amyloid arthropathy B. Gouty arthritis C. Pyrophosphate arthropathy D. Rheumatoid arthritis E. Erosive osteoarthritis 2. Regarding amyloid arthropathy, what is the substance commonly deposited in the bone and joints? A.  β2-Microglobulin B. Urate crystals C. Calcium pyrophosphate D. Fibrous connective tissue

4. In the United States, secondary or reactive amyloidosis is associated most commonly with what arthritis? A. Systemic lupus erythematosus (SLE) B. Ankylosing spondylitis C. Rheumatoid arthritis D. Behçet syndrome

See the Supplemental Figures section for additional figures and legends for this case.

403

ANSWERS CASE 199 Amyloid Arthropathy 1. A, B, and D.  The findings are nonspecific but erosive changes with relatively mild joint narrowing in an atypical distribution, making amyloid arthropathy the primary consideration, particularly if soft tissue masses are observed. Some erosion appears to result in overhanging edges, so gout would need to be excluded in this patient. 2. A. β2-Microglobulin is the most common protein deposited. 3. D. Amyloidomas are benign accumulations of amyloid, and even when they contain plasma cells, it is due to benign clonal proliferation of the plasma cells. Because they are not malignant, there is no potential for metastasis. 4. C. In the United States, secondary amyloidosis is associated with rheumatoid arthritis (RA) in 75% of cases, although juvenile chronic arthritis, SLE, Behçet syndrome, and ankylosing spondylitis are also commonly listed associated diseases.

Comment Clinical Information Amyloidosis refers to conditions that are characterized by the deposition of amyloid in the tissues. There are a large number of unrelated proteins that comprise the amyloid fibril, including immunoglobulin light chains, calcitonin, and β2-microglobulin. The deposition of amyloid can lead to renal failure, pericardial and myocardial disease, organomegaly, pulmonary disease, and gastrointestinal disease.

Amyloid Arthropathy Approximately 5% to 15% of patients have articular or osseous involvement with deposition of amyloid occurring in the bone, synovium, and periarticular soft tissues, and this can involve the axial or appendicular skeleton. It is most prevalent in renal patients who are undergoing long-term hemodialysis, because the amyloid proteins are not filtered by the standard dialysis membranes. The articular changes of

Figure S199-2  The wrist radiograph shows cystic changes in the proximal carpal row, and distal radius and ulna. Note the soft tissue mass causing an erosion in the ulna (arrow).

amyloid arthropathy are bilateral and are most frequent in the shoulders, hips, wrists, and knees. Soft tissue nodules from accumulated amyloid may occur around the elbow, hand, and wrist, resembling those of rheumatoid arthritis. About 10% to 30% of patients on long-standing dialysis have carpal tunnel syndrome.

Imaging The radiographic findings in amyloid arthropathy usually include asymmetric soft tissue masses, periarticular osteoporosis, preservation of the joint spaces, subchondral cyst formation, effusions, and/or erosions (Figures S199-1 to S199-4). The symmetric joint involvement and erosions are similar to RA but the lack of significant joint narrowing is distinctive. Subchondral cyst formation and lytic bone lesions are more common in the wrists and larger joints such as the shoulder and hip joints. Destructive spondyloarthropathy mimics bacterial diskitis and is characterized by end-plate erosions, disk space narrowing, and absence of osteophytes. Erosions are most commonly seen in the anterior aspect of the disk and in the facet joints. On magnetic resonance imaging, amyloid deposits demonstrate low to intermediate signal intensity on both T1- and T2-weighted sequences (Figures S199-5 and S199-6). Reference Sheldon PJ, Forrester DM. Imaging of amyloid arthropathy. Semin Musculoskelet Radiol. 2003;7:195–203.

Cross-Reference Musculoskeletal Imaging: The Requisites, 4th ed, 302–304.

Figure S199-1  Frontal knee radiograph shows marginal erosions (arrows) and central subchondral cysts.

404

CASE 200

Figure 200-1

Figure 200-2

HISTORY:  A 22-year-old man presents with a tender mass.

3. What percentage of Nora lesions recur after excision? A. None B. Less than 5% C. 25% D. 50%

1.  What could be included in the differential diagnosis? (Choose all that apply.) A. Florid reactive periostitis B. Turret exostosis C. Nora lesion D. Osteochondroma E. Parosteal osteosarcoma 2. Regarding bizarre parosteal osteochondromatous proliferation (BPOP), what is the risk for malignant transformation? A. 0% B. 10% C. 25% D. 50%

4. What is the appearance of BPOP on a technetium-99m bone scan? A. Decreased uptake B. No uptake C. Abnormal uptake D. Target-like uptake

See the Supplemental Figures section for additional figures and legends for this case.

405

ANSWERS CASE 200 Nora Lesion 1. A, B, and C. The diagnosis in this patient is BPOP. Turret exostosis is a posttraumatic condition characterized by a bony, dome-shaped mass arising from the dorsal periosteum. Florid reactive periostitis can mimic a mass occasionally.The phalanx is an unusual location for both osteochondroma and parosteal osteosarcoma. 2. A.This is a benign lesion without malignant potential. 3. D. BPOP may show marked local invasion, and about one half recur after surgical excision. 4. C.  Abnormally increased uptake is typical of the lesion.

Comment Clinical Information Nora lesion, or BPOP, is an uncommon lesion characterized by the exophytic outgrowth from the cortex consisting of bone, cartilage, and fibrous tissue. It usually involves the hands and feet (4 times more common in the hands) but has been described in the skull, long bones, maxilla, and sesamoid bones. The most common location is in the proximal and middle phalanges, and the metacarpal and metatarsal bones. Most patients are young adults presenting with a tender mass or an incidental mass after trauma. The treatment is simple excision, although about one half do recur.

Pathology Histologically, BPOP is a well-circumscribed mass with a cartilage cap and bone tissue. The superficial area shows fibrocartilaginous tissue with high cellularity and spindle-shaped chondrocytes scattered within a myxoid stroma. The appearance of the bony trabeculation is immature with high osteoblastic activity. The absence of cellular atypia helps to distinguish this lesion from osteosarcoma, but it may be difficult to discriminate it from an osteochondroma, which is rare in the small tubular bones.

Figure S200-2  Lateral view shows that the mass arises from the cortex (arrow).

Imaging On radiographs, BPOP appears similar to an osteochondroma, arising from the surface of the bone. Most do not show corticomedullary continuity, helping to distinguish these lesions from osteochondromas, but occasionally it may occur (Figures S200-1 to S200-4). Magnetic resonance imaging is useful in distinguishing BPOP from malignant lesions by demonstrating preservation of the cortical bone under the cartilaginous cap and homogeneous enhancement. On T1-weighted images, the lesion is low in signal intensity, and is high in signal intensity on T2-weighted and short tau inversion recovery sequences. No soft tissue mass or peritumoral infiltration is evident with BPOP. Reference Berber O, Dawson-Bowling S, Jalgaonkar A, et al. Bizarre parosteal osteochondromatous proliferation of bone: clinical management of a series of 22 cases. J Bone Joint Surg Br. 2011;93:1118–1121.

Cross-Reference Musculoskeletal Imaging: The Requisites, 4th ed, 386–389.

Figure S200-1  Frontal radiograph of the hand shows a focal area of bone proliferation in the proximal third finger (arrow).

406

Supplemental Figures

CASE 1

Figure S1-4  Axial T2-weighted image shows a fluid-fluid level from the layering of blood products within the cyst (blue arrow). The periostitis (red arrows) is related to the pathologic fracture (yellow arrow).

Figure S1-3  Radiograph of an older patient shows a larger lesion more distally located with a complete fracture (arrows) and a “fallen fragment” sign.

Figure S1-5  Observation of the “fallen fragment” sign (arrow) will make even atypical cases relatively easy to recognize.

408

CASE 2

Figure S2-3  Closer inspection of each radiograph shows that there are thinner dense metaphyseal bands in the metacarpals and phalanges.

CASE 3

Figure S3-4  Sagittal T2-weighted image shows an osteochondral defect in the anterior surface of the capitellum (arrow).

Figure S3-3  Another patient shows a large effusion (anterior “sail” sign) and a small ossific fragment of bone in the coronoid fossa (arrow).

409

CASE 4

Figure S4-3  Multiplanar sagittal computed tomographic image shows a typical posterior dislocation with the proximal tibia perched against the back edge of the condyle (arrow).

Figure S4-4  Three-dimensional volume-rendered computed tomographic image depicts that the dislocation is associated with numerous fracture fragments (arrows), which would be important during reduction.

Figure S4-5  This patient had a combination of a medial and anterior dislocation.

410

Figure S4-6  In addition to numerous ligamentous tears, patients usually depict extensive bone and soft tissue injuries such as bone contusions (red arrow) and fractures, muscle strains (yellow arrow), meniscal tears, and capsular rupture.

CASE 5

Figure S5-3  Close-up view of a patient with Hadju-Cheney disease shows the characteristic bandlike pattern of acro-osteolysis.

Figure S5-4  Close-up view of a patient with severe multicentric reticulohistiocytosis shows a periarticular pattern of acro-osteolysis.

CASE 6

Figure S6-2  Diffuse joint space narrowing (arrows) results in axial migration of the femoral head and ring osteophytes.

Figure S6-3  Corner osteitis (arrows) is manifested by sclerosis, eventually producing squaring of the vertebral bodies.

411

Figure S6-4  Syndesmophytes at several levels (arrows) result in a “bamboo” spine with undulating fusion of the vertebral bodies and apophyseal joints. Note “tram track sign” with fusion of the spinous processes (red arrow).

Figure S6-5  Osteoporosis late in the disease makes the spine vulnerable to fractures after minor trauma, which may lead to the development of pseudoarthrosis (arrow).

CASE 7

Figure S7-2  Patient with metastatic breast cancer shows vertebra plana of the T8 vertebra (arrow). Figure S7-3  Sagittal reformatted computed tomographic image of the prior patient shows complete collapse of the T8 vertebral body, retropulsion into the canal, and increased density of the bone.

412

Figure S7-4  Kümmell disease of the lumbar spine with marked collapse of the L1 vertebra and a linear collection of gas within the vertebral body (arrow). Figure S7-5  Sagittal reformatted computed tomographic image shows collapse of the L1 vertebral body with marked loss of height centrally and confirms the gas within the vertebral body (arrow).

CASE 8

Figure S8-3  Internally rotated view of the shoulder in another patient demonstrates two foci of calcifications; the smaller deposit is located at the infraspinatus tendon insertion (yellow arrow) and the larger focus of calcium (red arrow) is in the bursa.

Figure S8-4  Sagittal T2-weighted image shows that the calcification has intermediate signal intensity and little surrounding inflammation.

413

Figure S8-5  Axial T2-weighted image in another patient shows associated interstitial edema in the subscapularis tendon surrounding the calcific deposit (arrow). Figure S8-6  Sagittal T2-weighted image shows a globular focus of low signal intensity (arrow) with surrounding edematous tissue and fluid in the subacromial/subdeltoid bursa indicating bursitis.

CASE 9

Figure S9-3  Cortical breakthrough is common and is associated with a large soft tissue mass with osteoid matrix (arrows).

414

Figure S9-4  Coronal T2-weighted image shows areas of low signal intensity (arrows) representing osteoid matrix-rich areas.

Figure S9-5  Ninety percent of osteosarcomas produce osteoid matrix (arrow), but only 50% are sufficient to be termed osteoblastic. The matrix can be characterized in the soft tissue mass that breaks through the cortex (asterisk).

Figure S9-6  The telangiectatic type of osteosarcoma is purely lytic and can mimic an aneurysmal bone cyst. This patient had a pathologic fracture of the lateral cortex (arrow).

CASE 10

Figure S10-2  Sagittal reformatted computed tomographic image shows that there is ossification of the anterior longitudinal ligament from diffuse idiopathic skeletal hyperostosis (yellow arrows) in addition to ossification of the posterior longitudinal ligament (red arrows).

Figure S10-3  T2-weighted image in another patient shows marked narrowing of the spinal canal caliber from C2 to C6 as a result of ossification of the posterior longitudinal ligament, and signal intensity changes in the cord from myelopathy (arrows).

415

CASE 11

Figure S11-3  Undertubulation has resulted in an Erlenmeyer deformity with widening of the metaphysis (arrows). There is loss of the normal medullary-cortical demarcation.

Figure S11-4  Note that there are areas in the end plate that show normal bone density (arrows). This is referred to as a “sandwich” vertebra.

CASE 12

Figure S12-3  This patient has been followed for 4 weeks and is beginning to show increased density in the proximal pole indicative of ischemia as well as sclerotic changes at the fracture line (arrow).

416

Figure S12-4  Another patient shows fracture displacement with ulnar deviation (arrow).

Figure S12-5  T1-weighted image shows an occult fracture of the scaphoid (arrow) and bone contusions in the proximal capitate and mid-hamate (asterisks).

CASE 13

Figure S13-3  There is a pathologic fracture of the lateral cortex of the radius in the proximal aspect of the lesion (arrow) with overlying soft tissue swelling.

Figure S13-4  Axial T2-weighted image demonstrates a soft tissue mass protruding from the medullary cavity of the knee (asterisk). A fluid-fluid level is not an uncommon finding (arrow).

Figure S13-5  Lateral view of calcaneus shows tumor recurrence manifested as an enlarging zone of lucency (arrows) surrounding the methyl methacrylate cement in the area of prior resection.

417

CASE 14

Figure S14-2  After an avulsion by the hamstring insertion, the bone overgrowth of the ossification center (arrows) can mimic a neoplastic process. The key to the diagnosis is location and a history of trauma.

Figure S14-3  This radiograph shows an avulsion of the anterior inferior iliac spine (arrow). Occasionally, this can be a very subtle abnormality with a thin sliver of bone avulsed from the pelvis by a traction injury of the rectus femoris muscle.

Figure S14-4  Pain caused by avulsion of the anterior superior iliac spine (arrow) by the sartorius muscle. This injury often heals with significant bone overgrowth due to inferior displacement. Figure S14-5  Greater trochanteric avulsion fracture (arrow) occurs in both young and old people but the mechanism is similar, a sudden traction by the gluteal medius/minimus tendon attachments.

418

CASE 15

Figure S15-3  Lateral radiograph shows a distended olecranon bursa with relatively well-defined margins (arrows) differentiating it from soft tissue swelling.

Figure S15-4  Axial magnetic resonance image on the same patient shows the fluid in the bursa (asterisk) and thickened walls but no surrounding inflammation consistent with simple bursitis. The bone marrow in the olecranon is normal.

CASE 16

Figure S16-3  Forearm radiograph shows shortening of the ulna from a large sessile osteochondroma (arrow).

Figure S16-4  Hip radiograph shows erosive changes in the femoral head from repeated trauma against an osteochondroma arising from the acetabulum (arrow). Numerous other lesions are evident in the femur.

419

Figure S16-5  Axial T2-weighted image shows development of a pseudobursa filled with fluid (arrow) over a sessile osteochondroma at a point of friction.

Figure S16-6  Large pedunculated osteochondroma with a characteristic cauliflower appearance (arrows). The margins should be closely evaluated for changes that could indicate malignant transformation.

CASE 17

Figure S17-4  Squared iliac bones (asterisks) and a narrow horizontal sacrum result in small and deep greater sciatic notches. Horizontal acetabula morphology (arrows) causes flattening of the acetabular angles. This constellation of findings has been referred to as a “champagne glass” appearance.

Figure S17-3  Progressive narrowing of the interpediculate distance and shortening of the pedicles contribute to significant spinal stenosis (arrows). The lumbosacral lordosis is exaggerated, resulting in a horizontal sacrum.

420

CASE 18

Figure S18-3  Anteroposterior views of the left lower femur show thickening of the medial cortex of the femoral diaphysis associated with well-formed “flowing or dripping candle wax” periosteal new bone (arrows). This condition is distinctive because it usually affects one limb or both limbs on one side of the body.

Figure S18-4  Coronal T1-weighted image shows that the periosteal new bone is homogeneously low in signal intensity, indicating that it is quite dense (arrows).

CASE 19

Figure S19-3  Pelvic radiograph shows asymmetry of the proximal femoral metaphysis with areas of bone rarefaction on the left side (arrow) and a more generalized loss of bone density in the left side of the pelvis and hip. Figure S19-4  Hip radiograph shows avascular necrosis of the left femoral head resulting in deformation of the epiphysis and increased density (arrow).

421

CASE 20

Figure S20-4  Inversion recovery magnetic resonance image shows a chondroblastoma (arrow) with multicompartmentalization and a fluid-fluid level. Hemorrhagic cysts may occur in 10% of cases, simulating those of aneurysmal bone cysts.

Figure S20-3  Peritumoral edema (asterisk) and periostitis are common MR findings on fluid-sensitive sequences, giving the lesion a more aggressive appearance. Joint effusions are very common (arrow).

CASE 21

Figure S21-3  Erosions occur in the margins of the joints, particularly in the metacarpal heads (arrows). Proximal interphalangeal joint involvement is relatively common, but distal interphalangeal joints are usually spared until the disease is more diffuse.

422

Figure S21-4  Frontal radiograph of the elbow shows marked erosive changes in the articular surfaces that deform the capitellum, trochlea, and coronoid process of the ulna (arrows). Note the relative absence of reactive bone formation that is characteristic of rheumatoid arthritis.

Figure S21-5  Joint effusion elevates the anterior and posterior fat pads (arrows). The trochlear groove morphology is altered by diffuse erosions. Rheumatoid arthritis is common in the elbow joint.

Figure S21-6  Concentric narrowing of the hip joint results in a protrusion deformity where the medial femoral head cortex (yellow arrow) lies medial to the ilioischial line (red arrows).

CASE 22

Figure S22-2  Levine-Edwards type 1 fracture (arrow) with no subluxation of C2 on C3. Note the widened spinal canal and prevertebral soft tissue swelling (asterisk), common features of this injury.

Figure S22-3  Levine-Edwards type 3 fracture with significant ­anterolisthesis of C2 on C3 and a locked facet (arrow).

423

Figure S22-5  Sagittal reformatted computed tomographic image shows asymmetric narrowing of the C2-C3 disk space, anterior translation and angulation of the C2 body, and a posteroinferior body fracture of C2 (arrow) resulting in an insufficient posterior longitudinal ligament.

Figure S22-4  Anterior C2-C3 fusion with an anterior plate and body screws with a disk spacer (arrow) to reconstitute the disk height.

CASE 23

Figure S23-3  Frontal radiograph in a different patient shows an ulnar minus variance (yellow arrow) and a sclerotic lunate that is undergoing subchondral collapse (red arrows).

424

Figure S23-4  Another patient with more pronounced lunate collapse (arrow) and abnormal rotation of the scaphoid (asterisk).

CASE 24

Figure S24-3  Different patient with Staphylococcus septic arthritis shows periarticular osteopenia as well as deformation and loss of the cortical surfaces of both the acetabulum and the femoral head from bone lysis (arrow).

Figure S24-4  Bone scintigraphy image shows prominent uptake of the radionuclide (arrow) indicative of the active bone turnover where the joint is being destroyed by the infection.

CASE 25

Figure S25-3  Corresponding coronal magnetic resonance arthrogram image shows that the full-thickness cuff tear (asterisk) allows communication between the gadolinium contrast in the joint and the subacromial/subdeltoid bursa (arrow).

425

CASE 26

Figure S26-3  A different patient with bilateral symmetric sacroiliitis (arrows) related to ulcerative colitis.

Figure S26-5  Fat-saturated T1-weighted postgadolinium images show enhancement of the edematous marrow (asterisk) surrounding the erosions.

426

Figure S26-4  T2-weighted image shows bone marrow edema on the iliac side of both joints (asterisks).

CASE 27

Figure S27-3  Patient with diffuse prostate metastases shows diffusely sclerotic lesions in both innominate bones, but notice that there are contiguous areas of normal marrow (yellow arrows). Although there is complete loss of the corticomedullary junction in areas that are infiltrated, the cortex does not appear thickened (red arrows).

Figure S27-5  Another patient with Paget disease of the tibia shows an incomplete fracture (arrow) through the anterior part of the diaphysis. Note that the posterior cortex is intact and the bone is bowed due to softening.

Figure S27-4  Lateral skull shows classic “cotton wool” appearance. Note the marked thickening of the diploic space (arrows).

Figure S27-6  Bone scintigraphy of a patient with polyostotic Paget disease shows areas of increased radionuclide uptake in the right scapula, right ischium, and left proximal femur (arrows).

427

CASE 28

Figure S28-3  Three-dimensional volume-rendered computed tomographic image shows an actively dislocated shoulder with the humeral head perched against the anterior glenoid (arrow) characteristic of a subcoracoid-type anterior glenohumeral dislocation.

Figure S28-4  Axial computed tomographic image shows that the linear defect seen on the internal rotation view as the Hill-Sachs lesion represents the posteromedial aspect of the wedge-shaped impaction fracture (arrows).

Figure S28-6  The size of the fracture fragment is important. If it affects more than 25% of the glenoid fossa, it will likely require a Latarjet-Bristow procedure. If it affects less than 25%, then a Bankart procedure most likely will be sufficient.

Figure S28-5  This patient shows typical findings of an osseous Bankart lesion, with displaced fragments from the anterior glenoid rim (arrows).

428

CASE 29

Figure S29-3  Anteroposterior view of the sacroiliac joints in the same patient shows loss of the cortical margins (arrows) on both sides of each joint that is characteristic of the subchondral type of bone resorption. This is the same type of resorption that occurs in the distal clavicle. Figure S29-4  Close-up view of the hand shows classic subperiosteal bone resorption in the radial aspect of the middle phalanges (arrows).

Figure S29-6  Brown tumor in the fibula weakened the bone, and this patient presented with a pathologic fracture (arrows).

Figure S29-5  Lateral view of the lumbar spine shows a typical “rugger jersey” spine with increased sclerosis adjacent to the vertebral body end plates (arrows).

429

CASE 30

Figure S30-3  Coronal T1-weighted image shows the area of avascular necrosis (arrows) surrounded by an irregular band of low signal intensity.

Figure S30-4  Coronal T2-weighted image of a shoulder with avascular necrosis shows a “double line” sign in the periphery of the lesion. This line separates the viable and dying bone marrow. Note the subchondral fracture (arrows), which appears as a thin arcuate region of high signal intensity beneath the cortex because fluid fills in the cleft.

CASE 31

Figure S31-3  Diagram shows a T-shaped configuration of a Rolando fracture (arrow). Another common configuration is a Yshaped fracture.

430

Figure S31-4  Another patient shows marked lateral subluxation of the shaft fragment caused by the pull of the abductor pollicis longus tendon (arrow) resulting in widening of the intraarticular gap (asterisk).

CASE 32

Figure S32-3  Occasionally, a rim of sclerosis may be evident (arrows) surrounding areas of fibrous dysplasia.

Figure S32-5  T1-weighted image shows isointense tissue (asterisk) with scattered areas of lower signal intensity (arrows).

Figure S32-4  Right humerus radiograph shows heterogeneous areas of sclerosis in the proximal humerus, thickened cortical surfaces, and lytic areas filled with ground-glass density (arrows).

Figure S32-6  Fluid-sensitive magnetic resonance image shows heterogeneous areas of increased signal intensity (asterisk).

431

CASE 33

Figure S33-2  Many patients present with an externally rotated femur, which obscures the fracture of the femoral neck (asterisk).

Figure S33-3  Garden II fractures are difficult to visualize if the film quality is poor. There is a nondisplaced complete transcervical fracture of the femoral neck (arrows).

Figure S33-4  This patient has a Garden III fracture (arrows) with displacement of the lateral cortex and varus angulation. Figure S33-5  Complete displacement of the femoral neck fracture defines a Garden IV fracture (arrows).

432

CASE 34

Figure S34-3  Anteroposterior radiograph of the lumbar spine in a different patient with carcinoid shows a relatively dense L2 vertebral body but no other abnormality. Note the dense pedicle sign bilaterally (arrows).

Figure S34-4  This patient had Paget disease at L3 (arrow). The cortex is thickened, the trabeculation is coarse, and a picture frame appearance is evident. There is subtle enlargement as well.

CASE 35

Figure S35-3  Patient with a Colles fracture shows comminution of the dorsal cortex (arrow) but only minimal dorsal impaction. The articular surface is at neutral tilt.

Figure S35-4  Patient with a Frykman type 8 injury. Fracture lines in the radius extend to both radiocarpal and distal radioulnar joints (yellow arrows), and there is a fracture of the ulnar styloid (red arrow).

433

CASE 36

Figure S36-3  Coronal T2-weighed image of a type 1 acromioclavicular (AC) joint separation shows markedly edematous superior AC ligament (arrow) and surrounding soft tissue swelling but no clavicular displacement.

Figure S36-4  Patient with a type 3 acromioclavicular (AC) joint shows stripping of the superior AC ligament (yellow arrow), hemorrhagic fluid in the joint (asterisk), and edema in the trapezius muscle (red arrow).

Figure S36-5  Sagittal image in a patient with a type 3 acromioclavicular joint separation shows complete disruption of the conoid and trapezoid components of the coracoclavicular ligament (arrow) from the coracoid process (asterisk). Figure S36-6  Axial radiograph shows a posteriorly dislocated clavicle (arrow). This patient has a small dimple in the anterior aspect of the shoulder that provided a clue for the diagnosis.

434

CASE 37

Figure S37-3  Sagittal fluid-sensitive image of a grade 1 lesion shows subchondral edema (arrow) but no osseous or cartilaginous demarcation.

Figure S37-5  This knee shows a concave defect demarcated by some sclerosis and cystic changes at the margins of the osteochondritis dissecans (arrows). The fragment is osteopenic and detached (red arrow).

Figure S37-4  Sagittal T2-weighted image in a patient with a grade 3 lesion shows demarcation of the osteochondral fragment by joint fluid surrounding most of the fragment but sparing the anterior margin (arrow).

Figure S37-6  Axial computed tomographic image shows a concave defect in the posterolateral aspect of the medial femoral condyle (arrow). The fragment was loose in the joint.

435

CASE 38

Figure S38-3  Coronal short tau inversion recovery image shows that acute infarcts may have an aggressive appearance with periostitis (arrows) and intense bone marrow edema surrounding the infarcting bone (asterisk).

Figure S38-5  Mature asymptomatic infarct (asterisk) in the left humerus.

436

Figure S38-4  T2-weighted image shows mature infarcts in the medullary cavity of the femur and tibia (asterisk) and a focal area of avascular necrosis in the subchondral region of the medial femoral condyle (arrow).

Figure S38-6  Malignant transformation of an infarct with replacement of the inferior sclerotic demarcation of the infarct by a permeative bone lesion (asterisk) and periostitis (arrows).

CASE 39

Figure S39-3  A younger patient shows characteristic findings of infection with a lytic metaphyseal lesion (arrow) with surrounding sclerosis. The growth plate is not fused and appears normal.

Figure S39-4  Fluid-sensitive magnetic resonance image shows a marrow replacement by tumor (asterisk) associated with cortical infiltration, causing the periostitis laterally (red arrow) and a soft tissue mass medially (yellow arrows).

CASE 40

Figure S40-3  The shoulder is a common site of involvement. Note the multiple rounded, variably sized calcium deposits (asterisk) over the shoulder girdle.

Figure S40-4  Sagittal fluid-sensitive image shows that some of the masses are cystic with fluid-fluid levels caused by the layering of calcium (arrows).

437

CASE 41

Figure S41-4  Image from a bone scan shows diffuse periosteal uptake (arrows) in the forearm and hands.

Figure S41-3  In the shoulder, there was involvement of the proximal humerus, clavicle, and scapula (arrows).

Figure S41-6  Patient with venous stasis shows smooth periosteal reactive changes in the distal tibia (arrows) and numerous phleboliths. Figure S41-5  Patient with hypertrophic osteoarthropathy shows smooth periostitis in the distal radius, first and fifth metacarpals, and second and fourth phalanges (arrows).

438

CASE 42

Figure S42-3  This patient had mixed-type neuroarthropathy of the hip with lysis of the femoral head and neck (yellow arrow) and fragmentation of the greater trochanter (asterisk) and osseous debris in the acetabular fossa (red arrow). Figure S42-4  This diabetic patient shows classic findings of hypertrophic neuroarthropathy (asterisk) with dislocations, disorganization, increased density, and joint debris.

CASE 43

Figure S43-4  Sagittal reformatted computed tomographic image shows that the fragment of bone (asterisk) was connected to the posterior cruciate ligament (arrow) and came off the posterior aspect of the tibial eminence.

Figure S43-3  Type II tibial eminence fracture on a T2-weighted image depicted as a discontinuity of the cortex (arrow) with surrounding bone marrow edema. The fragment (asterisk) is elevated anteriorly but remains attached posteriorly.

439

CASE 44

Figure S44-3  Axial computed tomographic image shows widened marrow cavities in the iliac bones and marked coarsening of the trabeculae in the innominate bones (asterisk) and sacrum.

Figure S44-4  Anteroposterior radiograph of the lumbar spine shows diffuse osteopenia, marked thinning of the cortices, and coarsening of the trabeculation. Note that numerous disk spaces are narrowed from early degeneration. The patient had a cholecystectomy (arrow).

Figure S44-6  Chest computed tomography shows a posterior mediastinal mass in a patient with extramedullary hematopoiesis (arrow). Figure S44-5  Chest radiograph shows extramedullary hematopoiesis with expansion of several of the ribs (arrows).

440

CASE 45

Figure S45-4  Osseous variant. Sagittal T2-weighted image shows fractures bisecting the spinous process and vertebral body (arrows). There is a large subcutaneous hematoma dorsally (asterisk).

Figure S45-3  Anteroposterior radiograph of the lumbar spine shows bilateral horizontal fractures of the L3 transverse processes (arrows) bisecting them into superior and inferior fragments. The widening of the interspinal distances between the spinous processes of the L2 and L3 vertebrae results in an “empty hole” sign (asterisk).

Figure S45-5  Ligamentous variant. Sagittal reformatted computed tomographic image shows distraction of the interspinous distance between the L2 and L3 spinous processes (asterisk) and a dorsal subcutaneous hematoma (arrow).

441

CASE 46

Figure S46-3  This patient shows absence of the distal sacrum (arrow) but no other significant abnormality.

CASE 47

Figure S47-4  Reformatted computed tomographic image shows distortion of the distal femur by numerous enchondromas, some that occupy the medullary cavity and others that are more peripherally located (arrows).

Figure S47-3  Frontal view of the right hip shows enchondromas in both the proximal femur and the lateral aspect of the right ilium with characteristic chondroid matrix (asterisks).

442

Figure S47-5  Axial computed tomographic image shows chondroid matrix formation in an intramedullary lesion (asterisk) and in one that is more pedunculated (arrow).

Figure S47-6  Follow-up radiograph on this patient shows an amputation of the fifth finger. Note that there is expansion of the head of the second proximal phalanx indicating malignant transformation (arrow) (compare with Figure S47-1).

CASE 48

Figure S48-3  Sagittal reformatted computed tomographic image shows dorsal subluxation of the second metatarsal base (yellow arrow) and a small fleck of bone arising from the third tarsometatarsal joint (red arrow).

Figure S48-4  Axial short tau inversion recovery magnetic resonance image shows marked marrow edema in the base of the second and third metatarsal bases and focal disruption of the Lisfranc ligament (arrow).

443

Figure S48-5  Another patient shows lateral subluxation of all five rays consistent with a complete homolateral pattern of subluxation. Note the offset of the first and second tarsometatarsal (TMT) joints (red arrows) and widening of the third TMT joint (yellow arrow).

Figure S48-6  This patient shows widening of the intermetatarsal space, an avulsion fracture fragment (yellow arrow) and medial subluxation of the medial cuneiform (red arrow) consistent with a partial incongruity or divergent pattern.

CASE 49

Figure S49-3  Flexion shows that the os odontoideum (arrow) moves with the anterior arch of C1, resulting in narrowing of the spinal canal (asterisk).

444

Figure S49-4  Sagittal reformatted computed tomography shows the anomalous ossicle and the similarity in appearance to a type 2 odontoid fracture. Note the hypertrophied anterior arch of C1 (yellow arrow) and hypoplasia of the posterior arch (red arrow).

CASE 50

Figure S50-3  Frontal spot image from a bone scan shows fusiform linear activity along the medial cortex of the distal femur (arrow).

Figure S50-4  Coronal T1-weighted image shows the longitudinal fracture as a linear area of isointense signal within the medial femoral cortex (arrows).

CASE 51

Figure S51-3  Oblique radiograph of the left foot shows soft tissue masses adjacent to erosions in the first metatarsal head and proximal phalanx (arrows). This is a classic location for gout.

Figure S51-4  Hand radiograph in a patient with severe involvement. There is lysis of the bone from the tophi at the second and fifth metacarpophalangeal joints and arthritis mutilans (asterisk) of the carpus of the wrist. The tophi appear dense because they contain calcifications (arrows).

445

Figure S51-5  T2-weighted image shows a predominantly low signal intensity mass in the ankle consistent with a tophus (arrow).

Figure S51-6  Dual-energy computed tomography can differentiate urate crystals from calcium by using specific attenuation characteristics, which can be useful in identifying deposits (arrows) shown in green.

CASE 52

Figure S52-4  Three-dimensional volume-rendered computed tomographic image clearly depicts a pseudoarthrosis (arrow). The   patient had regained some of his mobility.

Figure S52-3  Axillary view in another patient shows a chronically unreduced posterior shoulder dislocation with a large trough lesion (arrows).

446

Figure S52-5  Anteroposterior radiograph of the shoulder shows a positive rim sign (widening of the joint space greater than 6 mm) (asterisk), a broken scapulohumeral arch sign (arrows), and absence of the “half-moon” sign (overlap of the humeral head and glenoid rim).

Figure S52-6  Fluid-sensitive magnetic resonance image shows a slightly posteriorly subluxed humeral head. Note the fragment of bone that is displaced posteriorly and medially representing the bony reverse Bankart fracture (arrow).

CASE 53

Figure S53-3  Pathologic fracture (arrows) has developed where the bone has weakened from the chronic infection.

Figure S53-4  Anteroposterior view of the left calf shows a 3-cm geographic-shaped radiolucent lesion (arrow) in the proximal tibial shaft surrounded by osteosclerosis and thickening of the cortex.

447

Figure S53-6  Axial T2-weighted image shows the abscess within the tibia (arrow) with prominent endosteal and periosteal reactive changes.

Figure S53-5  Lateral view shows the lesion more clearly as an abnormality centered within the medullary cavity (arrow). The bone surrounding the abscess produces fusiform thickening of the cortex and marked endosteal reaction.

CASE 54

Figure S54-3  T2-weighted image shows a linear low signal intensity abnormality in the subchondral region of the weight-bearing surface of the femoral head (arrow) suggestive of an insufficiency fracture.

448

CASE 55

Figure S55-2  Sagittal T1-weighted image shows homogeneous high T1 signal intensity except for the central calcification (arrow). These findings are characteristic of a Milgram stage 2 lesion.

Figure S55-3  This patient has a stage 3 lesion, which appears as a well-circumscribed intramedullary lesion that is nearly completely ossified (arrow).

Figure S55-4  The T1-weighted image, however, shows scattered areas of lipomatous tissue (arrow) with the ossified mass. This lesion was excised and pathologically proven to be an intraosseous lipoma.

449

CASE 56

Figure S56-3  Finger radiograph shows diffuse swelling about the distal interphalangeal (DIP) joint and erosive changes at the margins of the DIP joint associated with proliferation of new bone, producing the “Mickey Mouse” sign (arrows).

Figure S56-4  This patient has a classic “sausage finger” (arrows) characteristic of the oligoarticular pattern.

Figure S56-5  This finger shows advanced psoriatic arthritic changes with a “pencil-in-cup” (arrow) deformity and surrounding soft tissue swelling.

Figure S56-6  Polyarthritis pattern with involvement of the interphalangeal (IP) joints of the fingers. Note the ankylosis of the fourth distal IP and fifth proximal IP joints (red arrows) and the periostitis (yellow arrows) in the middle phalanges of the second through fourth rays.

450

CASE 57

Figure S57-3  Sagittal T2-weighted image shows increased T2 signal intensity in both the supraspinatus (asterisk) and infraspinatus muscles (arrow) consistent with subacute impingement of the suprascapular nerve.

Figure S57-4  Sagittal T2-weighted image shows increased T2 signal intensity in the infraspinatus muscle (asterisk) consistent with impingement of the suprascapular nerve below the spinoglenoid notch.

Figure S57-5  Sagittal T1-weighted image shows isolated atrophy of the teres minor muscle (asterisk).

Figure S57-6  Sagittal T2-weighted image shows edema in the infraspinatus (asterisk) and teres minor (arrow) muscles. This indicates involvement of two nerves, the suprascapular and axillary nerves, diagnostic of Parsonage-Turner syndrome.

451

CASE 58

Figure S58-3  This patient presented with a broken locking ring (arrow).

Figure S58-4  This patient presented with pain and diminished range of motion 6 years after the surgery. There is severe wear of the weight-bearing portion of the arthroplasty, causing superior migration of the head of the femoral implant (arrow).

Figure S58-5  Dislocations remain one of the most common causes of hip revision surgery. Note that the head (asterisk) is disarticulated posteriorly from the acetabular cup.

Figure S58-6  This patient had loosening of the acetabular cup with malrotation. Note the zone of lucency surrounding the cup (arrows).

452

CASE 59

Figure S59-4  Axial computed tomographic image shows central areas of calcifications (red arrow) within the soft tissue mass (yellow arrow). Figure S59-3  Radiograph of the proximal humerus shows a wellcircumscribed lesion on the medial surface of the humerus associated with a rim of sclerosis and saucerization of the bone (arrows).

Figure S59-5  Coronal T2-weighted image of the tibia shows a periosteal lesion measuring 4 cm at its base that is associated with peritumoral edema (arrows). Biopsy confirmed the diagnosis of periosteal chondrosarcoma.

Figure S59-6  Axial fluid-sensitive image shows an amorphous mass associated with the area of scalloping in the tibia with areas of low signal intensity that corresponded to calcifications (arrow).

453

CASE 60

Figure S60-3  Axial computed tomographic image shows a “reverse hamburger bun” sign (arrow) and a fracture of the adjacent lamina.

Figure S60-4  Sagittal reformatted image shows a “perched” facet joint (arrow).

Figure S60-5  This patient presented with bilateral interfacetal dislocations. Note the widening of the interspinous distance (arrow), bilateral perched facet joints, widening of the posterior disk space, and anterolisthesis of the C5 vertebral body (asterisk).

454

Figure S60-6  Sagittal fluid-sensitive magnetic resonance image shows the disruption of the interspinous and supraspinous ligaments (asterisk) and signal intensity changes in the spinal cord (arrow) at the level of kyphosis in a patient with bilateral interfacetal dislocations.

CASE 61

Figure S61-3  Wrist radiograph in another patient with marked periarticular osteopenia in the carpus and distal radius (arrows).

Figure S61-5  Bone scan shows increased uptake in a periarticular distribution (asterisk).

Figure S61-4  Fluid-sensitive magnetic resonance image shows bone marrow edema (asterisks) that is characteristic in the warm phase of reflex sympathetic dystrophy.

Figure S61-6  Patient in the cold phase shows contractures of the third through fifth fingers (arrows).

455

CASE 62

Figure S62-3  Axial computed tomographic image shows the characteristic appearance of an osteoid osteoma in the subperiosteal region of the cortex (red arrow) with surrounding periosteal reaction (yellow arrows).

Figure S62-4  Coronal reformatted computed tomographic image shows the nidus of an osteoid osteoma (arrow) with intense surrounding sclerosis.

CASE 63

Figure S63-3  This patient had central and marginal erosions in the proximal interphalangeal joints (arrows) and prominent soft tissue swelling. Figure S63-2  Close-up view of an anteroposterior radiograph of the right hand shows the soft tissue swelling about the second and third proximal interphalangeal (IP) joints. Note the central erosions in the third distal IP joint with a typical gull wing deformity (arrow). Erosions are also evident in the second, third, and fifth proximal IP joints.

456

CASE 64

Figure S64-3  Pyomyositis in the invasive stage. There is an enlarged lymph node in the medial aspect of the elbow (arrow). Note the relative absence of cellulitis or suppuration.

Figure S64-4  Suppurative phase of pyomyositis. T2-weighted image shows an abscess in the vastus lateralis muscle (arrow).

Figure S64-5  Fat-saturated T1-weighted image after intravenous contrast shows rim enhancement of the abscess (arrow).

Figure S64-6  Computed tomographic image shows that the abscess had enlarged significantly over the last few days (asterisk).

457

CASE 65

Figure S65-3  Anteroposterior radiograph of the right elbow, with normal left elbow as a comparison, shows separation of the right medial epicondylar apophysis with heterogeneous density and fragmentation (arrow).

Figure S65-2  Anteroposterior radiograph in an 8-year-old patient shows complete avulsion of the medial epicondylar apophysis that has become entrapped in the medial joint space (arrow). Note the empty area in the medial humerus that should contain the apophysis (asterisk).

Figure S65-4  Radiograph of an older patient shows complete avulsion of a previously fused medial epicondylar apophysis (arrow). This can occur with an elbow dislocation.

CASE 66

Figure S66-2  Coronal T1-weighted image shows the incomplete cleft in the dorsum of the navicular (arrow). The plantar aspect of the bone is intact.

Figure S66-3  Axial T2-weighted image shows bone marrow edema on both sides of the fracture (arrow) but more pronounced medially (asterisk).

Figure S66-5  This patient developed avascular necrosis. Note the distracted fracture in the navicular (red arrows) and the increased density in the lateral fragment (yellow arrow).

Figure S66-4  Three-dimensional volume rendered computed tomographic image depicts the navicular stress fracture well (arrow).

459

CASE 67

Figure S67-3  Lateral radiograph of the knee shows significant narrowing of the patellofemoral joint and extensive subchondral cyst formation in the patella (asterisk). The chondrocalcinosis of the menisci are also evident (arrows).

Figure S67-4  T2-weighted image shows a joint effusion and bone marrow edema in the femoral head and acetabulum, manifestations of synovial inflammation. There is also a perilabral cyst (arrow) indicative of a tear.

Figure S67-5  Corresponding radiographs taken 2 years apart show rapid joint loss in the second radiograph and formation of large subchondral cysts (arrows) and osteophytes. Osteoarthritis does not progress this quickly.

460

CASE 68

Figure S68-3  Axial T1-weighted image of this patient shows heterogeneous signal intensity in the chordoma (asterisk).

Figure S68-5  Coronal T1-weighted image in another patient shows a large mass in the midline (asterisk).

Figure S68-4  Axial T2-weighted image shows the characteristic polylobulated hyperintense appearance of this lesion and a low signal intensity pseudocapsule (arrows).

Figure S68-6  There is massive infiltration of the parasacral soft tissues, including the gluteal musculature (asterisk) and the presacral fat (arrows), with anterior displacement of the gastrointestinal tract.

461

CASE 69

Figure S69-2  Radiograph shows an anterior-posterior compression type 2 injury with subtle diastasis of the anterior sacroiliac joints (yellow arrows) and the symphysis pubis. An important observation is a fracture of the L5 transverse process that indicates a posterior ring injury (red arrow).

Figure S69-4  Radiograph shows a vertical shear injury with superior displacement of the left innominate bone relative to the sacrum and right side of the pelvis. Note the disarticulated left sacroiliac joint and symphysis pubis (arrows).

462

Figure S69-3  Patient with lateral compression injury type 1 with a buckle fracture of the left sacrum (yellow arrow) and minimally displaced fractures of the right superior and inferior pubic rami (red arrows).

Figure S69-5  Three-dimensional reformatted computed tomographic image shows a combined pelvic ring fracture with elements of a lateral compression injury in the right pubic bone (arrows), and a vertical shear in the left innominate bone (asterisk).

CASE 70

Figure S70-3  Gadolinium-enhanced image in another patient shows marrow enhancement most intense at the end plates (asterisk) and at the infected soft tissues (arrows).

Figure S70-2  Coronal reformatted computed tomographic image shows erosive changes in the end plates (arrows) with surrounding reactive sclerosis and disk space loss.

CASE 71

Figure S71-2  A young patient with achondroplasia shows disproportionate fibular growth (arrows), which is likely the cause of the genu varum deformity.

Figure S71-3  Physiologic genu varum (asterisk) occurs in children younger than age 2 years.

463

CASE 72

Figure S72-3  This patient had widening of the fourth and fifth carpometacarpal (CMC) joints (arrows). Compare these joints to the second and third CMC joints.

Figure S72-4  Lateral view shows dorsal soft tissue swelling, dorsal subluxation of the fourth and fifth metacarpal bases, and a fracture of the hamate (arrow).

CASE 73

Figure S73-3  T1-weighted image in another patient shows decreased signal intensity in the posterolateral aspect of the talus (asterisk), the area most vulnerable to vascular injury.

464

Figure S73-4  Coronal reformatted computed tomographic image shows a demarcated area of sclerosis (asterisk) consistent with avascular necrosis in the area depicting bone marrow edema on the magnetic resonance image. Note absence of the Hawkins sign.

CASE 74

Figure S74-3  Shoulder radiograph in a patient with traumatic brain injury shows heterotopic ossification extending from the glenoid to the anatomic neck of the humerus (arrows).

Figure S74-4  Involvement of the elbow is common in patients with neurogenic conditions and burns. This burn victim shows heterotopic ossification in the medial aspect of the elbow (arrows).

Figure S74-5  Pelvic radiograph in a paraplegic patient shows marked osteopenia and ossific masses surrounding both hip joints (arrows).

Figure S74-6  Preoperative computed tomography in another patient shows near ankylosis of the medial aspect of the joint by large ossifications (asterisk).

465

CASE 75

Figure S75-4  Close-up of another patient with a medial Segond fracture (arrow) after a twisting injury.

Figure S75-3  Sagittal T2-weighted image shows complete disruption of the posterior cruciate ligament (arrow).

Figure S75-5  Coronal T1-weighted image shows the bone marrow edema (arrow) in the donor site of the defect.

466

Figure S75-6  Coronal T2-weighted image shows a tensile contusion (red arrow) and stripping of the meniscotibial ligament (yellow arrow). The force of injury was just short of creating a medial Segond fracture.

CASE 76

Figure S76-3  Longitudinal sonogram shows normal Achilles tendon with linear echogenic architecture and smooth surface contours (arrows). Note posterior acoustic shadowing from calcaneus (asterisk).

Figure S76-4  Lateral radiograph shows attenuated Achilles tendon (arrow).

Figure S76-5  Sagittal T2-weighted image shows complete tear of the Achilles tendon with a large gap (asterisk) and separated free ends of the tendon (arrows).

Figure S76-6  Sagittal T2-weighted image shows a partial tear at the musculotendinous junction (arrow) with interstitial edema within a thickened tendon. Note the surrounding edema in the perivenous soft tissues, as well as the Kager fat pad (asterisk).

467

CASE 77

Figure S77-3  Index finger shows fullness of the subungual soft tissues and a scalloped area of erosion in the dorsal aspect of the tuft of the distal phalanx (arrow).

Figure S77-4  Coronal T1-weighted image shows a hypointense mass dorsal in the distal phalanx (arrow).

Figure S77-5  Sagittal enhanced fat-saturated T1-weighted image shows intense enhancement (arrow) typical of a glomus tumor and enhancement of the marrow.

Figure S77-6  Another patient shows lysis of nearly the entire distal phalanx. In this case, it was destruction of the bone from extension of a subungual melanoma but could easily have been from a lung metastasis.

468

CASE 78

Figure S78-3  Another patient shows similar sheetlike calcifications in the thigh (arrows).

Figure S78-4  Occasionally, dystrophic calcifications (asterisk) localize in a periarticular distribution.

Figure S78-5  Axial T2-weighted image of the thigh shows hyperintense signal (asterisk) in the quadriceps muscles and perimuscular edema. Calcific deposits appear low in signal intensity (arrows).

Figure S78-6  Coronal T2-weighted image in a patient with juvenile dermatomyositis shows intense edema in the quadriceps musculature (asterisk) and perimuscular edema.

469

CASE 79

Figure S79-3  Coronal inversion recovery magnetic resonance image shows a Stener lesion, with retraction of the ulnar collateral ligament into a “yo-yo” sign (yellow arrow), indicating that it is lying superficial to the adductor pollicis aponeurosis. The bone defect is also shown (red arrow).

Figure S79-4  Fracture fragments that involve more than 30% of the articular surface or are significantly rotated are more likely to be treated surgically. Note the articular step-off (arrow).

Figure S79-5  This patient did not present until 3 weeks later, and the avulsed fragment was distracted from the rest of the bone (arrow).

Figure S79-6  Occasionally, an area of lysis is the key to diagnosis (yellow arrow). The avulsed fragment is quite small in this patient (red arrow).

470

CASE 80

Figure S80-3  Axial T2-weighted image in another patient shows a polylobular low signal mass (arrows) in the volar aspect of the third finger.

Figure S80-2  Coronal T1-weighted image shows a well-contained hypointense to isointense solid mass (arrow).

Figure S80-4  Lateral view of the thumb shows a dense soft tissue mass in the volar aspect of the base of the thumb (arrow).

Figure S80-5  Axial fat-saturated T1-weighted image after intravenous administration of gadolinium shows heterogeneous but intense enhancement of the mass (arrow) in Figure S80-4.

471

CASE 81

Figure S81-2  Sagittal fat-saturated T2-weighted image shows marked thickening of the plantar fascia with interstitial edema (arrow) as well as perifascial edema at the muscle-fascia and subcutaneous fat-fascia interfaces.

Figure S81-3  Sagittal T1-weighted image in another patient shows marked thickening of the plantar fascia (arrow) and perifascial inflammation.

Figure S81-4  Coronal T2-weighted image shows that the thickened fascia involved the central (yellow arrow) and lateral (red arrow) cords.

Figure S81-5  Patient with chronic plantar fasciitis with an enthesophyte (red arrow) arising from the medial plantar process and a thickened fascia (yellow arrows).

472

CASE 82

Figure S82-3  This patient has protrusio acetabuli (arrows) with diffuse joint space narrowing, resulting in overcoverage of the femoral head.

Figure S82-4  This patient has cranial acetabular retroversion with a “bow tie” sign from the bony protuberance in the anterior aspect of the superior acetabulum (yellow arrow). The posterior rim line (red arrows) should not project medial to the anterior rim line.

CASE 83

Figure S83-4  Axial computed tomographic image shows a cleavage plane (arrow) between the mass and the cortex, but as the tumor grows this cleavage plane disappears as the osteosarcoma reattaches to the cortex.

Figure S83-3  Lateral radiograph of the knee in a different patient shows an exophytic osteoblastic mass arising from the posterior metadiaphyseal region of the femur (asterisk), the most common location typical of a parosteal osteosarcoma.

473

CASE 84

Figure S84-3  Sagittal T1-weighted image of the ankle shows low signal intensity synovium within the anterior and posterior capsule (arrows).

Figure S84-4  Axial T2-weighted image shows blooming of the low signal intensity synovium (arrows) caused by hemosiderin-laden macrophages.

Figure S84-6  Axial computed tomographic image shows a large cystic mass arising from beneath the periosteum of the femur. Areas of increased density represent acute blood products in this pseudotumor (arrow). Figure S84-5  Radiograph of a pseudotumor shows scalloping of the bone from a dense soft tissue mass (asterisk).

474

CASE 85

Figure S85-3  This patient has mild radiation-induced dextroscoliosis centered at L3 (asterisk).

Figure S85-4  Lateral view shows the bone-in-bone deformity (arrows).

CASE 86

Figure S86-3  Another patient shows classic multiple subluxations, particularly in the left hand. There is widening of the scapholunate interval from ligamentous stretching (yellow arrow). Note the subluxation of the interphalangeal joint of the right thumb resulting in “hitchhiker’s thumb” (red arrow).

Figure S86-4  Frontal radiograph of the hip shows increased density in the femoral head consistent with avascular necrosis (arrow).

475

CASE 87

Figure S87-3  Axial proton density-weighted image shows a polylobulated soft tissue mass that is isointense to fat, predominantly in the posterior compartment of the thigh but infiltrating the vastus intermedius muscle. It abuts the posteromedial surface of the femur, producing the scalloped appearance seen on the radiographs (yellow arrow). Phleboliths appear low in signal intensity (red arrows).

Figure S87-2  Radiograph of the distal right thigh shows a large soft tissue mass in the posteromedial aspect of the thigh associated with scalloping of the adjacent femoral cortex (arrow) and reactive periostitis. It contains numerous calcifications compatible with phleboliths (asterisk).

Figure S87-4  T2-weighted image shows tubular areas of high signal intensity from the angiomatous component and fluid-fluid levels (arrow).

Figure S87-5  There was overgrowth of the proximal ulna (asterisk) and periosteal reactive changes from a surrounding hemangioma in the proximal forearm.

476

CASE 88

Figure S88-3  Axial computed tomographic image shows coarsening of the trabeculation and loss of mineralization (arrows) in the innominate bones and sacrum.

Figure S88-4  Shoulder radiograph in another patient shows typical changes with diffuse osteopenia and trabeculation that appears smudged and indistinct (asterisk).

Figure S88-5  Lateral spine radiograph shows numerous osteomalacic compression fractures (arrows) and a “rugger jersey” spine.

Figure S88-6  Forearm radiograph shows a linear lucency in the distal shaft of the ulna (arrow). No lucency surrounds this lesion, indicating the absence of a healing response.

477

CASE 89

Figure S89-3  Coronal post-intravenous contrast image shows diffuse enhancement of the neuroma (arrow).

Figure S89-4  Another patient shows a Morton neuroma (arrow) arising from between the second and third metatarsal heads.

CASE 90

Figure S90-3  In the odontoid view, the lateral edges of the C1 lateral masses should align with the lateral cortex of C2. If they do not (arrows), look for asymmetric widening of the space to the odontoid process (asterisk).

478

Figure S90-4  About 33% of patients with a Jefferson fracture (yellow arrow) have a concomitant fracture of C2. In this patient, it was a displaced fracture at the base of the odontoid process (red arrow).

CASE 91

Figure S91-2  Hip radiograph shows a classic bone-in-bone appearance in the femur (arrows).

Figure S91-3  Lateral radiograph of the spine shows characteristic central end-plate depressions in the thoracic and upper lumbar spine (asterisks) and typical biconcave deformities of the lower lumbar vertebrae.

CASE 92

Figure S92-3  Axillary view of the shoulder shows a transverse lucency (arrow) with marginal nonbridging callus formation in a patient with nonunion.

Figure S92-4  A patient with an anterior subcoracoid glenohumeral joint dislocation shows a displaced fracture of the greater tuberosity (asterisk) and the tip of the coracoid process (arrow).

CASE 93

Figure S93-3  Another patient with a congenital form, type III, with multiple fractures of the lower extremity (arrows).

Figure S93-5  Right leg shows significant deformities including bowing of the femur, tibia, and fibula (arrows). The bones are markedly osteopenic with extremely thin cortices.

480

Figure S93-4  Adult patient with numerous rib and proximal humeri fractures (arrows).

Figure S93-6  The left thigh shows extensive callus formation involving the entire length of the left femur (asterisk).

CASE 94

Figure S94-3  Radiograph of the hip shows mild narrowing of the superior joint space, femoral osteophyte formation laterally, prominent cystlike erosions in the articular surface of the femoral head (arrows), and constriction of the femoral neck region.

Figure S94-4  Sagittal T2-weighted image shows that the erosive changes caused by the low signal intensity synovial masses (asterisks) are not isolated to the femoral head and neck, but also involve the acetabulum (arrow).

CASE 95

Figure S95-3  This patient shows a peroneus brevis avulsion fracture (arrow) that extends to the articular surface, producing a triangular fragment of bone from the fifth metatarsal tuberosity.

Figure S95-4  This patient shows a small avulsion fracture (arrow) involving the tip of the tuberosity of the base of the fifth metatarsal bone. This appearance is typical of an avulsion of the lateral cord of the plantar fascia.

481

CASE 96

Figure S96-3  Sagittal T2-weighted image shows a double posterior cruciate ligament sign from the extruded fragment (arrow) appearing anterior to the normal posterior cruciate ligament.

Figure S96-5  Sagittal proton density-weighted image shows a fragment extruded into the intercondylar notch (arrow).

482

Figure S96-4  Sagittal proton density-weighted image shows that the posterior horn fragment has “flipped” anteriorly (arrow), giving the appearance of two anterior horns.

Figure S96-6  Sagittal T2-weighted image shows a posteriorly flipped fragment (arrow).

CASE 97

Figure S97-3  Anteroposterior radiograph of the thoracolumbar spine shows that the paravertebral ossification is more pronounced on the right side of the spine (arrows). The pulsatile effect of the aorta suppresses the bony outgrowth on the left side of the spine.

Figure S97-4  Sagittal proton density-weighted image shows that the ossifications terminate at the superior and inferior end plates. Note the anteriorly extruded disks (arrows).

Figure S97-5  Anteroposterior radiograph of the spine shows ossification of the iliolumbar ligaments (arrows). Note that paraarticular enthesopathy results in pincer-type femoroacetabular impingement with overcoverage of the femoral head (asterisk).

Figure S97-6  Sagittal reformatted computed tomographic image shows characteristic changes of diffuse idiopathic skeletal hyperostosis as well as ossification of the posterior longitudinal ligament (arrow).

483

CASE 98

Figure S98-2  Coronal T1-weighted image shows a small fragment avulsed from the proximal humerus (arrow) with surrounding marrow edema. This fracture was radiographically occult.

Figure S98-3  Coronal reformatted computed tomographic image shows a displaced greater tuberosity fragment (yellow arrow). Note the initial site of impaction with a depressed cortical fragment (red arrow).

Figure S98-4  Three-dimensional volume-rendered computed tomographic image shows the entire fragment (arrow) and its position with respect to the remainder of the humeral head. Figure S98-5  This patient had a split-type isolated greater tuberosity fracture (arrows).

484

CASE 99

Figure S99-4  Axial fluid-sensitive image shows the extent of the space-occupying process (arrow).

Figure S99-3  Coronal T1-weighted image shows an ovoid, heterogeneous soft tissue mass of intermediate signal intensity in the anterior joint recess (arrow).

Figure S99-5  Sagittal fluid-sensitive image in a different patient shows more extensive arthrofibrosis involving the anterior joint recess and the intercondylar region (arrows).

Figure S99-6  This patient had arthrofibrosis surrounding the proximal posterior cruciate ligament (arrow).

485

CASE 100

Figure S100-4  T2-weighted image of a patient with de Quervain tenosynovitis shows thickened tendon sheath with low signal intensity corresponding to fibrosis in the first extensor compartment (arrow). Figure S100-3  Fat-saturated T1-weighted image shows diffuse enhancement of the sheaths after intravenous administration of gadolinium (arrows).

CASE 101

Figure S101-3  Sagittal T2-weighted image shows focal cartilage edema (arrow) overlying the insufficiency fracture.

486

Figure S101-4  T1-weighted image shows marked cartilage denudement, cortical irregularity, subchondral marrow changes, and a complex tear of the medial meniscus (arrow).

CASE 102

Figure S102-3  In some patients, the accessory soleus muscle is small. In others, the anomalous muscle is large and fleshy (asterisk). These patients often present with claudication.

Figure S102-4  Axial image in a patient presenting with tarsal tunnel syndrome shows an accessory flexor digitorum longus muscle (arrow) within the tarsal tunnel.

CASE 103

Figure S103-3  Coronal reformatted computed tomographic image shows that there was a fibrous coalition involving the middle subtalar joint (arrow).

Figure S103-4  Axial T1-weighted image in another patient shows an osseous talocalcaneal coalition with osseous continuity across the joint (arrow).

CASE 104

Figure S104-3  This patient had a type III superior labral anterior posterior lesion with a bucket-handle tear of the superior labrum (yellow arrow). Note the intact biceps tendon (red arrow).

Figure S104-5  This patient had a type IV superior labral anterior posterior lesion with a labral tear (yellow arrow) that involves the biceps anchor (red arrow).

488

Figure S104-4  Another patient with a type III superior labral anterior posterior lesion shows a typical “double Oreo cookie” sign (arrow).

Figure S104-6  Another patient with a type IV superior labral anterior posterior lesion shows a bulbous biceps anchor with increased T2 signal intensity (arrow).

CASE 105

Figure S105-3  Frontal pelvic radiograph shows a focal area of bone lysis in the right ischium (arrow) where the hamstring tendon has been avulsed.

Figure S105-4  This patient shows a partial tear of the hamstring origin with only the conjoint tendon retracted (yellow arrow). The semimembranosus component is still attached (red arrow).

Figure S105-5  Sagittal fluid-sensitive image shows the tendinous disruption (asterisk) and the intact semimembranosus tendon (arrows).

Figure S105-6  Axial image shows perineural edema surrounding the sciatic nerve (arrow) in a patient with a complete and retracted tendon origin.

489

CASE 106

Figure S106-3  Axial T2-weighted image in a different patient shows a long head of the biceps tendon (yellow arrow) that is subluxed and perched on the lesser tuberosity. The lateral slip of the coracohumeral ligament is still attached to the lateral lip. The subscapularis tendon insertion is partially torn (red arrow).

Figure S106-4  Axial gradient-recalled echo image in another patient shows that the subscapularis tendon is completely torn from the lesser tuberosity (red arrow). The biceps tendon is perched on the lesser tuberosity (yellow arrow).

Figure S106-5  Axial gradient-recalled echo image shows complete intraarticular dislocation of the biceps tendon (yellow arrow). Note the “empty hole” sign (red arrow).

490

CASE 107

Figure S107-2  Hand radiograph shows periarticular osteopenia, soft tissue swelling, and erosive changes (arrows).

Figure S107-3  Sagittal T1-weighted image shows a volarly tilted lunate (asterisk) with an erosion anteriorly (arrow).

Figure S107-5  Shoulder radiograph shows deformity of the glenohumeral joint (arrow) from severe synovitis and erosions.

Figure S107-4  Frontal radiograph of the elbow shows marked overgrowth of the radial head (arrow) and narrowed joints.

491

CASE 108

Figure S108-2  Sagittal proton density-weighted image shows complete disruption of the proximal patellar tendon (arrow).

Figure S108-3  This patient had a rupture of the distal aspect of the patellar tendon (arrow).

Figure S108-4  T2-weighted image shows a high-grade partial tear. The majority of the tendon has avulsed off the patella. The rectus femoris component (arrows) remained intact but was completely stripped off the bone.

492

Figure S108-5  Another patient with a ruptured patellar tendon shows a high-riding patella, a bony avulsion fracture (arrow), and distention of the prepatellar bursa (asterisk).

CASE 109

Figure S109-3  Axial proton density-weighted image distal to the pisiform shows a flattened median nerve (yellow arrow) and persistent bowing of the flexor retinaculum (red arrow).

Figure S109-5  Another patient with carpal tunnel syndrome shows a markedly enlarged median nerve (arrow), bowed retinaculum, and loss of the dorsal carpal tunnel fat.

Figure S109-4  T2-weighted image at the level of the carpal tunnel shows increased signal intensity of the median nerve (arrow).

Figure S109-6  Ultrasound of a patient with carpal tunnel syndrome shows a dramatic change in the caliber of the median nerve with a thickened morphology at the carpal tunnel (yellow arrow) and a normal appearance more distally (red arrow).

493

CASE 110

Figure S110-2  Another patient shows a displaced femoral implant with prominent periprosthetic geographic bony lysis (yellow arrows) and lucency at the implant-cement interface (red arrows).

Figure S110-3  Reformatted coronal computed tomographic image shows a granuloma adjacent to the iliac bone with debris (arrow).

Figure S110-4  Lateral radiograph shows normal postoperative appearance of a total knee arthroplasty.

494

Figure S110-5  Radiograph obtained 3 years later shows periprosthetic bone lysis in the anterior femur (yellow arrow) and extensive lucencies surrounding the tibial implant. Note the fractured cement interfaces (red arrows). Granulomatous deposits are present throughout the joint (asterisks).

CASE 111

Figure S111-3  Axial fluid-sensitive image shows that the fracture fragment (blue arrow) is located in between the Gerdy tubercle (yellow arrow) and the oblique band of the lateral collateral ligament (red arrow).

Figure S111-5  Another patient with a Segond fracture. The computed tomographic image shows involvement of the posterior aspect of the Gerdy tubercle (arrow). The radiograph was normal.

Figure S111-4  Anteroposterior radiograph of the knee shows a typical Segond fracture (arrow) as a cortical avulsion from the lateral tibial cortex occurring about 4 mm below the level of the articular surface.

Figure S111-6  Several months later, a coronal T1-weighted image showed a tubercle at the location of the healed Segond fracture (arrow).

495

CASE 112

Figure S112-2  This patient had breast cancer, and her skull shows several variably sized lytic lesions (arrows) from metastatic disease.

Figure S112-3  This skull shows a salt-and-pepper appearance with numerous tiny hyperlucent areas caused by resorption of the trabecular bone (asterisk) affecting the inner and outer tables in a patient with hyperparathyroidism.

CASE 113

Figure S113-4  Sagittal fat-saturated T1-weighted image after intravenous gadolinium shows the entire length of the wood splinter (arrow) surrounded by an enhancing phlegmon.

Figure S113-3  Coronal proton density-weighted image shows a punctate low signal intensity abnormality (yellow arrow) consistent with a wood splinter with overlying cellulitis (red arrow).

496

CASE 114

Figure S114-3  Another patient with the boutonnière deformity of the index finger shows typical findings with marked flexion of the proximal interphalangeal joint (arrow) and soft tissue swelling.

Figure S114-4  Axial T1-weighted image at the level of the neck of the proximal phalanx of the index finger shows absence of the central slip of the extensor tendon from retraction (arrow). The normal extensor apparatus may be seen in the third and fourth fingers.

CASE 115

Figure S115-3  Sagittal T2-weighted image shows the devastating effects of the fracture on the spinal canal, with marked narrowing resulting in edema and hemorrhage in the cord (yellow arrow). Both the anterior and posterior longitudinal ligaments (red arrows) are torn, as well as the interspinous ligaments.

Figure S115-4  One sixth of flexion teardrop fractures involve the anterosuperior aspect of the body (arrow). Note the subtle widening of the interspinous distance (asterisk).

497

CASE 116

Figure S116-3  This patient with anterior impingement had focal edema in the superolateral aspect of Hoffa’s fat pad below the level of the patella (yellow arrow).

Figure S116-4  Sagittal T2-weighted image shows the characteristic location of the edema (asterisk) between the inferior pole of the patella (yellow arrow) and the anterior surface of the lateral femoral condyle (red arrow).

Figure S116-5  Sagittal proton density-weighted image shows distortion in the posterior aspect of Hoffa’s fat pad (arrow).

Figure S116-6  T2-weighted image shows low signal intensity in the abnormality (arrow) consistent with chronic Hoffa disease.

498

CASE 117

Figure S117-3  Left hip radiograph shows a patchy area of mixed lysis and sclerosis (yellow arrow) in the proximal femur with adjacent periostitis (red arrow).

Figure S117-4  Sagittal fat-saturated enhanced T1-weighted image shows tumor extension through small cortical channels (arrows).

CASE 118

Figure S118-3  This patient shows a bowing deformity of the tibia associated with a cystic abnormality in the distal diaphysis (arrow) but no break.

499

CASE 119

Figure S119-3  Axial T2-weighted image in a patient with type 1 tear shows a thickened posterior tibialis tendon (arrow) and erosive changes in the retromalleolar region of the tibia.

Figure S119-4  Sagittal T2-weighted image shows intermediate signal intensity changes within the thickened posterior tibialis tendon (arrows).

Figure S119-5  Axial T2-weigted image in a patient with a type 3 tear shows high signal intensity within an empty tendon sheath (arrow).

Figure S119-6  Sagittal T1-weighted image shows an abrupt termination to the posterior tibialis tendon at the level of the medial malleolus (arrow).

500

CASE 120

Figure S120-3  Distention of the deep infrapatellar bursa (arrow) is not an uncommon finding in patients with acute Osgood-Schlatter disease.

Figure S120-4  Another patient with acute Osgood-Schlatter disease shows anterior soft tissue swelling, interstitial edema, enlargement of the distal patellar tendon (arrow), and mild marrow edema in the tibial tubercle and proximal tibia anteriorly.

Figure S120-5  In this patient, the characteristic peritendinous ossicle demonstrates edema (arrow).

Figure S120-6  Another patient shows numerous ossicles (­yellow ­arrow) adjacent to an irregular tibial tubercle associated with thickening of the distal patellar tendon (red arrow) and anterior soft tissue swelling.

501

CASE 121

Figure S121-4  Chest radiograph shows diffuse osteosclerosis (arrows) and cardiomegaly.

Figure S121-3  Sagittal reformatted computed tomographic image shows patchy areas of sclerosis replacing the marrow throughout the spine (arrows) and sacrum.

CASE 122

Figure S122-3  Axial proton density-weighted image shows a polylobulated mass (arrow) arising dorsal to the carpus.

502

Figure S122-4  T2-weighted image shows that the fluid-containing mass (arrow) appears to arise dorsal to the scapholunate interval. Note the grapelike clustering typical of a ganglion cyst; however, it is not unusual to see a unilocular appearance when the cyst is smaller.

CASE 123

Figure S123-3  Another patient shows findings of both osteopoikilosis and osteopathia striata with thickening of the cortex in the radius, second metacarpal base, trapezium, and scaphoid (arrows), and small osteoblastic lesions in the trapezium, scaphoid, capitate, and lunate.

Figure S123-4  Shoulder radiograph shows punctate bone islands in the glenoid (yellow arrow) and thick striated linear densities in the proximal humerus (red arrows).

CASE 124

Figure S124-3  Schmorl nodes from protrusion of the nucleus pulposus create concave defects in the end plates (arrows). Figure S124-2  Osteopenia is another common manifestation of Scheuermann disease. Note that the vertical trabeculation is conspicuous (arrows).

503

CASE 125

Figure S125-3  Osseous changes can appear highly aggressive in the acute phase, mimicking a neoplastic process. Note the rapid bone lysis about the second proximal interphalangeal joint in this patient (arrow).

Figure S125-5  Coronal short tau inversion-recovery image shows marked dorsal subcutaneous fat edema, an ulcer (asterisk), and bone marrow edema of the fifth metatarsal head (arrow).

504

Figure S125-4  The hands and feet are vulnerable to penetrating injuries. This patient had a needle fragment in the distal phalanx and developed osteomyelitis around the foreign body, resulting in bone lysis (arrow).

Figure S125-6  Enhanced fat-saturated T1-weighted image shows intense enhancement of the fifth metatarsal head and the surrounding soft tissues consistent with osteomyelitis and cellulitis (arrow).

CASE 126

Figure S126-3  Another patient with a Monteggia fracture-­ dislocation shows apex posterior angulation of the ulnar fracture (arrow) and posterior dislocation of the radial head (asterisk) ­consistent with a Bado type II injury.

Figure S126-4  This patient shows a fracture distal to the coronoid process (arrow) and lateral dislocation of the radial head (asterisk) consistent with a Bado Monteggia type III injury.

Figure S126-5  Radiograph shows Galeazzi fracture-dislocation with a midshaft fracture of the radius (arrow) and abnormal position of the ulnar head (asterisk).

Figure S126-6  Lateral radiograph shows apex dorsal angulation of the radius fracture (arrow) and dorsal ulnar head dislocation (asterisk).

505

CASE 127

Figure S127-3  Coronal computed tomographic image in a different patient shows a juvenile Tillaux fracture. Notice that the frontal view appears similar to a triplane fracture.

Figure S127-4  Sagittal image shows the absence of a posterior tibial metaphysis fracture (asterisk) and in a juvenile Tillaux fracture.

CASE 128

Figure S128-3  Axial T1-weighted image in a different patient shows numerous low signal intensity uniform-sized chondral bodies in the left hip joint (arrow).

506

Figure S128-4  Another patient shows numerous ossified intraarticular bodies (asterisk) and within a distended Baker cyst (arrow) posteriorly.

CASE 129

Figure S129-3  Axial image after intravenous contrast shows heterogeneous enhancement of the soft tissue mass (arrows) and the periosteum.

Figure S129-2  Coronal short tau inversion recovery magnetic resonance image shows signal intensity changes throughout the ulna (asterisk) and a large soft tissue mass distally (arrows).

Figure S129-4  Frontal view of the right femur shows a geographic lytic lesion in the femoral shaft with mild endosteal scalloping of the medial cortex (arrow).

Figure S129-5  Short tau inversion recovery sequence shows that the Ewing lesion is much larger than suspected from the radiograph, with associated periostitis (arrows).

507

CASE 130

Figure S130-2  Sagittal computed tomographic image shows that the avulsion fragment is attached to the anterior longitudinal ligament (arrow).

Figure S130-3  This young patient sustained a more severe hyperextension trauma. Note the prevertebral soft tissue swelling (red arrow) and the teardrop fracture involving the C3 vertebra (yellow arrow).

CASE 131

Figure S131-3  Axial T2-weighted image with fat saturation shows involvement of the central third of the tendon (arrow) with thickening. Tendinosis tends to involve the medial or central third of the tendon because the physiologic valgus angle in the knee results in greater tensile stress on the medial side of the tendon. Figure S131-4  Fat saturation is a nice magnetic resonance technique to make the edema in the tendon and the patella (arrow) more conspicuous in cases of suspected partial tears.

508

CASE 132

Figure S132-3  This patient has a more proximally located bisphosphonate fracture with the typical periosteal and endosteal reactive changes (arrows) and a transverse fracture through the lateral cortex.

Figure S132-4  In patients who present with a complete fracture, be sure to look for the volcano-like periosteal reaction either on the initial film (arrow) or on the postreduction radiograph. If observed, the other leg should be imaged.

CASE 133

Figure S133-3  Sagittal T2-weighted image shows interstitial edema in the proximal aspect of the posterior cruciate ligament (arrow). Figure S133-4  This patient had a distal avulsion of the posterior cruciate ligament (arrow).

509

CASE 134

Figure S134-4  Axial T2-weighted image on another patient shows a tear through the medial patellar retinaculum (arrow) and extensive anterior soft tissue swelling. Figure S134-3  Axial T2-weighted image on a different patient shows disruption of the medial patellofemoral ligament from its femoral attachment with retraction (yellow arrow). There is also a defect in the vastus medialis aponeurosis adjacent to the medial collateral ligament (red arrow).

Figure S134-5  Axial T2-weighted image shows a large chondral defect in the medial patellar facet (yellow arrow). In this situation, it is important to identify the location of the chondral fragment (red arrow).

510

Figure S134-6  Axial computed tomographic image shows an osteochondral fragment in the lateral aspect of the suprapatellar bursa (arrow).

CASE 135

Figure S135-3  Magnetic resonance imaging is ideal for depicting the periarticular soft tissue involvement. Note the abscess (asterisk) elevating the anterior longitudinal ligament (arrows).

Figure S135-4  This patient has classic skip lesions involving the L2-3 and L4-5 disk levels. Both the anterior longitudinal (yellow arrow) and posterior longitudinal (red arrow) ligaments are involved, as well as the paraspinal soft tissues (blue arrows).

CASE 136

Figure S136-3  This patient had more severe dysplasia with enlargement of the femoral head (coxa magna) and thickening of the femoral neck associated with valgus angulation of the femoral angle (asterisk). The acetabular inclination is steep and the fossa appears shallow (arrows).

Figure S136-4  A different patient shows typical residual changes from untreated congenital hip dislocation, with high-riding femoral head articulating with pseudoacetabulum (yellow arrow) in the lateral aspect of the iliac bone. Note that native acetabulum is poorly formed (red arrow).

511

CASE 137

Figure S137-3  Coronal reformatted computed tomographic image shows superior subluxation of the left clavicular head and an intraarticular fracture of the head with a displaced fragment (arrow).

Figure S137-5  This patient had a posterior dislocation of the right clavicular head (arrow). The relationship of the head of the clavicles with the adjacent structures posterior to them is important and includes the vessels, nerves, trachea, and esophagus.

512

Figure S137-4  A lordotic view can be helpful in emphasizing the asymmetry of the position of the clavicular heads. Note the medial dislocation of the left clavicle (arrows) in this patient.

CASE 138

Figure S138-3  Coronal T2-weighted image shows the ribbonlike deformity of the retracted tendon (arrow).

Figure S138-4  Axial T2-weighted image in a patient with a partial tear shows attenuation of the biceps tendon (yellow arrow) at the radial tuberosity and distention of the bicipital bursa (red arrow).

CASE 139

Figure S139-4  This patient had a broad-based subungual exostosis (arrow).

Figure S139-3  Oblique radiograph shows lack of cortical continuity (arrow).

513

CASE 140

Figure S140-3  A compressive stress fracture (arrows) that extends through a large area of the neck has an increased risk of fracture.

Figure S140-4  The tensile type of stress fracture is difficult to detect and may present with a complete fracture (arrow). Prior radiographs showed progressive osteopenia until the bone failed.

CASE 141

Figure S141-3  Axial T1-weighted image with the patient in dorsiflexion shows persistent anterior dislocation of the peroneus longus tendon (yellow arrow) and a portion of the disrupted superior peroneal retinaculum (arrowhead).

514

Figure S141-4  Sagittal image shows the tendon over the anterior surface of the fibula (arrows) instead of its normal posterior location.

CASE 142

Figure S142-3  Axial fat-saturated T1-weighted image after intravenous administration of gadolinium shows extensive liquefaction of the musculature (asterisk) with rim enhancement. Numerous bubbles are present (arrows) and manifest as ovoid regions of signal void.

Figure S142-2  Another patient presented with extensive gas forming in the soft tissues (arrows) and a markedly swollen calf.

Figure S142-4  T2-weighted image in another patient shows extensive edema in the vastus medialis, adductor longus, gracilis, and sartorius muscles (asterisk).

Figure S142-5  After intravenous contrast, there is enhancement of the deep fascia and some of the adjacent musculature. Note the gas bubbles within the soft tissues (arrows).

515

CASE 143

Figure S143-3  Proton density-weighted image shows trabecular impaction adjacent to the deepened lateral notch (arrow). Note the anterior drawer sign.

Figure S143-4  Sagittal proton density-weighted image shows a fracture of the posterolateral tibia (arrow).

CASE 144

Figure S144-3  This patient had a typical wedge-like defect involving the distal articular surface of the third metacarpal head (arrow). Note that the joint space is normal and there is no change in the bone density yet.

516

Figure S144-4  Another patient with a fight bite with reactive sclerosis surrounding the impaction fracture (arrow).

CASE 145

Figure S145-3  In an oblique-posterior fracture (arrow), the odontoid process fragment can displace posteriorly.

Figure S145-4  In an oblique-anterior fracture (arrow), the odontoid process fragment can displace anteriorly. In horizontal fractures, displacement may be either anterior or posterior.

CASE 146

Figure S146-3  There is acro-osteolysis of the distal aspects of the toes (arrows).

Figure S146-4  The wrist in another patient shows erosive arthropathy of the first carpometacarpal joint and soft tissue calcifications. The distal ulna has a tapered appearance from the erosions (arrow).

517

CASE 147

Figure S147-3  Coronal T1-weighted image in a different patient shows a lateral discoid meniscus with a horizontal tear and a large parameniscal cyst (arrow).

Figure S147-4  Sagittal image in another patient with a lateral discoid meniscus shows a defect in the anterior horn (arrow).

Figure S147-5  This patient had a lateral discoid meniscus with a radial tear (arrow) and a horizontal tear of the anterior horn with a parameniscal cyst (asterisk).

Figure S147-6  Example of a meniscal flounce (arrow), a normal fold in the meniscus created by rotation of the knee.

518

CASE 148

Figure S148-3  T2-weighted image shows effusions with diffuse synovitis resulting in marginal erosions (arrows) surrounded by bone marrow edema.

Figure S148-4  Lateral view of patient with severe deformity of arthritis mutilans (arrow).

CASE 149

Figure S149-3  This patient had significant distention of the sheath of the flexor hallucis longus tendon with fluid (arrow). Note the typical features of os trigonum syndrome in the bones. Figure S149-4  This patient sustained a fracture of the medial tubercle of the posterior talus process. The bone fragment mimics an os trigonum (arrow).

519

CASE 150

Figure S150-3  Pelvic radiograph shows squared iliac bones with flat acetabulum and curved “telephone handle” femora with flared metaphysis (arrows).

CASE 151

Figure S151-3  Three months later, the fracture has become more displaced (arrow).

520

Figure S151-4  Nearly 1 year after surgery, the acromion process fracture (arrow) has become further displaced laterally and inferiorly, severely affecting the mobility of the arthroplasty.

CASE 152

Figure S152-3  The left forearm shows a permeative pattern of bone destruction in the ulna, periosteal reactive changes, and a prominent focus of bone lysis with cortical disruption (arrow). This patient had melanoma metastasis.

Figure S152-4  The ulna shows tiny, lytic areas of bone lysis with poor margination and cortical tunneling that characterizes the permeative pattern. Note the pathologic fracture proximally (arrow).

Figure S152-6  There is a large lytic lesion destroying much of the acetabulum (arrows). This patient had colon cancer.

Figure S152-5  This patient developed a pathologic fracture through a permeative lytic lesion in the proximal left humerus (­arrow). She had breast cancer.

521

CASE 153

Figure S153-3  Another patient with a larger fragment of bone (yellow arrow) and a clear donor site for the acute fracture fragment (red arrow).

Figure S153-4  The olecranon bursa was distended (arrow) in this patient with a triceps tendon avulsion.

Figure S153-6  Corresponding axial T2-weighted image to Figure S153-5 shows interstitial edema at the musculotendinous junction (arrow).

Figure S153-5  Sagittal T2-weighted image in a patient with a partial tear of the triceps tendon involving the medial head. There is a tensile contusion in the olecranon process (arrow).

522

CASE 154

Figure S154-4  Axial computed tomographic image shows that the orientation of a column fracture is in the medial to lateral direction (arrows) at the level of the tectum.

Figure S154-3  A three-dimensional computed tomographic image shows the transverse component of the fracture (asterisk), disruption of the posterior wall (yellow arrow), and disruption of the anterior column (red arrows).

Figure S154-5  Axial computed tomographic image shows that the orientation of a transverse fracture is in the anterior to posterior direction (arrows).

523

CASE 155

Figure S155-2  Radiograph of another patient shows an area of bone lysis where the ulnar collateral ligament pulled off the sublime tubercle (arrow). Figure S155-3  Coronal short tau inversion recovery image shows interstitial edema in the ulnar collateral ligament (arrow) and a bone contusion (asterisk) in the capitellum consistent with a valgus injury of the elbow.

Figure S155-4  Coronal short tau inversion recovery image in another athlete shows a tear of the proximal attachment of the anterior band of the ulnar collateral ligament (yellow arrow), bone edema in the medial epicondyle and capitellum (red arrow), and edema in the flexor digitorum superficialis muscle.

Figure S155-5  This patient shows a tear in the middle of the anterior band (yellow arrow) and marked edema in the flexor digitorum superficialis muscle (asterisk).

CASE 156

Figure S156-3  A partial gluteus medius tear may manifest as an area of tendinous attenuation (arrow). Note the adjacent area of marrow edema in the greater trochanter.

Figure S156-4  Sagittal image shows partial avulsion of the anterior fibers of the gluteus medius tendon (arrow).

Figure S156-5  This patient had a complete tear of the gluteus minimus insertion (yellow arrow) and distention of the trochanteric bursa (red arrow).

Figure S156-6  Calcific tendinitis of the gluteal cuff (arrow) is a straightforward radiographic diagnosis.

525

CASE 157

Figure S157-3  If the fracture heals, it often results in abnormal morphology of the calcaneus with variably sized bony protuberances (arrow).

Figure S157-2  Another patient with a typical neuropathic avulsion fracture. Note that the fragment (asterisk) usually affects the superior portion of the calcaneal tuberosity (arrow).

CASE 158

Figure S158-3  This rheumatoid arthritis patient shows vertical subluxation, or cranial settling, of the atlantoaxial articulation. Pannus destruction of the ligaments and joints causes the ring of the atlas to descend down on the odontoid process (arrow).

526

Figure S158-4  This patient shows the effects of platybasia. The tip of the odontoid process is herniating into the foramen magnum (arrow) and violates the Chamberlain line.

CASE 159

Figure S159-3  Coronal short tau inversion recovery image shows a polylobular lesion with heterogeneous signal intensity that is larger than was appreciated on the radiograph (arrows).

Figure S159-4  Coronal fat-saturated T1-weighted postgadolinium image shows enhancement of the lesion peripherally and in the stroma between the chondroid nodules. Note the “puddling” of contrast typical of chondrosarcoma (arrow).

Figure S159-5  Axial image shows cortical infiltration where there was endosteal scalloping (arrow) with adjacent periostitis.

Figure S159-6  Bone scan image shows increased uptake in the lesion and the periosteal reactive changes (arrow).

527

CASE 160

Figure S160-3  Another typical feature of ulnar abutment syndrome is cystic changes in the subchondral region of the lunate medially (arrow).

Figure S160-4  This patient had severe impaction, resulting in flattening and sclerosis of the adjacent surfaces of the ulna and lunate (arrow).

Figure S160-5  This patient had features of both ulnar abutment syndrome (yellow arrow) and ulnar styloid impaction syndrome. Note the nonunion fracture of the ulnar styloid (red arrow).

Figure S160-6  Corresponding T2-weighted image to Figure S160-5 shows a small defect in the articular surface of the triquetrum where the tip of the styloid process struck against it (arrow).

528

CASE 161

Figure S161-3  Coronal reformatted computed tomographic image shows the full extent of the defect in the articular surface of the femoral head (yellow arrow). There was also a second bone fragment (red arrow).

Figure S161-4  Axial computed tomographic image shows the anterior aspect of the femoral head fracture (arrow).

Figure S161-5  Frontal view of the left hip shows a vertically oriented lucency through the medial aspect of the femoral head consistent with a fracture (arrows).

529

CASE 162

Figure S162-3  Patient with scapholunate dissociation. Note the widening of the scapholunate interval (arrow), referred to as the “David Letterman” sign, indicative of a tear of the scapholunate ligament.

Figure S162-4  This radiograph shows another patient with a ­volarly tilted lunate (arrow).

Figure S162-5  Lateral view confirms a perilunate dislocation with a volarly tilted but articulated lunate (asterisk) and a dorsally dislocated capitate (arrows).

Figure S162-6  Naviculocapitate syndrome. Note the fracture of the scaphoid (arrow) and fracture of the capitate with a rotated proximal pole (asterisk) producing an “igloo” sign. This injury is usually associated with a perilunate dislocation as well.

530

CASE 163

Figure S163-3  Coronal T1-weighted image shows lipomatous tissue floating in the suprapatellar bursa (arrow), which is markedly distended with fluid.

Figure S163-4  Sagittal fluid-sensitive image shows the frondlike subsynovial nature of the adipose tissue (arrows).

CASE 164

Figure S164-4  A gravity stress view shows lateral subluxation of the talus increasing the medial clear space (arrow), indicative of a syndesmotic injury.

Figure S164-3  This patient has a Weber B2 injury. There is an oblique fibular fracture at the level of the ankle joint (yellow arrow) and disruption of the deltoid ligament resulting in widening of the medial tibiotalar joint space (red arrow).

531

CASE 165

Figure S165-3  Coronal T1-weighted image shows a narrow zone of transition (arrow) with sharp demarcation with the normal bone marrow.

Figure S165-4  Sagittal fat-saturated T1-weighted image after contrast shows minimal peripheral enhancement (arrow).

Figure S165-5  This patient shows an involuting fibroxanthoma with sclerotic areas of new bone that is filling in the lesion (arrow).

Figure S165-6  On magnetic resonance, an involuting lesion will eventually become low in signal intensity (arrow).

532

CASE 166

Figure S166-2  Axial T2-weighted image in another patient shows acute compartment syndrome involving the lateral compartment (arrow) manifested as increased T2 signal intensity and fascial edema.

Figure S166-3  Gadolinium-enhanced image shows homogeneous enhancement of the peroneal muscles (arrow) indicating viability.

Figure S166-4  This patient had delayed muscle soreness after running a marathon, manifested as increased T2 signal intensity in different muscles (arrows).

Figure S166-5  Coronal T2-weighted image in a patient shows rhabdomyolysis of the quadriceps muscles (arrows) after a squatting competition. The creatine kinase level was 20 times greater than normal.

533

CASE 167

Figure S167-3  The typical location of an elastofibroma dorsi (asterisk) is deep to the serratus anterior (yellow arrow) and latissimus dorsi (red arrow) muscles.

Figure S167-4  Axial proton density-weighted image shows the streaky high signal intensity of fat within the elastofibroma dorsi (arrow).

CASE 168

Figure S168-3  Axial T2-weighted image shows a complete tear of the musculotendinous junction (asterisk). Note the intact tendon pedicle attached to the humerus (arrow).

534

Figure S168-4  Lower axial image shows the hemorrhagic fluid (asterisk) tracking inferiorly on the chest and arm, resulting in the characteristic ecchymosis pattern seen with this injury.

Figure S168-5  This patient had a large intramuscular tear involving the clavicular head (asterisk) but the tendon (arrow) and myotendinous junction were intact.

Figure S168-6  There is interstitial edema in the inferior aspect of the sternal head (arrow) in this patient with a grade 2 muscle strain.

CASE 169

Figure S169-3  Coronal T1-weighted image shows hypertrophy of the short radiolunate ligament (arrow).

Figure S169-4  The deformity is more pronounced in this patient. The radial tilt is severe (arrow), and there is a significantly longer ulna (asterisk).

535

CASE 170

Figure S170-2  Anteroposterior radiograph in another patient with posttraumatic osteonecrosis shows the tip of a dynamic hip screw (used to stabilize a femoral neck fracture) penetrating through the femoral head surface, resulting in mechanical erosive changes in the adjacent acetabulum (arrow). Figure S170-3  Increased displacement and angulation of the subtrochanteric fracture are causing the dynamic screw to back out (yellow arrow) and penetrate through the femoral head (red arrow).

Figure S170-4  Impaction of the peritrochanteric fracture has caused the femoral neck screw (arrow) to nearly completely disengage from the rod.

536

Figure S170-5  Angulation of the subtrochanteric fracture has caused the intramedullary rod to bend and break (arrow).

CASE 171

Figure S171-2  Sagittal T2-weighted image shows a gap in the tendon allowing the fluid in the joint (asterisk) to communicate with the prepatellar bursa (arrow). Note the low-lying patella.

Figure S171-3  Sagittal T2-weighted image in a patient with a partial quadriceps tendon tear. Note that only the vastus intermedius component (arrow) is intact.

CASE 172

Figure S172-4  This patient had a Salter type 4 injury with a fracture of the metaphysis (yellow arrow) and epiphysis (red arrow) at the base of the second proximal phalanx.

Figure S172-3  This patient had an impaction injury of the thumb with a Salter type 2 fracture of the distal phalanx (arrow). These types of fractures are at risk for infection if a subungual hematoma is drained using a nonsterile technique.

537

CASE 173

Figure S173-3  Another patient shows a linear lucency in the anterior process (arrow).

Figure S173-5  This patient had a large fracture fragment (arrow).

538

Figure S173-4  Sagittal proton density image shows that the calcaneocuboid limb (yellow arrow) of the bifurcate ligament is attached to the bone fragment (red arrow).

Figure S173-6  Sagittal short tau inversion recovery image shows intense marrow edema in the calcaneus (asterisk) and fracture fragment (arrow). This patient had a dorsiflexion “nutcracker” injury mechanism.

CASE 174

Figure S174-3  Coronal proton density-weighted image shows a longitudinal split of the peroneus brevis tendon (arrow).

Figure S174-4  Consecutive axial T1-weighted images show the split components of the peroneus tendon (arrows) coalescing into one tendon at the level of the distal calcaneus.

CASE 175

Figure S175-3  Axial T2-weighted image in another patient shows a heterogeneous ovoid mass in the suprapatellar bursa (arrow), the second most common location for this lesion.

Figure S175-4  Sagittal T1-weighted enhanced image shows that the lesion minimally enhances (arrow) in this patient, but it may occasionally show marked enhancement.

539

CASE 176

Figure S176-3  Glenolabral articular disruption lesion with a long cleft (arrow).

Figure S176-4  Glenolabral articular disruption lesion with a short cleft (arrow).

Figure S176-5  Glenolabral articular disruption lesions may have marked articular cartilage surface irregularity (arrow).

Figure S176-6  A glenolabral articular disruption lesion may be associated with an osseous defect (arrow).

540

CASE 177

Figure S177-3  Coronal T1-weighted image shows the growth of the osteochondroma (arrow).

Figure S177-4  Axial T2-weighted image shows a markedly thickened cartilage cap (arrows) which had transformed into a chondrosarcoma eroding the underlying bone. The surrounding high-T2intensity fluid is within a pseudobursa.

Figure S177-5  Axial T1-weighted fat-saturated image after gadolinium shows heterogeneous enhancement typical of a chondrosarcoma (arrow).

Figure S177-6  Another image closer to the bone shows heterogeneous enhancement of the cap (arrow).

541

CASE 178

Figure S178-3  Sagittal T1-weighted image in a normal patient shows the normal configuration of the flexor digitorum tendons and the pulley apparatus (arrows show A1, A2, A3, A4, and A5 from proximal to distal).

CASE 179

Figure S179-3  Axial computed tomographic image of the chest shows absence of the scapula where the rotator cuff muscles are located (arrow).

542

Figure S178-4  Sequential sagittal T1-weighted images show a flexion deformity of the proximal interphalangeal joint and an anteriorly bowed flexor digitorum tendon (arrow).

CASE 180

R S P

Figure S180-3  Fat-saturated T1-weighted image after intravenous gadolinium shows areas of nodular and peripheral enhancement (arrows).

Figure S180-4  Lateral ankle radiograph shows a large calcified mass in the Kager fat pad that was a biopsy-proven synovial cell sarcoma.

Figure S180-5  Axial proton density magnetic resonance image in another patient shows septations (arrow) within a lesion and a low signal intensity pseudocapsule surrounding the mass.

Figure S180-6  T2-weighted image shows high signal intensity, giving the appearance of a fluid cavity. Peritumoral edema is seen above the lesion (arrow).

543

CASE 181

Figure S181-3  Remodeling of the cortex of the third and fourth proximal phalanges is evident (arrows).

Figure S181-5  Axial fat-saturated enhanced T1-weighted image shows marked enhancement of the hypertrophied synovium in the involved tendons (arrow).

544

Figure S181-4  Remodeling of the bones can result in a tubular appearance (arrow).

CASE 182

Figure S182-4  Axial T2-weighted image shows edema in the adductor muscle group (asterisk). Figure S182-3  Coronal T1-weighted image shows architectural distortion of the left thigh musculature (arrow).

Figure S182-5  Axial T2-weighted image in another diabetic patient with diabetic muscle infarction shows edema in the quadriceps and adductor muscles (asterisks).

Figure S182-6  Fat-saturated T1-weighted image after gadolinium shows devitalized muscles with lack of enhancement (arrow).

545

CASE 183

PSL

Figure S183-4  Enhanced coronal fat-saturated T1-weighted image shows heterogeneous enhancement of the mass (arrow) and the infiltrative nature of the lesion.

Figure S183-3  Axial T1-weighted image shows a nodular mass of intermediate signal intensity in the plantar fascia in a patient with plantar fibromatosis (asterisk).

CASE 184

Figure S184-4  Neonate with bilateral proximal focal femoral deficiency. The right femur did not develop and the left is only partially formed (arrows) (Aitken type D).

Figure S184-3  Anteroposterior radiograph of the hip in another patient shows an Aitken type B proximal focal femoral deficiency with a pseudoarthrosis between the head and shaft (arrow), varus angulation, and a moderately dysplastic acetabulum.

546

CASE 185

Figure S185-3  Ischiofemoral impingement syndrome is a bilateral process in many patients. Note the atrophic quadratus femoris muscles (arrows) in both hips.

Figure S185-4  Interstitial edema in the quadratus femoris muscle is characteristic early in the disease process (arrow).

< 17mm Figure S185-5  The ischiofemoral space is narrowed when the distance between the lesser trochanter and ischium (red lines) is less than 17 mm.

Figure S185-6  The quadratus femoris space is narrowed when the distance between the lesser trochanter and hamstring tendon (red lines) is less than 8 mm.

547

CASE 186

Figure S186-3  Coronal T1-weighted image shows mass-like fullness in the periarticular region of the hip arthroplasty (arrows) partly related to fibrosis.

Figure S186-4  Corresponding T2-weighted image shows low signal intensity (arrows) in areas of metallosis. Note that bone lysis also shows signal intensity consistent with metallosis (asterisk).

Figure S186-5  Routine follow-up radiograph shows metal-on-metal arthroplasty in appropriate alignment. Note heterotopic ossification laterally (arrow).

Figure S186-6  Two years later, there is pronounced bone lysis surrounding the acetabular cup (arrows), which has shifted and appears vertically tilted.

548

CASE 187

Figure S187-3  The hand radiograph shows hemihypertrophy of the lateral aspect of the hand and wrist (asterisk) involving the bones, muscles, and connective tissues. Figure S187-4  Frontal radiograph of the left foot in a patient with macrodystrophia lipomatosa shows osseous overgrowth of the bones in the first ray (asterisk). Note the splaying of the end of the distal phalanx. The surrounding soft tissues are enlarged and lobulated.

CASE 188

IAL Figure S188-4  Patient with rice bodies in the flexor tendons (arrow).

Figure S188-3  Coronal T2-weighted image shows the tubular appearance of the components of the hemangioma (arrow). It lacks the distinctive oval morphology seen with intraarticular bodies.

549

CASE 189

Figure S189-3  Axial T2-weighted image shows hyperintense signal changes in the humerus bone marrow (asterisk) and numerous soft tissue masses along the chest wall related to extramedullary hematopoiesis (arrows).

SR

Figure S189-4  Sagittal T1-weighted image of the spine demonstrates diffusely low signal intensity in the bone marrow of the vertebral bodies (asterisks).

SR

IL Figure S189-5  Another patient with acute myeloid leukemia shows low signal intensity marrow replacement in the humerus (asterisks).

550

Figure S189-6  Corresponding T2-weighted image shows a pathologic fracture in the greater tuberosity (arrow).

CASE 190

Figure S190-3  Axial proton density image in another patient shows a fluid collection at the gluteal–subcutaneous fat interface with a well-defined pseudocapsule (arrow).

Figure S190-5  Axial proton density image in a third patient shows a degloving injury isolated to the subcutaneous fat with floating necrotic debris (arrow).

Figure S190-4  Coronal short tau inversion recovery image shows the full extent of the Morel-Lavallée lesion (asterisk) in a typical location over the greater trochanter.

Figure S190-6  Corresponding T2-weighted image shows the low signal intensity capsule (arrows) indicative of a more chronic lesion.

551

CASE 191

Figure S191-3  Coronal short tau inversion recovery image in a different tennis player shows the characteristic tubular-shaped fluid accumulation between the soleus and medial gastrocnemius muscles (arrows).

Figure S191-4  Axial T2-weighted image shows hemorrhagic fluid between edematous soleus and medial gastrocnemius muscles (arrow). The “blooming” was the result of hemosiderin.

CASE 192

Figure S192-3  Radiograph of the other shoulder demonstrates less severe deformity of the right humeral head (arrow).

552

Figure S192-4  Hip radiograph in a child shows small, irregular femoral epiphysis (arrow).

CASE 193

Figure S193-3  Axial T1-weighted fat-saturated postgadolinium image shows variable enhancement in the periphery of the lesion and a large anterior soft tissue mass (asterisk).

Figure S193-4  Sagittal contrast-enhanced T1-weighted image shows enhancing tumor nodules in the inferior aspect of the lesion (arrow) and the periphery.

CASE 194

Figure S194-3  Frontal radiograph of the hand of a neonate shows an obvious deformity. There is absence of the left thumb (asterisk) and autoamputations beyond the middle phalanges of the third through fifth fingers (arrow).

Figure S194-4  This patient has abnormal development of the first proximal phalanx and autoamputation of the second through fifth toes, with absence of the distal phalanges mimicking acro-osteolysis (arrows).

553

CASE 195

Figure S195-3  Another patient shows synovial hypertrophy in the axillary recess but no capsular thickening (arrow), indicating an earlier stage of adhesive capsulitis.

Figure S195-4  Sagittal image shows the synovitis surrounding the structures in the rotator interval and biceps sling (arrow).

Figure S195-5  Sagittal T1-weighted image in another patient shows marked synovial hypertrophy throughout the glenohumeral joint (asterisks).

Figure S195-6  Sagittal T2-weighted image at level of rotator interval shows the characteristic changes of adhesive capsulitis (arrow).

554

CASE 196

Figure S196-4  Computed tomography confirms the findings in both proximal femora (arrows). Figure S196-3  Anteroposterior pelvic radiograph shows symmetric osteosclerosis of both femora. Note the areas of bone lysis within the medullary cavity (arrows).

Figure S196-5  Coronal T1-weighted image of both femora shows that the medullary cavity in the diaphysis and metaphysis has been replaced by low signal intensity tissue (asterisks).

Figure S196-6  Corresponding short tau inversion recovery image shows numerous foci of increased signal intensity within the marrow. Periostitis (arrows) was depicted as linear high signal intensity adjacent to the cortical margins of the bone.

555

CASE 197

Figure S197-3  Fat-saturated T1-weighted image after contrast shows variable enhancement and areas of necrosis (asterisk).

Figure S197-4  Coronal T1-weighted image in another patient shows a thigh mass with heterogeneous intermediate to low signal intensity (arrow), although the variability is less pronounced than in the first patient.

Figure S197-5  Axial T2-weighted image shows a well-circumscribed lesion with areas of high and intermediate signal intensity (arrow).

Figure S197-6  Fat-saturated T1-weighted image after contrast shows areas of necrosis (arrows).

556

CASE 198

Figure S198-2  Radiograph of an older child shows the characteristic bowing deformity of the ulna (arrow) and absence of the thumb in a patient with thrombocytopenia–absent radius syndrome.

CASE 199

Figure S199-3  The hand radiograph shows erosive changes in the second and third proximal interphalangeal joints and the second and fifth metacarpophalangeal joints (arrows), similar to those of rheumatoid arthritis. Note the arthritis mutilans in the wrist.

Figure S199-4  Shoulder radiograph shows marginal erosions (arrow) in the humerus and soft tissue fullness in the joint space (asterisk).

557

Figure S199-5  Coronal T1-weighted image in another patient shows intermediate soft tissue masses where there are erosions (arrow). Figure S199-6  Sagittal T2-weighted image shows intense diffuse synovitis (asterisk).

CASE 200

Figure S200-3  Nora lesion can be quite sizeable and show local invasion (arrows).

558

Figure S200-4  Patient with florid reactive periostitis of the second proximal phalanx (arrow). This condition can progress to bizarre parosteal osteochondromatous proliferation in some patients.